0% found this document useful (0 votes)
189 views181 pages

Pathology Final Exam Corrected

Uploaded by

Alenta Kuriakose
Copyright
© © All Rights Reserved
We take content rights seriously. If you suspect this is your content, claim it here.
Available Formats
Download as PDF, TXT or read online on Scribd
0% found this document useful (0 votes)
189 views181 pages

Pathology Final Exam Corrected

Uploaded by

Alenta Kuriakose
Copyright
© © All Rights Reserved
We take content rights seriously. If you suspect this is your content, claim it here.
Available Formats
Download as PDF, TXT or read online on Scribd
You are on page 1/ 181

Final 2021

1. A 12-year-old girl has complained of a sore throat for the past 3 days. On physical
examination, she has a temperature of 38.4° C and pharyngeal erythema with
minimal exudate. A throat culture grows group A β-hemolytic Streptococcus. The
pharyngitis resolves, but 3 weeks later, the girl develops fever and chest pain.
Her anti–streptolysin O titer is 1:512. Which of the following immunologic
mechanisms has most likely led to the chest pain?
A. Breakdown of T cell anergy
B. Failure of T cell–mediated suppression
C. Molecular mimicry
D. Polyclonal lymphocyte activation
E. Release of sequestered antigens

2. A 25-year-old man presents for a routine physical examination. The


patient is tall (6 ft, 5 in) and has long fingers (shown in the image). One
year later, he suffers a dissecting aortic aneurysm. This patient most likely
carries a mutation in a gene that encodes which of the following proteins?
A. Collagen
B. Dystrophin
C. Elastin
D. Fibrillin
E. Myosin
Marfan Syndrome → fibrillin

3. A 40-year-old woman presents with a painless lesion on her right ear lobe (PIC 6).
She reports that her ears were pierced 4 months ago. The most likely diagnosis is
A. Keloid
B. Hypertrophic scars
C. Normal scarring
D. Wound dehiscence

4. Molluscom contagiosum

5. Transduction
6. Polycystic Kidneys

7. Melanoma

8. A 45-year-old man presents with increasing abdominal girth and yellow discoloration
of his skin and sclera. Physical examination reveals hepatomegaly and jaundice. A
Prussian blue stain of a liver biopsy is shown in the image. What is the major
intracellular iron storage protein in this patient’s hepatocytes?
A. Bilirubin
B. Haptoglobin
C. Hemoglobin
D. Hemosiderin
E. Transferrin

9. Soon after birth, a term infant develops tetany with marked hypocalcemia. This is
treated, but at one month of age, a systolic heart murmur is heard on auscultation
of the chest. Later in infancy, it is noted that the baby has been almost constantly ill
with one infection after another, including respiratory syncytial virus, Candida, and
Pneumocystis carinii (jirovecii) pneumonia diagnosed. Which of the following
primary immunodeficiency disorders is the child most likely to have?
A. DiGeorge anomaly
B. Severe combined immunodeficiency
C. Common variable immunodeficiency
D. Bruton disease
E. Hyper IgM syndrome
10. A 9-month-old girl with a history of recurrent pulmonary infections is found to
have a congenital defi ciency of adenosine deaminase, which is associated with
a virtual absence of lymphocytes in her peripheral lymphoid organs. What is
the appropriate diagnosis?
A. Bruton X-linked agammaglobulinemia
B. DiGeorge syndrome
C. Isolated IgA defi ciency
D. Severe combined immunodeficiency

11. A 26-year-old woman presents to the dermatology clinic for fatigue, weakness, and
fevers for the past month. She reports significant weight loss despite eating a normal
diet. She reports that she sunburns very easily and has a facial rash that is hard to
cover with makeup. On physical exam, she has a butterfly rash with nasolabial
sparing on her face, several discoid lesions on her fingers, and a erythematous rash
on her chest in a V-neck distribution. On laboratory exam, she has a highly positive
antinuclear antibody and positive anti-double-stranded DNA antibody.What is the
most likely diagnosis?
A. Systemic lupus erythematosus → (SLE)
B. Sjogren's syndrome
C. Sclerodermia
D. Marfans Syndrome

12. A 14-year-old girl with amenorrhea is concerned because of the delayed onset of
menses. She has shortened stature and a wide, webbed neck; broad chest; and
secondary sexual characteristics consistent with those of a much younger girl. Which
of the following chromosomal changes is most consistent with these findings?
A. 5p-
B. 22q11-
C. 45,XO
D. 46,XY

13. On day 28 of her menstrual cycle, a 23-year-old woman experiences onset of


menstrual bleeding that lasts for 6 days. She has had regular cycles since
menarche. Which of the following processes most likely occurs in her endometrial
cells to initiate the onset of menstrual bleeding?
A. Apoptosis
B. Atrophy
C. Caseous necrosis
D. Liquefactive necrosis

14. In an experiment, thrombus formation is studied in areas of vascular damage. The


propagation of a thrombus in an area of vascular injury to adjacent normal arteries
is prevented. Which of the following substances diminishes thrombus propagation
by activating protein C?
A. Platelet factor 4
B. Prothrombin
C. Thrombomodulin
D. Tumor necrosis factor (TNF) 13
15. A neonate born to 41-year-old woman in her 39th w eek of gestation has a
flattened face and epicanthal folds, oblique palpebral fissures and a single palmar
crease. The child's echocardiography reveals an endocardial cushion defect. Which
of the following most likely occurred prior to conception?
A. Meiotic non-disjunction
B. Expansion of trinucleotide repeats
C. Deletion of chromosomal part
D. Robertsonian translocation

16. A 15-year-old patient is referred to your office by a teacher who is concerned


about the patient's learning abilities. After evaluating the patient, you diagnose
mild mental retardation. Cytogenetic studies of the patient's buccal mucosal cells
reveal a 47 XXY karyotype. Further evaluation is likely to reveal:
A. Short stature, broad chest, amenorrhea
B. Macroorchidism, large jaw and ears
C. Tall stature, gynecomastia, infertility
D. Short stature, hypotonia, obesity

17. An infant born prematurely to a 42-year-old Caucasian female is small for


gestational age. Physical examination reveals microcephaly, low-set ears,
prominent occiput and small mandible. The infant's fists are clenched and the
fingers overlap. A bilateral foot deformity is observed. Which of the following is
the most likely karyotype abnormality in this infant?
A. Trisomy 21
B. Trisomy 13
C. Trisomy 18
D. 47,XXY

18. A stillborn fetus delivered at the 23rd week of gestation is found to have an
edematous neck and broad chest. Autopsy reveals aortal coarctation, a bicuspid
aortic valve, and kidneys that are fused at the midline. Which of the following is
the most likely karyotype abnormality in this fetus?
A. Trisomy 21
B. Trisomy 18
C. Trisomy 13
D. 47,XXY
E. 45,XO

19. An infant born prematurely to a 38-year-old Caucasian female is small for


gestational age. Physical examination reveals a bilateral cleft lip, microcephaly,
and microphthalmos. Viscera protrude from an umbilical opening in the child's
abdominal wall. Which of the following karyotypes is most likely in this case?
A. Trisomy 18
B. Trisomy 21
C. Trisomy 13
D. Marfan syndrome
20. A 5-year-boy has developed features that suggest puberty over the past 6
months. On physical examination, the boy has secondary sex characteristics,
including pubic hair and enlargement of the penis. Which of the following
morphologic features is most likely to be seen in his adrenal glands?
A. Cortical atrophy
B. Cortical hyperplasia
C. Cortical nodule
D. Medullary atrophy
E. Medullary hyperplasia
F. Medullary nodule

21. A 3-year-old child is being evaluated by his physician during an office visit. Over the
past 2 years, the child has had several bouts of ostitis media and one severe case of
pneumonia, which required hospitalization. At the time of the examination, the
physician also notes that the child’s skin is generally very white. The child’s mother
mentions that one of the child’s grandmother’s sisters died at a young age from
recurrent infections. After further laboratory testing, the physician makes a
diagnosis of
A. severe combined immunodeficiency syndrome
B. chronic granulomatous disease
C. hyper-IgM syndrome
D. Chédiak-Higashi syndrome

22. A 50-year-old man presents to the hospital after a briefბepisode of chest wall pain.
An ECG and cardiac enzyme tests reveal no abnormalities. A routine chest
radiograph reveals only a single 2-cm coin lesion in the right lower lobe of the lung
with a “popcorn” pattern of calcifications. The man is scheduled for surgery, and
the lesion is removed. The pathology report describes the mass as a disorganized
nodule of cartilage, with no cellular features of malignancy. What is the general
term for this lesion?
A. Choristoma
B. Hamartoma
C. Adenoma
D. Sarcoma

23. An 8-month-old male infant is admitted to the hospital because of a bacterial


respiratory infection. The infant has no previous history of viral or fungal infections.
The infant responds to appropriate antibiotic therapy, but is readmitted several weeks
later because of severe otitis media. Over the next several months, the infant is
admitted to the hospital multiple times for recurrent bacterial infections. Initial
workup at this time finds very low serum levels of IgG along with an absence of CD19+
cells. Further evaluation finds a lack of B cells past the pre-B stage. The disorder this
boy most likely has is associated with a defect involving which one of the following?
A. Adenosine deaminase
B. Bruton tyrosine kinase
C. Wiskott-Aldrich syndrome protein
D. IL-2 receptor
24. A neonate develops spastic contractions on the second postpartum day. Laboratory
studies show hypocalcemia. MRI studies demonstrate aplasia of the thymus and
parathyroid glands. What is the appropriate diagnosis?
A. Adenosine deaminase defi ciency
B. Common variable immunodefi ciency
C. DiGeorge syndrome
D. Transient hypogammaglobulinemia of infancy
E. Wiskott-Aldrich syndrome

25. A 19-year-old college sophomore is referred by his ophthalmologist because of the


finding of ectopia lentis (dislocation of the lens), which has resulted in visual
difficulties that have interfered with his performance on the varsity basketball team.
The patient is very tall, with long limbs and long, slender, spiderlike fingers. His chest
has a “caved-in” appearance, and he also has a modest degree of scoliosis. A
midsystolic “click” is heard, and an echocardiogram reveals mitral valve prolapse. The
most likely diagnosis is
A. Ehlers-Danlos syndrome
B. Fabry disease
C. Hurler syndrome
D. Marfan syndrome

26. A 2-year-old child has been followed for mental retardation and slow development,
as well as multiple birth defects. The child has a high-pitched catlike cry. On
examination, microcephaly, hypertelorism, micrognathia, epicanthal folds, low-set
ears, and hypotonia are noted. Karyotypic analysis would be expected to show
A. 5p-
B. 22q11-
C. 45,XO
D. 46,XY

27. A 34-year-old Jewish woman presents with weakness and a feeling of “fullness” in
the left side of her abdomen. A medical history finds that she is Jewish and her
family is from the eastern portion of Europe (Ashkenazi Jews). Physical examination
reveals a markedly enlarged spleen, but no neurologic abnormalities are found.
Examination of her retina is within normal limits. Laboratory examination reveals
decreased numbers of red blood cells, white blood cells, and platelets in the
peripheral blood. Elevated levels of angiotensin converting enzyme (ACE) and acid
phosphatase are found in the peripheral blood. Histologic sections from a bone
marrow biopsy reveal numerous macrophages that have a delicate fibrillar
(“crumpled tissue paper”) cytoplasm due to the accumulation of PAS-positive
material. What is the best diagnosis for this individual?
A. Sandhoff disease
B. Neiman-Pick disease
C. Gaucher disease
D. Tay-Sachs disease
28. A 1-year-old female infant is hospitalized for pneumonia. Bacterial cultures of the
sputum have grown Pseudomonas aeruginosa. She has had two prior
hospitalizations for severe respiratory infections. Her mother has noted that when
she kisses her child, the child tastes “salty.” The child has had weight loss that the
mother attributes to frequent vomiting and diarrhea with bulky,foul-smelling fatty
stools. The child is small for her age. Which of the following critical proteins is
altered in this condition?
A. Cystic fibrosis transmembrane conductance regulator
B. Dystrophin
C. a-1,4-Glucosidase
D. Lysyl hydroxylase

29. Which one of the listed disorders is a type of childhood malnutrition that is caused
by insufficient dietary protein and most commonly occurs when a child is weaned
from breast milk and placed on a diet that is high in starches and carbohydrates but
deficient in protein?
A. Beriberi
B. Kwashiorkor
C. Marasmus
D. Osteomalacia
E. Pellagra
Kwashiorkor is caused by protein deficiency but with adequate caloric intake.
30. Marasmus is not characterized by
A. swelling of limbs
B. replacing tissue proteins
C. impaired growth
D. protein deficiency

31. A 25-year-old woman begins breast feeding the infant. Which of the following cellular
processes that began in the breast during pregnancy allowed her to breastfeed the
infant for this period of time?
A. Epithelial dysplasia
B. Ductal epithelial metaplasia
C. Lobular hyperplasia
D. Stromal hypertrophy

32. A 32-year-old man is involved in a vehicular accident and sustains fractures of the
right femur and tibia and the left humerus. The fractures are stabilized surgically. He is
in stable condition for 2 days, but then suddenly becomes severely dyspneic. Which of
the following complications from his injuries is the most likely cause of his sudden
respiratory difficulty?
A. Fat embolism
B. Right hemothorax
C. Amnionic Embolism
D. Athero emboli
33. A 31-year-old man is on a scuba diving trip and descends to a depth of 50 m in the
Blue Hole off the coast of Belize. After 30 minutes, he has a malfunction in his
equipment and quickly returns to the boat on the surface. He develops difficulty
breathing within 5 minutes, with dyspnea and substernal chest pain, followed by a
severe headache and vertigo. An hour later, he develops severe, painful myalgias and
arthralgias. These symptoms abate within 24 hours. Which of the following occluding
his arterioles is the most likely cause of his findings?
A. Fat globules
B. Fibrin clots
C. Nitrogen gas bubbles
D. Platelet thrombi
E. Ruptured atheromatous plaque

34. An 8-month-old boy with a history of recurrent pneumonia is found to have almost
no circulating IgG. Cellular immunity is normal. His brother had this same disease and
died of echovirus encephalitis. His parents and sisters have normal serum levels of
IgG. What is the appropriate diagnosis?
A. DiGeorge syndrome
B. Isolated IgA defi ciency
C. Severe combined immunodefi ciency
D. Wiskott-Aldrich syndrome
E. X-linked agammaglobulinemia of Bruton

35. A 31-year-old gives birth to a healthy boy. Woman dies after a short illness. Clinical
presentation was sudden dyspnea, headache, fever, seizure. At autopsy, blood vessels
in the lungs contained fetal debris, as did other vessels of multiple organs. The
possible cause of death most likely is:
A. Gas emboli
B. Amniotic fluid emboli
C. Bone marrow emboli
D. Myocardium infarction

36. You're most likely to see caseous necrosis in


A. Myocardium
B. Tuberculous lung
C. Gangrenous diabetic foot
D. Brain

37. You're most likely to see liquefactive necrosis in


A. Myocardium
B. Tuberculous lung
C. Gangrenous diabetic foot
D. Brain

38. In Question Deficiency Glanzmann thrombasthenia → GpIIb-IIIa Complex


39. In Question GpIIb-IIIa Complex → Deficiency Glanzmann thrombasthenia
40. 16-year-old girl has had frequent nosebleeds since childhood. Her gums bleed
easily, even with routine tooth brushing. She has experienced menorrhagia since
menarche at age 13 years. On physical examination, there are no abnormal
findings. Laboratory studies show hemoglobin, 14.1 g/dL; hematocrit, 42.5%;
MCV, 90 μm3; platelet count, 277,400/mm3; and WBC count, 5920/mm3. Her
platelets fail to aggregate in response to ADP, collagen, epinephrine, and
thrombin. The ristocetin agglutination test result is normal. There is a deficiency
of glycoprotein IIb/IIIa. Prothrombin time is 12 seconds, and partial
thromboplastin time is 28 seconds. What is the most likely diagnosis?
A. Disseminated intravascular coagulation
B. Glanzmann thrombasthenia
C. Immune thrombocytopenic purpura
D. Vitamin C deficiency
E. Von Willebrand disease

41. patient presents with a large wound to his right forearm that is the result of a
chain saw accident. You treat his wound appropriately and follow him in your
surgery clinic at routine intervals. Initially his wound is filled with granulation
tissue, which is composed of proliferating fibroblasts and proliferating new
blood vessels (angiogenesis). Which of the following substances is thought to
be the most important growth factor involved in angiogenesis?
A. Epidermal growth factor (EGF)
B. Platelet-derived growth factor (PDGF)
C. Transforming growth factor-alpha (TGF-α)
D. Vascular endothelial growth factor (VEGF)

42. An 8-month-old male infant is admitted to the hospital because of a bacterial


respiratory infection. The infant responds to appropriate antibiotic therapy, but
is readmitted several weeks later because of severe otitis media. Over the next
several months, the infant is admitted to the hospital multiple times for
recurrent bacterial infections. Workup finds a lack of B cells past the pre-B stage.
The infant has no previous history of viral or fungal infections. Which of the
following is the most likely diagnosis?
A. Common Variable Immunodeficiency
B. DiGeorge’s syndrome
C. Isolated IgA deficiency
D. Agammaglobulinemia of Bruton

43. Which one of the listed changes correctly describes the pathophysiology involved in
the production of pulmonary edema in patients with congestive heart failure? (Pretest)
A. Decreased plasma oncotic pressure
B. Endothelial damage
C. Increased hydrostatic pressure
D. Increased vascular permeability
E. Lymphatic obstruction

44. A 45-year-old woman who works while standing for long periods notices at the
end of her 8-hour shift that her lower legs and feet are swollen, although there
was no swelling at the beginning of the day. There is no pain or erythema
associated with this swelling. She is otherwise healthy and takes no medications;
laboratory testing reveals normal liver and renal function. Which of the following
mechanisms best explains this phenomenon?
A. Excessive free water intake
B. Hypoalbuminemia
C. Increased hydrostatic pressure
D. Lymphatic obstruction
E. Secondary aldosteronism

45. What type of leukocyte actively participates in acute inflammatory processes and
contains myeloperoxidase within its primary (azurophilic) granules and alkaline
phosphatase in its secondary (specific) granules?
A. Neutrophils
B. Eosinophils
C. Monocytes
D. Lymphocytes

46. A 94-year-old woman has dyspnea and an increasing cough with frothy sputum
production for the past month. She is afebrile. A chest radiograph shows the
findings in the figure. Which of the following is the most likely mechanism for
development of her pulmonary infiltrates?
A. Decreased sodium intake
B. Hypoalbuminemia
C. Increased hydrostatic pressure
D. Inflammation
E. Pulmonary venous obstruction

47. A 71-year-old male with a history of poorly controlled hypertension due to


noncompliance with medications is brought to the emergency room by his family
because of increasing shortness of breath. An X-ray of the chest reveals bilateral
pleural effusions and enlargement of the heart. Given these features, of the
following, which is the most likely causative mechanism for the condition
producing his shortness of breath?
A. Increased hydrostatic pressure
B. Increased vascular permeability
C. Decreased colloid osmotic pressure
D. Lymphatic obstruction
E. Sodium retention

48. A 37-year-old woman has noticed a lump in her left breast over the past 2
months. On physical examination, the skin overlying the left breast is thickened,
reddish orange, and pitted. Mammography shows a 3-cm underlying density. A
fine-needle aspirate of the density is performed and on microscopic examination
shows carcinoma. Which of the following mechanisms best explains the gross
appearance of the skin of her left breast?
A. Chronic inflammation
B. Chronic passive congestion
C. Ischemic necrosis
D. Lymphatic obstruction
E. Venous thrombosis
49. A 35-year-old male living in a southern region of Africa presents with increasing
abdominal pain and jaundice. He has worked as a farmer for many years, and
sometimes his grain has become moldy. Physical examination reveals a large mass
involving the right side of his liver, and a biopsy specimen from this mass confirms the
diagnosis of liver cancer (hepatocellular carcinoma). The pathogenesis of this tumor
involves which of the following substances?
A. Aflatoxin B1
B. Direct-acting alkylating agents
C. Vinyl chloride
D. Azo dyes
E. β-naphthylamine

50. A 20-year-old woman presents with a skin rash. The rash is localized to the
extensor surfaces of her elbows and knees, and is composed of multiple well-
demarcated circular-to-oval–shaped plaques and papules covered by a silvery
scale overlying reddened erythematous skin. The silvery scale can be scraped
off, revealing inflamed skin underneath. She states that several family
members have a similar rash. Which of the following is the most likely
diagnosis?
A. Neurodermatitis (lichen simplex chronicus)
B. Psoriasis
C. Pemphigus vulgaris
D. Bullous pemphigoid
E. Dermatitis herpetiformis

51. 34-year-old male presents with multiple large, sharply defined, silver-white
scaly plaques on the extensor surfaces of his elbows and knees and on his scalp.
Physical examination reveals discoloration and pitting of his fingernails. Lifting
of one of the scales on his elbows produces multiple minute areas of bleeding
(positive Auspitz sign). Histologic sections from one of the scaly plaques would
most likely reveal
A. Subepithelial bullae
B. Regular elongation of the rete ridges
C. Liquefactive degeneration of the basal layer of the epidermis
D. Increased granular cell layer
E. Chronic inflammation below a zone of degenerated collagen

52. A 50-year-old man presents for a routine physical examination, which demonstrates
an enlarged liver. During the visit, he describes memos from his supervisor at work
regarding chronic exposure to vinyl chloride. The patient has an elevated risk for
which of the following tumors?
A. Angiosarcoma of the liver
B. Carcinoid tumor
C. Hepatic adenoma
D. Lymphoma
E. Metastatic colon cancer
53. A 47-year-old male presents with pain in the midportion of his chest. The pain is
associated with eating and swallowing food. Endoscopic examination reveals an
ulcerated area in the lower portion of his esophagus. Histologic sections of tissue
taken from this area reveal an ulceration of the esophageal mucosa that is filled
with blood, fibrin, proliferating blood vessels, and proliferating fibroblasts.
Mitoses are easily found, and most of the cells have prominent nucleoli. Which
one of the following correctly describes this ulcerated area?
A. Caseating granulomatous inflammation
B. Dysplastic epithelium
C. Granulation tissue
D. Squamous cell carcinoma
E. Noncaseating granulomatous inflammation

54. A starving, 4-year-old, African boy presents with apathy, generalized edema, and
an enlarged fatty liver. The physician notes that, despite generalized growth failure,
subcutaneous fat is preserved. What is the appropriate diagnosis?
A. Beri-beri
B. Kwashiorkor
C. Marasmus
D. Pellagra

55. A 45-year-old woman presents with abdominal pain and vaginal bleeding. A
hysterectomy is performed and shows a benign tumor of the uterus derived from a
smooth muscle cell. What is the appropriate diagnosis?
A. Angiomyolipoma
B. Leiomyoma
C. Myxoma
D. Rhabdomyoma

56. Reversible injury encompasses:


A. Metaplasia, hypertrophy, atrophy, and fibrosis
B. Hyperplasia, hypertrophy, and necrosis
C. Hyperplasia, hypertrophy, apoptosis
D. Hyperplasia, metaplasia, hypertrophy, atrophy, and necrosis
E. Hyperplasia, hypertrophy, atrophy, and metaplasia

57. A 42-year-old man has blunt force trauma. Two weeks later, at the site of injury
produced a yellow-brown color. Whish of the following substances has most likely
accumulated?
A. Lipofuscin
B. Bilirubin
C. Melanin
D. Hemosiderin
E. Glycogen

58. Edema occurs due to_____________?


A. Increased capilary permeability
B. Decreased capilary permeability
C. Decreased interstitial fluid
D. Decreased blood flow
59. A 60-year-old women has severe chest pain for the past 7 hours. On physical
examination she is afebrile, but has tachycardia. Lab studies show elevated serum
troponin I. A coronary angiogram reveals > 70% occlusion of the left anterior
descending artery. If there is an irreversible injury to myocardial fibers, which of the
following cellular changes occurs? (45) (Ch.2)
A. Glycogen stores are depleted
B. Cytoplasmic sodium increases
C. Nuclei undergo karyorrhexis
D. Intracellular pH diminishes
E. Blebs form on cell membranes

60. A 47-year-old man with a history of heavy smoking complains of chronic cough. A
“coin lesion” is discovered in his right upper lobe on chest X-ray. Bronchoscopy and
biopsy fail to identify a mass, but the bronchial mucosa displays squamous
metaplasia. What is the most likely outcome of this morphologic adaptation if the
patient stops smoking?
A. Atrophy
B. Malignant transformation
C. Necrosis and scarring
D. Persistence throughout life
E. Reversion to normal

61. An 8-year-old girl with asthma presents with respiratory distress. She has a
history of allergies and upper respiratory tract infections. She also has history of
wheezes associated with exercise. Which of the following mediators of
inflammation is the most powerful stimulator of bronchoconstriction and
vasoconstriction in this patient?
A. Bradykinin
B. Complement proteins
C. Interleukin-1
D. Leukotrienes
E. Tumor necrosis factor-α

62. A 56-year-old woman dies of a chronic neurodegenerative disease. Autopsy reveals


spongiform encephalopathy with brain amyloidosis. This patient’s amyloidosis most
likely belongs to which of the following categories?
A. Ab
B. AA
C. AE
D. APrP = AD

63. A 42-year-old woman gives birth to a neonate with multiple congenital abnormalities.
Physical fi ndings included a fl at facial profi le, slanted eyes, epicanthal folds, Brushfi
eld spots, short nose, short neck, dysplastic ears, clinodactyly, a large protruding
tongue, and a pronounced heart murmur. What is the most common cause of this
developmental birth disease?
A. Chromosomal deletion
B. Expansion of trinucleotide repeat
C. Frameshift point mutation
D. Nondisjunction
64. Which of the following statements are FALSE:
A. Necrosis is programmed cell death causes cell shrinkage
B. A myocardial infarct is considered apoptosis
C. Necrosis is energy dependent
D. All of the above

65. A 51-year-old women comes to the physician with substernal burning pain following
meals for many years. The physician decides to perform upper Gl endoscopy with
biopsies. Upper GI endoscopy shows erythematous area of the lower esophageal
mucosa above the gastroesophageal junction. There is no mass lesion, no ulceration,
and no hemorrhage noted. The diagnosis of gastroesophageal reflux disease (GERD) is
made. The biopsies show the presence of columnar epithelium with goblet cells.
Which of the following mucosal alterations is most likely represented by these
findings?
A. Ischemia
B. Hyperplasia
C. Carcinoma
D. Dysplasia
E. Metaplasia

66. An 87-year-old woman dies peacefully in her sleep. At autopsy, a rest of pancreatic
tissue is identifi ed in the wall of the lower esophagus. This fi nding represents an
example of which of the following congenital tumor-like conditions?
A. Choristoma
B. Hamartoma
C. Hemangioma
D. Teratoma

67. A 20-year-old woman has an ovarian tumor removed. The surgical specimen is 10 cm
in diameter and cystic. The cystic cavity is found to contain black hair and sebaceous
material. Histologic examination of the cyst wall reveals a variety of benign
differentiated tissues, including skin, cartilage, brain, and mucinous glandular
epithelium. What is the diagnosis?
A. Adenoma
B. Chondroma
C. Hamartoma
D. Teratoma

68. A 30-year-old woman presents to the emergency room complaining that she has
the “worst headache” of her life. Her temperature is 37°C (98.6°F), blood pressure
135/85 mm Hg, and pulse 90 per minute. The patient shows no evidence of muscle
weakness or ataxia. Imaging studies reveal subarachnoid hemorrhage, and an
angiogram shows a saccular aneurysm. Which of the following is the most likely
diagnosis?
A. Epidural hematoma
B. Concusion
C. Subarachnoid hemorrhage (SAH)
D. Subdural hematoma
69. A 16-year-old girl complains that she has not started menstruating like other girls
her age. The patient is short (4 ft, 11 in) and has a thick-webbed neck. Physical
examination reveals widely spaced nipples and poor breast development. If this
patient’s genetic disease was caused by nondisjunction during mitosis of a somatic
cell in the early stages of embryogenesis, which of the following is the patient’s
most likely karyotype?
A. 45,X
B. 45,X/46,XX
C. 47,XX,+21
D. 47,XXY

70. All of the following are ultrastructural changes of reversible cell injury except:
A. Dilation of the endoplasmic reticulum
B. Cell membrane blebbing
C. Karyorrhexis
D. Detachment of polysomes

71. A 25-year-old woman sustains a deep, open laceration over her right forearm in a
motorcycle accident. The wound is cleaned and sutured. Which of the following cell
types mediates contraction of the wound to facilitate healing?
A. Endothelial cells
B. Fibroblasts
C. Macrophages
D. Myofi broblasts
E. Smooth muscle cells

72. A patient came to your office with a chief complaint of erythema and intense
pruritus on both hands after wearing Latex gloves. What type of Hypersensitivity
reaction is this patient having?
A. Anaphylactic
B. Cytotoxic
C. Immune Complex
D. Delayed
E. None of the Above

73. During a routine checkup, a 50-year-old man is found to have blood in his urine.
He is otherwise in excellent health. An abdominal CT scan reveals a 2-cm right
renal mass. You inform the patient that staging of this tumor is key to selecting
treatment and evaluating prognosis. Which of the following is the most important
staging factor for this patient?
A. Histologic grade of the tumor
B. Metastases to regional lymph nodes
C. Proliferative capacity of the tumor cells
D. Somatic mutations in the p53 tumor suppressor gene

74. Which of the following characteristic of apoptosis?


A. Absence of inflammation
B. Cell swelling
C. Leaking of enzymes
D. Affects numerous cells at a time
75. A 32-year-old man is involved in a vehicular accident and sustains fractures of
the right femur and tibia and the left humerus. The fractures are stabilized
surgically. He is in stable condition for 2 days, but then suddenly becomes
severely dyspneic. Which of the following complications from his injuries is the
most likely cause of his sudden respiratory difficulty?
A. Amnionic embolism
B. Fat embolism
C. Athero emboli
D. Right hemothorax

76. Protein accumulation in cell (!!!)


A. Russell bodies
B. Steatosis
C. Psammoma bodies
D. Brown atrophy

77. Metastatic calcification:


A. Is encountered in areas of necrosis.
B. Commonly develops in aging or damaged heart valves.
C. Can occur with normal serum levels of calcium.
D. Can be associated with metastatic malignancy.
E. Is commonly associated with hypoparathyroidism.

78. A 40-year-old male with Down syndrome is brought to your clinic by his mother.
She reports that over the past few months he has started having difficulty
managing his daily routine at his assisted-living facility and no longer seems like
himself. She says that last week he wandered away from the facility and was
brought back by police. Additionally, he has stopped taking his regular
antiepileptic medication, and she is concerned that he might have a seizure. TSH
is checked and is normal. Which of the following is most likely to be responsible
for this man's current presentation?
A. Expansion of trinucleotide repeats
B. Abnormal protein accumulation
C. Hormone deficiency
D. Premature degradation of a protein

79. A 30-year-old man with a marfanoid habitus presents for genetic counseling. His
father, paternal uncle, and paternal great-grandfather died of sudden cardiac
deaths. His father, specifically, suffered from an aortic dissection. As part of this
patient's work-up, he recently had cardiac imaging, which reveals a 5 cm aortic
aneurysm. He is sent for further surgical consultation. Which of the following is
the most likely diagnosis?
A. Marfans syndrome
B. Ehlers-Danlos syndrome
C. Hurler syndrome
D. Osteogenesis imperfecta
80. A 50-year old man has blood pressure of 150/90 mm Hg. If this person remains
untreated foryears, which of the following cellular alterations will be seen in the
myocardium?
A. atrophy B
B. metaplasia
C. hypertrophy
D. dysplasia

81. A 4-year-old male is evaluated for frequent epistaxis and mucous membrane
bleeding. Physical examination shows diffuse petechiae on the patient’s distal
extremities. Peripheral blood smear shows an absence of platelet clumping. An
ELISA binding assay reveals that platelet surfaces are deficient in GIIb/IIIa
receptors. Serum platelet count is normal. Which of the following is the most
likely diagnosis?
A. Hemophilia A
B. Thrombotic thrombocytopenic purpura
C. Bernard-Soulier disease
D. Glanzmann’s thrombasthenia

82. The hallmark of chronic inflammation is


A. Increased permeability of microcirculation
B. Migration of macrophages at site of injury
C. Tissue destruction
D. Mononuclear cell infiltration

83. A Caucasian newborn with facial dysmorphia and cleft palate is found to have a
deletion involving the long arm of chromosome 22. These findings are most
consistent with:
A. Kartagener's syndrome
B. DiGeorge syndrome
C. Marfan syndrome
D. Down syndrome

84. A 20-year-old woman presents with malar rash, arthralgias, low-grade fever, and
high titer antibodies to double-stranded DNA and to the Sm (Smith) antigen.
Which of the following forms of hypersensitivity is the primary mechanism of the
abnormalities found in this disorder?
A. Type I (immediate or anaphylactic) hypersensitivity
B. Type II (antibody-mediated or cytotoxic) hypersensitivity
C. Type III (immune complex-mediated disorders) hypersensitivity
D. Type IV (cell-mediated) hypersensitivity

85. The uterine endometrial lining thickens during the proliferative phase of the
menstrual cycle. This is due to
A. Hypertrophy
B. Hyperplasia
C. Dysplasia
D. Metaplasia
86. The type of necrosis seen in injury associated with acute pancreatitis is:
A. Thraumatic necrosis
B. Enzymatic fat necrosis
C. Cell-mediated necrosis
D. Caseation necrosis

87. An arterial thromboemboli is most likely to produce a hemorrhagic infarct in


whish organ?
A. Kidney
B. Liver
C. Lungs
D. Bone

88. Which of the following statements in NOT true of red infarcts?


A. It occurs in organs with double blood supply
B. Commonly found in the heart and kidneys
C. Seen in venous occlusion with infarction
D. Seen in organs with well developed anastomosis

89. A 26-year old woman presents with increasing fatigue, arthritis, shortness of
breath, and a bimalar, photosensitive erythematous rash. Biopsies from this rash
reveal liquefactive degeneration of the basal layer of the epidermis with a
perivascular lymphoid infiltrate. Physical examination shows bilateral pleural
effusions, the fluid from which when examined histologically reveals multiple
erythrocytes phogocytized by phagocytic leukocytes. Which of the following is
the most likely diagnosis?
A. Dermatomyositis
B. systemic lupus erythematosus
C. Sjogren’s syndrome
D. rheumatoid arthritis

90. Which of the following is best stained by Congo red stain?


A. Glycogen
B. Fat
C. Mucin
D. Amyloid

91. A 30-year-old male presents to the dermatologist with silvery, scaling plaques
on his elbows and knees. His mother has been afflicted with the same condition
in the past. The most likely diagnosis is
A. Acne vulgaris
B. Pemphigus vulgaris
C. Pityriasis rosea
D. Psoriasis

92. Which of the following are characteristics of antemortem clots:


A. lines of Zahn
B. currant jelly clots
C. both
D. neither
93. A 3-year-old boy brought to a pediatrician to establish care after his parents
moved across the country. On presentation, his parents sat that He is hard to
control at home and can behave aggressively with other children. Physical exam
reveals obesity and small testicles. He does not follow the physician's directions
during developmental testing; however, he appears to be globally delayed in
motor function. His parents also recall that he had some problems with feeding
shortly after birth but the do not recall the cause of the issues. This patient's
disease is most closely associated with which of following principles? (!!!)
A. Trinucleotide repeat
B. Imprinting
C. Mosaicism
D. Translocation

94. A 6-month-old boy is brought to his pediatrician for pneumonia. He was treated
2 weeks ago for severe otitis media. On examination, his tonsils and adenoids
are absent. Heart sounds are normal, and there are crackles in the left lower lung
field. Serum levels of IgG, IgA, and IgM are low, and B-lymphocytes are
undetectable. What is the underlying defect responsible for this patient’s
recurrent infections?
A. Impaired B-cell maturation
B. Impaired cytokine receptor signaling
C. Failure of development of the third and fourth pharyngeal pouches
D. Disruption of the CD40-CD40 ligand interaction

95. A woman who is allergic to cats visits a friend who keeps several pet cats. During
the visit, she inhales cat dander and within minutes, she develops nasal
congestion and abundant nasal secretions. Which of the following substances is
most likely to produce these findings?
A. Tumor necrosis factor
B. Bradykinin
C. Histamine
D. Complement C5a

96. Type of inflammation induced by indigestible foreign bodies.


A. Serous
B. Granulomatous
C. Suppurative
D. Fibrinous

97. A 3-year old child touches a lit candle. Within several hours, there is marked
erythema of the skin of the fingers on the child’s left hand, and small blisters
appear on the finger pads. Which of the following terms best describe the
process?
A. fibrinous inflammation
B. ulceration
C. abscess formation
D. serous inflammation
98. Which of the following is a feature of necrosis?
A. Reduced cell size
B. Intact cellular contents
C. Intact plasma membrane
D. Karyolysis

99.
100.
101.
102.

Failure of development of the third and fourth pharungeal pouches


Disruption of the CD40-CD40 lingand interaction
Sjogren syndrome SSA : no saliva no tears— lymphocyte

Adenosine deaninase = severe combined immunodeficiency


“ IL 2 4 7 9 15 21 “

Aplastic of parathyroid +thyroid glands = DiGeorge syndrome

Tingling + pain in tips of fingered = mixed connective tissue diseasss

Psoriasis = knees - elbow -scalp - skin lesions - auspitz sign

Child taste salty + foul smelling fast th stools = cystic fibrosis

Spider like fingers + long limbs + ectopic lenses = Marian syndrome

Catlike cry = cri de chat , chromosome 5p , hTERT

Turner syndrome = 45xo - short stature , webbed neck , broad chest

Gaucher disease = chilotriosidase + angiotensin converting enzyme =


glucocerebrosidase
Lack of B cells past the pre_b stage = agammaglobulinemia of Bruton

Hepatocellular carcinoma = aflatoxin B1


Heptao angiosarcoma = vinyl chloride

Albinism / white skin / silvery hair skin = chediack higashi disease

Proliferating blood vessels + proliferating fibroblasts = Granulation tissue

Ehlers danlos syndrome = autosomal dominant

Pt +ptt- factor viii = X chromosome

Ube3a = Angelman syndrome = inappropriate laughter chromosome 15

Cystic fibrosis = chromosome 7 - CFTR gene

Wiskott Aldrich syndrome = x linked , eczema , thrombocytopenia,


polysaccharide …inborn

Wernick Korsakof syndrome = ataxia , nystagmus , thiamine deficiency

Systemic sclerosis = chalky white substance , heart burn , claw like appearance
= centromeric proteins

Egfr + pten mutations = glioblastoma multiforme “GBM”

Protein deficiency = kwashiorkor


Calorie deficiency = marasmus

Proapoptic = BAX ,BAK


Anti-apoptotic = BCL-2 , BCL-xl
FINAL PATHOLOGY
1. An 86-year-old man with a history of recurrent urinary tract presents with fever,
tachypnea, tachycardia, mental obtundation, and reduced blood pressure. Which
of the following forms of shock is most likely?
A. Anaphylactic shock
B. Cardiogenic shock
C. Hypovolemic shock
D. Septic shock

2. A 55-year-old develops an acute myocardial infarction because of the sudden


occlusion of the left anterior descending coronary artery. The areas of myocardial
necrosis within the ventricle can best be described as (Ch.2)
A. Coagulative Necrosis
B. Liquefactive Necrosis
C. Caseous Necrosis
D. Fibrinoid Necrosis
E. Fat Necrosis

3. A 25 -year-old man develops marked right lower quadrant abdominal pain over the
past day. On physical examination there is rebound tenderness on palpation over
the right lower quadrant. Laparoscopic surgery is performed, and the appendix is
swollen, erythematous, and partly covered by a yellowish exudate. It is removed,
and a microscopic section shows infiltration with numerous neutrophils. The pain
experienced by this patient is predominantly the result of which of the following
two chemical mediators?
A. Complement C3b and IgG
B. Interleukin-1 and tumor necrosis factor
C. Histamine and serotonin
D. Prostaglandin and bradykinin

4. Which of the following cellular adaptation to stress is described


below? Shrinkage in the size of cells by the loss of cell substance
producing autophagic vacuoles (Ch.2)
A. Hypertrophy
B. Atrophy
C. Hyperplasia
D. Metaplasia
E. Dysplasia

5. The term hemangioma refers to (Ch.6)


A. An irregular accumulation of blood vessels
B. Proliferation of non-neoplastic fibrous connective tissue
C. Maturation and spatial arrangement of cells
D. Normal tissue misplaced within another organ
E. Metastatic involvement of surrounding tissue
6. What are bradykinin, serotonin and prostaglandins?
A. Pain inducing chemical mediators
B. Mediators of apoptosis
C. Necrosis chemokines
D. Neurotransmitters

7. A 26-year-old female presents with severe pain during menses (dysmenorrhea).


To treat her symptoms, you advise her to take indomethacin (COX Inhibitor) in the
hopes that it will reduce her pain by interfering with the production of
A. Bradykinin
B. Histamine
C. Leukotrienes
D. Phospholipase A2
E. Prostaglandin F2

8. A 24-year-old woman dies few hours after the labor. Review of the
clinical history reveals that she had become acutely ill with dyspnea,
hypotension, and seizures, and a chest radiograph had demonstrated
evidence of pulmonary edema. Autopsy findings were fetal skin. The
possible cause of death most likely was: (8) (Ch.4)
A. Bone marrow emboli
B. Pulmonary emboli
C. Widespread thrombosis
D. Septic emboli

9. After binding to Fas ligsnd (CD95L), Fas (CD95) self-associates and


activates Fas-associated death domain protein (FADD), which in turn
induces apoptosis by stimulating
A. bcl-2
B. Caspase 8
C. Cytochrome a3
D. Cytochrome p450
E. Elastase 6

10. Clawlike flexion deformity of the finger is associated with which disorder? (Ch.5)
A. Scleroderma
B. Sjogren syndrome
C. SLE
D. Rheumatoid arthritis
E. Parkinson's disease

11. Which of these cytokines is mainly responsible for cancer cachexia? (Ch.6)
A. TGF beta
B. IL-17
C. TNF
D. IL-10
12. A 31-year-old gives birth to a healthy boy. Woman dies after a short
illness. Clinical presentation was sudden dyspnea, headache, fever,
seizure. At autopsy, blood vessels in the lungs contained fetal debris,
as did other vessels of multiple organs. The possible cause of death
most likely is: (12)
A. Gas emboli
B. Amniotic fluid emboli
C. Bone marrow emboli
D. Myocardium infarction

13. A 12-year-old boy suddenly develops severe testicular pain. He is taken to the
emergency room, where he is evaluated and immediately taken to surgery. There
his left testis is found to be markedly hemorrhagic due to testicular torsion. This
abnormality caused a hemorrhagic testicular infarction because of (Ch.4)
A. Arterial occlusion
B. Septic implantation
C. The collateral blood supply of the testis
D. The dual blood supply of the testis
E. Venous occlusion

14. A 49-year-old woman who has been diagnosed with early-stage


breast cancer undergoes surgery for a lumpectomy to remove a small
tumor detected by mammography. The pathology report confirms the
early stage of the cancer and further comments on the fact that there
is significant desmoplasia in the surrounding tissue. The term
desmoplasia refers to (Ch.6)
A. an irregular accumulation of blood vessels.
B. maturation and spatial arrangement of cells.
C. metastatic involvement of surrounding tissue.
D. normal tissue misplaced within another organ.
E. proliferation of non-neoplastic fibrous connective tissue.

15. A 51-year-old women comes to the physician with substernal burning pain
following meals for many years. The physician decides to perform upper Gl
endoscopy with biopsies. Upper GI endoscopy shows erythematous area of the
lower esophageal mucosa above the gastroesophageal junction. There is no
mass lesion, no ulceration, and no hemorrhage noted. The diagnosis of
gastroesophageal reflux disease (GERD) is made. The biopsies show the
presence of columnar epithelium with goblet cells. Which of the following
mucosal alterations is most likely represented by these findings?
A. Ischemia
B. Hyperplasia
C. Carcinoma
D. Dysplasia
E. Metaplasia
16. You are studying apoptosis. By the introduction of a naturally
chemical, you wish to shorten the life span of cultured cells that are
derived from a human liver and that have been exposed to radiation.
Which of the following effects, if caused by the introduced chemical,
would produce your desired outcome? (Ch.2)
A. Increased concentration of bcl-2
B. Increased concentration of bcl-xL
C. Increased concentration of BAX
D. Decreased concentration of Bad

17. The cardinal sign of inflammation called rubor is mainly the result of (Ch.3)
A. Increased vascular permeability of venules
B. Vasodilation of arterioles
C. Vasoconstriction of muscular arteries
D. Decreased vascular permeability of capillaries
E. Decreased interstitial hydrostatic pressure

18. Why do neutrophils only stay in inflammatory sites for 2-3 days max? (Ch.3)
A. They are replaced by macrophages
B. They undergo necrosis
C. They are damaged my inflammation
D. They undergo apoptosis

19. While hunting, a 45-year-old Caucasian male is stung by a bee. He


begins to feel unwell and is taken to the hospital. Upon arrival, it is
noted that the patient is hypotensive with a blood pressure of 60/40
mmHg. Furthermore, the patient is tachycardic and edematous. What
is responsible for this patient's presentation? (Ch.5)
A. Free IgE binding to the antigen
B. IgE on mast cell surfaces binding to the antigen
C. IgE activation of complement
D. Complexes of IgE binding to the antigen
E. IgE on eosinophils binding to the antigen

20. Which of these is not an oncogenic DNA virus? (Ch.6)


A. Polyoma virus
B. Hepatitis C virus
C. Kaposi sarcoma
D. EBV
E. HPV

21. A 39-year-old female presents with intermittent pelvic pain. Physical examination
reveals a 4-cm mass in the area of her right ovary. Histologic sections from this
ovarian mass reveal a papillary tumor with multiple, scattered small, round,
laminated calcifications. These structures are most likely the result of
A. Apoptosis
B. Dystrophic calcification
C. Enzymatic necrosis
D. Hyperparathyroidism
E. Metastatic calcification
22. A 38-year-old woman was hospitalized, an appendectomy was performed.
Three days after a surgery she had sudden onset of dyspnea, pain in the
chest, cough productive of frothy, blood-tinged sputum. Ventilation-
perfusion scintigraphy indicates a perfusion defect. Which of the following
pattern would most likely have been found? (Ch.3)
A. Hemorrhagic (red) infarct
B. Anemic (white or pale) infarct
C. Generalized thrombosis
D. Disseminated intravascular coagulation (DIC)

23. A 35-year-old woman had laparoscopic surgery 4 months ago. Now she has a
small 1 cm nodule beneath the skin at the incision site that was sutured. Which
of the following cell types is most likely to be most characteristic of the
inflammatory response in this situation? (Ch.3)
A. Neutrophil
B. Eosinophil
C. Plasma cell
D. Mast cell
E. Giant cell

24. A 52-year-old woman with sudden onset of severe abdominal pain comes to the
emergency department. On physical examination she has diffuse tenderness in all
abdominal quadrants. Laboratory study shows an elevated level of lipase (525 U/L).
The CT scan of the abdomen shows peritoneal fluid collections and decreased
attenuation along with enlargement of the pancreas (pancreatitis). Whish of the
following cellular changes is most likely to accompany these findings? (Ch.2)
A. Apoptosis
B. Liquefactive necrosis
C. Coagulative necrosis
D. Fat necrosis
E. Dry gangrene

25. Suppurative of purulent inflammation is characterized by? (Ch.3)


A. Edema
B. Inflammation of body cavities
C. Extravascular fluid
D. Large amount of pus

26. A 55 -year-old woman has had a cough with fever for 4 days. A chest radiograph
reveals infiltrates in the right lower lobe. A sputum culture grows Streptococcus
pneumoniae. The clearance of these organisms from the lung parenchyma would be
most effectively accomplished through generation of which of the following
substances by the major inflammatory cell type responding to this infection?
A. Hydrogen peroxide
B. Prostaglandin E2
C. Kallikrein
D. Platelet activating factor
E. Leukotriene B4
27. A 27-year-old woman presents to clinic for her annual physical. She has a
condition marked by joint pain, renal insufficiency, and an intermittent malar
rash on face. She reports that on a recent visit to her obstetrician following a
miscarriage, she tested positive for lupus anticoagulant. Which of the following
is true regarding this patient's condition?
A. Prolonged PT and a tendency toward bleeding
B. Prolonged PT and a tendency toward clotting
C. Prolonged PTT and a tendency toward bleeding
D. Prolonged PTT and a tendency toward clotting
E. Prolonged PT and PTT a tendency toward bleeding

28. A 40-year-old Caucasian female presents to your office with fever,


fatigue, cold fingers, and the malar rash. The presence of which of the
following antibodies has the highest specificity for diagnosis of this
patient's condition?
A. Ant-La
B. Anti-dsDNA
C. Anti-Ro
D. Anti-acetylcholine receptor
E. Anti-ssDNA

29. Which of these syndromes is not characterised by defective DNA


repair by homologous recombination?
A. Bloom syndrome
B. Xeroderma pigmentosum
C. Fanconi anemia
D. Ataxia telangiectasia

30. Which of these DNA repair mechanisms is defective in Hereditary


Nonpolyposis Colon Cancer Syndrome?
A. Mismatch repair
B. Base excision repair
C. Homologous recombination
D. Nucleotide excision repair

31. Which of these enzymes is not present in macrophages? (31)(P.68)


A. Kinase
B. Collagenase
C. Acid hydrolases
D. Elastase
32. A 27-year-old woman presents to her primary care physician for evaluation
of malaise, joint pains, and rash. Over the last 1-2 months, she has felt
generally unwell with fatigue and low-grade fever. More recently, she has
started to experience joint pains in her hands and noticed a rash over her
face after sun exposure. She is otherwise healthy and takes no medications.
Examination reveals an erythematous rash with a small amount of
underlying edema. Which of the following is the most common cardiac
manifestation of this patient's underlying condition? (Ch.3)
A. Valvular disease
B. Pericarditis
C. Coronary artery disease
D. Heart failure
E. Conduction arrhythmias

33. A 55-year-old tight-handed man with a long history of rheumatic heart disease
with mitral stenosis and atrial fibrillation is brought to emergency department
after collapsing to the floor at home. He is unable to speak or walk and has
right hemiplegia with a right extensor planter response. These finding most
likely result from: (Ch.4)(BRS)
A. Generalized thrombosis
B. Amniotic fluid
C. Middle cerebral arterial embolism
D. Hepatic vein thrombosis

34. A 5-year-old African American female has experienced recurrent respiratory


infections. To determine how well her cell-mediated immunity is performing, a
Candida skin injection is administered. After 48 hours, there is no evidence of
induration at the injection site. Of the following cell types, which one would
have mediated the reaction? (immune)
A. Plasma cells
B. Basophils
C. T-cells
D. Mast cells
E. Fibroblasts

35. A 82-year-old woman had sudden the loss of consciousness that persisted for over
an hour. When she gains consciousness, she had aphasia and right hemiparesis. A
cerebral angiogram revealed an occlusion to her left middle cerebral artery. After 3
Months, a computed tomographic (CT) scan shows a large 6 cm cystic area in her
left parietal lobe cortex. This CT finding is most likely the consequence of resolution
from which of the following cellular events? (Ch.2)
A. Coagulative necrosis
B. Caseous necrosis
C. Apoptosis
D. Liquefactive necrosis
E. Atrophy
36. A 40-year-old woman is bridged from heparin to warfarin for recent
provoked DVT. Soon after receiving warfarin, she develops dark lesions on
her arms. They are large, dark, and necrotic. Warfarin is stopped and vitamin
K and heparin are administered. Suspicious, her physician orders a specific
assay that measures the activity of protein C. What is the normal function of
this protein? (36) (Ch.4) (P.106)
A. Inhibits Thrombin
B. Prevents primary Hemostasis
C. Inhibits Factor V and VIII
D. Activates Platelets
E. Activates Coagulation Factor X

37. Which of these drugs is considered a direct carcinogenic agent?


A. Methotrexate
B. Aflatoxin
C. Cyclophosphamide
D. Nitrosamine

38. Sjogren syndrome is most commonly secondary.


A. True
B. False

39. A 42-year-old man has blunt force trauma. Two weeks later, at the
site of injury produced a yellow-brown color. Whish of the following
substances has most likely accumulated?
A. Lipofuscin
B. Bilirubin
C. Melanin
D. Hemosiderin
E. Glycogen

40. A 4-year-old boy presents with recurrent bacterial and fungal infections primarily
involving his skin and respiratory tract. Physical examination reveals the presence
of oculocutaneous albinism. Examination of a peripheral blood smear reveals
large granules within neutrophils, lymphocytes, and monocytes. The total
neutrophil count is found to be decreased. Further workup reveals ineffective
bactericidal capabilities of neutrophils due to defective fusion of phagosomes with
lysosomes. What is the correct diagnosis?
A. Ataxia-telangiectasia
B. Chédiak-Higashi syndrome
C. Chronic granulomatous disease
D. Ehlers-Danlos syndrome
E. Sturge-Weber syndrome
41. Therapeutic irradiation of the head and neck can give rise to papillary
thyroid cancer years later.
A. True
B. False

42. Adhesion of neutrophils on endothelial surface is mediated by?


A. Prostaglandins
B. Selectins and itegrins
C. Neurotransmitters
D. Chemokines

43. Which of the following proteins functions as a death receptor for the
extrinsic apoptotic pathway?
A. BCL2
B. Caspases
C. FasR
D. Bax/Bak

44. A 35-year-old Caucasian female presents to room with unilateral leg swelling. She
reports that she developed painful left-sided leg swelling and redness earlier in the
day. She is otherwise healthy and takes no medications. She denies any recent
prolonged travel. She experienced a similar episode affecting the opposite leg one
year ago and was diagnosed with a thrombus in the right femoral vein. On
examination, the left leg is erythematous and swollen. Passive dorsiflexion of the
left ankle elicits pain in the left calf. Ultrasound of the leg reveals a thrombus in the
left popliteal vein. A genetic workup reveals that she has an inherited condition.
What is the most likely pathophysiology of this patient's condition? (44) (Ch.4)
A. Elevated serum homocysteine
B. Thrombin inhibitor deficiency
C. Factor V inhibitor deficiency
D. Auto-antibodies directed against phospholipids
E. Resistance to clotting factor degradation

45. A 60-year-old women has severe chest pain for the past 7 hours. On physical
examination she is afebrile, but has tachycardia. Lab studies show elevated serum
troponin I. A coronary angiogram reveals > 70% occlusion of the left anterior
descending artery. If there is an irreversible injury to myocardial fibers, which of
the following cellular changes occurs? (45) (Ch.2)
A. Glycogen stores are depleted
B. Cytoplasmic sodium increases
C. Nuclei undergo karyorrhexis
D. Intracellular pH diminishes
E. Blebs form on cell membranes
46. In an evaluation of an 9-year-old boy who has had recurrent infections since the first
year of life, findings include enlargement of the liver and spleen, lymph node
inflammation, and a superficial dermatitis resembling eczema. Microscopic
examination of a series of peripheral blood smears taken during the course of a
staphylococcal infection indicates that the bactericidal capacity of the boy's
neutrophils is impaired or absent. Which of the following is the most likely cause of
this child's illness?
A. Defect in the enzyme NADPH oxidase
B. Defect in the enzyme adenosine deaminase (ADA)
C. Defect in the IL-2 receptor
D. Developmental defect at the pre-B stage
E. Developmental failure of pharyngeal pouches 3 and 4

47. During the early stages of the inflammatory response, histamine


induced vascular permeability is most likely to occur in (Ch.3)
A. Arteries
B. Precapillary
C. Arterioles
D. Capillaries
E. Postcapillary venules
F. Veins

48. Which of these enzymes is considered a tumor antigen? (Ch.6)


A. Hexosaminidase A
B. Telomerase
C. Collagenase
D. Tyrosinase

49. A patient presents with a large wound to his right forearm that is the result of chain
saw accident. You treat his wound appropriately and follow him in your surgery
clinic at routine intervals. Initially his wound is filled with granulation tissue, which is
composed of proliferating fibroblasts and proliferating new blood vessels
(angiogenesis). Which of the following growth factors is capable of inducing all the
steps necessary for angiogenesis is
A. Epidermal growth factor (EGF)
B. Transforming growth factor (TGF-a)
C. Platelet-derived growth factor (PDGF)
D. Basic fibroblast growth factor (FGF)
E. Transforming growth factor (TGF-b)

50. A 29-year-old female presents to her OB/GYN for a prenatal visit. She has just
learned that she is pregnant and estimates she is approximately 6 week along
given the timing of her last menstrual cycle. She has a history of recurrent fetal
loss. In total, she has had 4 miscarriages between the gestational ages of 8 and
15 weeks. Which of the following is the most likely diagnosis? (51) (Ch.4)
A. Protein C deficiency
B. Protein C deficiency
C. Antiphospholipid syndrome
D. Factor V Leiden mutation
E. Disseminated intravascular Coagulation
51. A 64-year-old male presents to the emergency department with sudden onset of
pleuritic chest pain and dyspnea on exertion. He has a history of lung cancer and is
currently being treated with outpatient chemotherapy. Studies reveal a pulmonary
embolus in the right superior segmental pulmonary artery. He is admitted to the ICU
and begun on unfractionated heparin (UFH). Seven days later you note the following
CBC: WBC 11,000 / Hgb 11 g/dl / Hct 42% / Platelets 130,000. On admission, his
platelet count was 340,000. What is the most likely diagnosis?
A. Protein C deficiency
B. Heparin-induced Thrombocytopenia
C. Antiphospholipid syndrome
D. Factor V Leiden mutation
E. Homocyteinemia

52. A 31-year-old woman with history of heavy and painful menstrual periods has
been having difficulty conceiving despite months of trying to become pregnant.
Bimanual pelvic examination and an ultrasound demonstrates a mass in the
uterus is presumed to be a leiomyoma. This mass is a (Ch.6)(BRS)
A. Malignant tumor of epithelial tissue
B. Malignant tumor of glandular tissue
C. Malignant tumor of mesenchymal tissue
D. Benign tumor of mesenchymal tissue
E. Benign tumor of surface epithelium

53. Pulmonary hypertension can be a complication of sclerodoma.


A. True
B. False

54. Biopsy performed on a 44 year old man with a gastric showed well-
developed and normal looking pancreatic cells. This constitutes a
A. Gastric carcinoma
B. Pancreatic adenocarcinoma metastasis
C. Pancreatic adenoma
D. Hamartroma
E. Choristoma

55. A 25-year-old woman begins breast feeding the infant. Which of the following
cellular processes that began in the breast during pregnancy allowed her to
breastfeed the infant for this period of time?
A. Epithelial dysplasia
B. Ductal epithelial metaplasia
C. Lobular hyperplasia
D. Stromal hypertrophy
56. What is the most common associated disorder with Sjogren syndrome? (Ch.5)
A. Scleroderma
B. Autoimmune thyroid disease
C. Polyomiositis
D. SLE
E. Vasculitits
F. Mixed connective tissue disease
G. Rheumatoid arthritis

57. A 60-year-old man with a history of multiple risk factors for coronary artery
disease (CAD) develops acute myocardial infarction. Thrombolytic therapy
is administered to restore coronary blood flow. After this therapy, the
myocardial fiber damage may increase because of which of the following
abnormalities?
A. Nuclear chromatin clumping
B. Reduced protein synthesis
C. Decreased intracellular pH from anaerobic glycolysis
D. Mitochondrial swelling
E. Increased free radical formation
F. Cytoskeletal intermediate filament loss

58. A 32-year-old pregnant, Rh- woman presents to the clinic for a routine
ultrasound; The fetus has become anemic and levels of unconjugated bilirubin
have increased dramatically. Which type of hypersensitivity reaction has most
likely occurred and what is mediating the disease state?
A. Type I hypersensitivity; antibody-mediated opsonization
B. Type IV hypersensitivity, cell killing via NK cells
C. Type II hypersensitivity, cell killing via cytotoxic T cells
D. Type II hypersensitivity, antibody-mediated opsonization
E. Type Il hypersensivity, cell killing via cytotoxic T cells

59. Which of these conditions can mimic Sjogren syndrome histologically?


A. Hyperthyroidism
B. Chronic sialdentisis
C. Scleroderma
D. Chronic vasculitis

Final V2
60. 34 year-old woman was diagnosed with drug induced lupus-like syndrome.
Which of the following drugs might have caused her condition?
A. Hydralazine
B. Rifampin
C. Amiodarone
D. Nifedipine
E. Propranolol
(hydralazine, procainamide, isoniazid, and D-penicillamine.)

61. 34 year-old woman was diagnosed with drug induced lupus-like


syndrome. Which of the following drugs might have caused her
condition?
A. Isoniazid
B. Lidocaine
C. Aspirin
D. Nicardipine
E. Propranolol
(hydralazine, procainamide, isoniazid, and D-penicillamine.)

62. 34 year-old woman was diagnosed with drug induced lupus-like


syndrome. Which of the following can be useful to (!!!)
A. Complement levels
B. Joint pain
C. Discoid Rash
D. Malar Rash
E. fatigue

63. 34 year-old woman was diagnosed with drug induced lupus-like syndrome.
Which…
A. Antihistone antibodies
B. Libman sacks endocarditis
C. Wight loss
D. Arthralgia
E. Subfebrile Temperature

64. A 56-year-old man dies of a 15-year progressive illness characterized by athetoid


movements and deterioration leading to hypertonicity, fecal and urine
incontinence, anorexia and weight loss, and eventually dementia and death. The
disease is known to have an autosomal dominant mode of inheritance and to be
due to an abnormality in a gene on chromosome 4 that is altered by increased
numbers of intragenic trinucleotide repeats. In addition, this disorder has an
earlier onset and is more debilitating in successive generations, a phenomenon
that might be due to
A. a shift from trinucleotide repeats to pentanucleotide repeats.
B. an increase in the number of trinucleotide repeats in successive generations.
C. defects in membrane receptors and transport systems.
D. imprinting variability in successive generations.
E. increased medical awareness of the condition.

65. The parents of a 17-year-old boy with Down syndrome seek


counseling because they are concerned that their son may develop a
life-threatening disorder known to be associated with his
chromosomal abnormality. The physician should be prepared to
discuss which of the following disorders in terms of its association
with Down syndrome?
A. Berry aneurysm of the circle of Willis
B. Creutzfeldt-Jakob disease
C. Lymphoblastic leukemia
D. Medullary carcinoma of the thyroid

66. Which of these alleles is most important for tissue grafting?


A. P53
B. U1rubonucleoprotein
C. HLA
D. Hemoglobin

67. Which protaglandin mediates pain?


A. PGI2
B. PGF2
C. PGD2
D. PGE2

68. What is Miculicz syndrome?


A. Hereditary pulmonary fibrosis
B. Hypertrophic cardiomyopathy and thyroiditis
C. Enlargment and fibrosis of salivary and lacrimal glands
D. Hyperplasia of the billiary tree

69. 30 year-old woman presents to her physician with malar rash and joint
pain. Her laboratory study shows increased Antihistone antibodies. Which
of the followins is the most likely cause of her condition?
A. Myasthenia Gravis
B. Rheumatoid Arthritis
C. Drug induced lupus-like syndrome
D. Systemic lupus erythematosus
E. Sogren Syndrome

70. A patient suffers a stroke and has left sided weakness and paralysis in
the upper extremity. The type of necrosis associated with a well-
developed infarct of the brain is:
A. Coagulative necrosis
B. Enzymatic fat necrosis
C. Liquefactive necrosis
D. Gangrenous necrosis
71. Which of the following cellular adaptation to stress is described
below? ,,Replacement of one fully differentiated cell type by another
type of cell ''
A. Hyperplasia
B. Dysplasia
C. Metaplasia
D. Atrophy
E. Hypertrophy

72. As part of a fourth-year elective, a medical student rotating through a medical


genetics service is assigned to counsel a patient who is concerned about a
family history of hypertension. To be properly prepared for the counseling
session, the student reviews course notes on modes of inheritance of various
disorders. Knowledge of which of the following modes of inheritance is most
pertinent to the upcoming discussion with the patient?
A. Autosomal dominant
B. Autosomal recessive
C. Multifactorial
D. X-linked dominant

73. Deficiency of which of the following proteins will lead to increased


bleeding time?
A. Coagulation Factor VII
B. Coagulation Factor X
C. Prothrombin mutation
D. Glycoprotein Ib
E. Protein C Deficiency

74. Which of these drugs proved clinically benefitial for patients with
IgG4-related disease?
□ Rituximab
□ Alprazolam
□ Ibuprofen
□ Methotrexate

75. Which of these is not an component of acute inflammation?


A. Neutrophils
B. Hemodynamic changes
C. Chemical mediators
D. Lymphocytes

76. Which group of people is most often affected by IgGr-related diseases?


A. Young men
B. Young women
C. Older females
D. Middle-aged and odlder men
77. A 25-year-old machinist is injured by a metal sliver in his left hand. Over
the next few days, the wounded area becomes reddened, tender, swollen,
and feels warm to the touch. Redness at the site of injury in this patient is
caused primarily by which of the following mechanisms?
A. Hemorrhage
B. Hemostasis
C. Neutrophil margination
D. Vasoconstriction
E. Vasodilation

78. What is rhe first stage of hemodynamic changes of acute inflammation?


A. Massive vasodilation
B. Neutrophil margination
C. Transient vasoconstriction
D. Increased vascular permeability

79. 20 years old man presents to the physician with increased tendency of
bleeding. Genetic testing has shown a mutation of platelet protein important
for Fibrinogen binding. Which of the following proteins is mutated?
A. Glycoprotein IIb/IIIa
B. Von Willebrand Factor
C. Glycoprotein Ib
D. Platelet Factor 4
E. B2 Integrin

80. 20 years old man presents to the physician with increased tendency of bleeding.
Genetic testing has shown a mutation of platelet protein important for
Subendothelial collagen adhesion. Which of the following proteins is mutated?
A. Glycoprotein IIb/IIIa
B. Von Willebrand Factor
C. Glycoprotein Ib
D. Platelet Factor 4
E. B2 Integrin

81. A circumscribed mass of light yellow crumbly to pasty “cheese-like''


material associated microscopically with a macrophage response is
characteristic of?
A. Caseous necrosis
B. Coagulative necrosis
C. Fibrinous necrosis
D. Gangrenous necrosis

82. A 14-year-old girl with amenorrhea is concerned because of the delayed


onset of menses. She has shortened stature and a wide, webbed neck; broad
chest; and secondary sexual characteristics consistent with those of a much
younger girl. Which of the following chromosomal changes is most consistent
with these findings?
A. 5p-
B. 22q11-
C. 45,XO
D. 46,XY

83. A 19-year-old college sophomore is referred by his ophthalmologist


because of the finding of ectopia lentis (dislocation of the lens), which
has resulted in visual difficulties that have interfered with his
performance on the varsity basketball team. The patient is very tall,
with long limbs and long, slender, spiderlike fingers. His chest has a
“caved-in” appearance, and he also has a modest degree of scoliosis.
A midsystolic “click” is heard, and an echocardiogram reveals mitral
valve prolapse. The most likely diagnosis is
A. Ehlers-Danlos syndrome
B. Fabry disease
C. Hurler syndrome
D. Marfan syndrome
84. Cholesterol crystals detached from atherosclerotic plaques, tumor
cells, bone marrow emboli, and parts of bullets are all examples of
what type of emboli?
A. Gas emboli
B. Air emboli
C. Solid particle emboli
D. Septic emboli

85. Which of the following preformed substances is released from mast


cells and platelets, resulting in increased vascular permeability in the
lungs of the patient described in the previous question?
A. Bradykinin
B. Hageman factor
C. Histamine
D. Leukotrienes (SRS-A)
E. Thromboxane A2

86. A 58-year-old woman is brought to the emergency department 4 hours after


vomiting blood and experiencing bloody stools. The patient was diagnosed with
alcoholic cirrhosis 2 years ago. Endoscopy reveals large esophageal varices, one
of which is actively bleeding. Which of the following best explains the
pathogenesis of dilated esophageal veins in this patient?
A. Decreased intravascular oncotic pressure
B. Increased capillary permeability
C. Increased intravascular hydrostatic pressure
D. Vasoconstriction of arterioles
E. Vasodilatation of capillaries
87. A 39-year-old man has a health screening examination. He has a routine chest
x-ray that shows a 2.5 cm nodule in the right lower lobe. The nodule has focal
calcifications. A wedge resection of the nodule is done. On microscopic
examination the nodule shows caseous necrosis and calcification. Which of
the following processes explains the appearance of the calcium deposition:
A. Excessive ingestion of calcium
B. Apoptosis
C. Vitamin D excess
D. Metastatic calcification
E. Dystrophic calcification

88. IgG4-related disease can affect every organ.


A. True
B. FALSE

89. Which of these prostaglandins mediates fever?


A. PGI2
B. PGD2
C. PGF2
D. PGE2
90. Laproscopic examination of the abdomen was performed on a 50
year old chronic alcoholic man. The surgeon noted digestion of tissue
with soap formation and calcification. Which of the following is this
most likely characteristic of?
A. Coagulative necrosis
B. Caseous necrosis
C. Enzymatic fat necrosis
D. Liquefactive necrosis
E. Apoptosis

91. A newly described neurologic disorder is found to affect multiple family member
in three generations that were available for study. In the first generation, two
sisters and one brother were affected. In the second generation, all of the children
of the first-generation sisters were affected, but none of the descendants of the
first- generation son. In the third generation, all of the children of the affected
second- generation women were affected, but none of the descendants of the
second generation men. The mode of inheritance exemplified here is
A. autosomal dominant
B. autosomal recessive
C. mitochondrial
D. X-linked dominant

92. What is the function of Thromboxane A2?


A. Vasoconstriction and platelet aggregation
B. Vasodilation
C. Clot dissolution
D. Vasodilation and platelet aggregation
93. A 2-year-old child has been followed for mental retardation and slow development,
as well as multiple birth defects. The child has a high-pitched catlike cry. On
examination, microcephaly, hypertelorism, micrognathia, epicanthal folds, low-set
ears, and hypotonia are noted. Karyotypic analysis would be expected to show
A. 5p-
B. 22q11-
C. 45,XO
D. 46,XY

94. A 20-year-old woman presents to the emergency room complaining of having


had a severe headache for 4 hours. Physical examination reveals numerous
small red spots on the extremities and a stiff neck. Her temperature is 38.7°C.
Lumbar puncture returns purulent fluid, with segmented neutrophils and Gram-
negative organisms resembling meningococci. A few hours later, the patient
goes into shock and becomes comatose. Severe endothelial injury in this
patient is primarily mediated by which of the following proteins?
A. α-Fetoprotein
B. IgG
C. Interferon-γ
D. Transforming growth factor-β
E. Tumor necrosis factor-α

95. A 70 years-old man with a history of multiple risk factors for coronary artery
disease(CAD) develops acute myocardial infarction. Thrombolytic therapy is
administered to restore coronary blood flow. After this therapy, the myocardial
fiber damage may increase because of which of the following abnormalities?
A. Cytoskeletal intermediate filament loss
B. Mitochondrial swelling
C. Reduced protein synthesis
D. Increased free radical formation
E. Decreased intracellular pH from anaerobic glycolysis

96. A 23-year-old woman complains of a recent onset of yellowing of her skin and
increasing abdominal girth. Physical examination reveals jaundice and ascites.
Ultrasound examination of her abdomen demonstrates thrombosis of the hepatic
veins. A liver biopsy discloses severe sinusoidal dilation within the centrilobular
regions. This pathologic finding is caused by which of the following hemodynamic
disorders?
A. Active hyperemia
B. Arterial embolism
C. Hematoma
D. Hemorrhage
E. Passive hyperemia

97. A screening test for phenylketonuria (PKU) is performed on umbilical cord blood
from a fair-skinned blond, blue-eyed infant born to dark-complexioned parents. The
test is reported as negative, and no dietary restrictions are imposed. At 1 year of age,
the child is seen again, this time with obvious signs of severe mental retardation, and
a diagnosis of PKU is made. The diagnosis was missed at birth because
A. cord blood is not a good source of fetal blood
B. the screening (Guthrie) test has low sensitivity
C. test should have been performed on maternal blood
D. the test was performed too early

98. 65 year old man presents to emergency department with severe and porolonged
chest pain. ECG show ST segment elevations. Soon after admission patient died.
Autopsy showed occlusion of left anterior descending artery. Which of the
following favored occlusion formation?
A. PGI2
B. NO
C. TXA2
D. Bradykinin
E. Vasodilation

99. Grafts exchanged between individuals of the same species are called
A. Autograft
B. Xenograft
C. Allograft
D. Alloplast

100. Which of the following cells are dominating the early phase of
type I Hypersensitivity reaction?
A. Eosinophils
B. Neutrophils
C. Mast cells
D. B Lymphocytes
E. T Lymphocytes

101. During a routine physical examination, a 41-year-old woman is noted to have


blueblack pigmented patches in the sclerae and gray-blue discoloration of the ear
cartilages. The extensor tendons of the hands exhibit similar discoloration when she
is asked to “make a fist.” On questioning, the patient vaguely remembers hearing
her mother say that the patient had dark discoloration on her diapers when she was
an infant. Her only current complaint is slowly increasing pain and stiffness of the
lower back, hips, and knees. A urine sample darkens on standing. These findings are
characteristic of a deficiency of which of the following enzymes?
A. Homogentisic oxidase
B. L-Iduronosulfate sulfatase
C. Ketoacid decarboxylase
D. Phenylalanine hydroxylase

102. A 50-year-old fire fighter emerges from a burning house with third-degree
burns over 70% of his body. The patient expires 24 hours later. Which of the
following was the most likely cause of death?
A. Congestive heart failure
B. Disseminated intravascular coagulation
C. Hypovolemic shock
D. Pulmonary saddle embolism
E. Toxic shock syndrome

103. A 1-year-old female infant is hospitalized for pneumonia. Bacterial cultures


of the sputum have grown Pseudomonas aeruginosa. She has had two prior
hospitalizations for severe respiratory infections. Her mother has noted that when
she kisses her child, the child tastes “salty.” The child has had weight loss that the
mother attributes to frequent vomiting and diarrhea with bulky,foul-smelling fatty
stools. The child is small for her age. Which of the following critical proteins is
altered in this condition?
A. Cystic fibrosis transmembrane conductance regulator
B. Dystrophin
C. a-1,4-Glucosidase
D. Lysyl hydroxylase

104. Which of these disease produces systemic fibrosis?


A. Scleroderma
B. SLE
C. Riedel thyroiditis
D. Rheumatoid arthritis

105. A pathologist notes cloudy swelling, hydropic change and fatty change in
the liver of a patient with a history of alcohol abuse. These morphological
changes are all examples of:
A. Early neoplastic change
B. Hyaline change
C. Patterns of cell death
D. Postmortem artefact
E. Reversible cell injury

106. A 56-year-old man dies of a 15-year progressive illness characterized by


athetoid movements and deterioration leading to hypertonicity, fecal and urine
incontinence, anorexia and weight loss, and eventually dementia and death. The
disease is known to have an autosomal dominant mode of inheritance and to be due
to an abnormality in a gene on chromosome 4 that is altered by increased numbers
of intragenic trinucleotide repeats. In addition, this disorder has an earlier onset and
is more debilitating in successive generations, a phenomenon that might be due to
A. a shift from trinucleotide repeats topentanucleotide repeats
B. an increase in the number of trinucleotiderepeats in successive generations
C. defects in membrane receptors and transport systems
D. imprinting variability in successivegenerations

107. Which of the following is essential for Type III Hypersensitivity reaction?
A. Immune complex formation
B. Antibody binding to cell surface antigen
C. T helper cells activating cell mediated immunity
D. Mast cell degranulation
E. Degranulation of Basophils

108. Which of the following genes in BCL gene family is anti-apoptotic?


A. BAD gene
B. BH3 gene
C. BAX gene
D. Bcl-2
E. BAK gene

109. Laboratory practitioner is studying the function platelets. He


has observed that addition of certain substance to the platelet
medium causes increased fibrinogen binding to the cells. Which of the
following substances is this practitioner is using?
A. Serotonin
B. ADP
C. Platelet Factor 4
D. Von Willebrand Factor
E. TXA2

110. An 8-year-old girl with asthma presents with respiratory distress. She has a
history of allergies and upper respiratory tract infections. She also has history of
wheezes associated with exercise. Which of the following mediators of inflammation
is the most powerful stimulator of bronchoconstriction and vasoconstriction in this
patient?
A. Bradykinin
B. Complement proteins
C. Interleukin-1
D. Leukotrienes
E. Tumor necrosis factor-α

111. A 54-year-old woman who has been diagnosed with early-stage breast
cancer undergoes surgery for a lumpectomy to remove a small tumor detected by
mammography. The pathology report confirms the early stage of the cancer and
further comments on the fact that there is significant desmoplasia in the
surrounding tissue. The term desmoplasia refers to
A. an irregular accumulation of blood vessels
B. maturation and spatial arrangement of cells
C. metastatic involvement of surrounding tissue
D. proliferation of non-neoplastic fibrous connective tissue

112. A 69-year-old man is brought to the emergency room complaining of


visual difficulty and weakness. On physical examination, the patient is aphasic
with a right-sided hemiplegia. Retinal hemorrhages are seen bilaterally. You
suspect that a thromboembolus coursed to the left middle cerebral artery and
smaller emboli traveled to the retinal arteries. Which of the following anatomic
sites is the most likely source for these emboli in this patient?
A. Adrenals
B. Deep leg veins
C. Heart
D. Liver
E. Lungs

113. A 32-year-old male patient admits to the ER after a femoral artery stabbing
wound. What type of emboli would you most suspect to occur in this patient?
A. Gas Emboli
B. Fat Emboli
C. Pulmonary emboli
D. Systemic thromboembolism

114. Which of the following cellular changes is not characteristic for


reversible cell injury?
A. The cytoplasm may contain so-called “myelin figures,” which
are collections of phospholipids
B. Nuclear changes such as Karyorrhexis
C. Cellular swelling
D. Membrane blebs
115. A 12-year-old black man with a known history of sickle cell disease presents
to the emergency department complaining of left upper quadrant pain suggestive
of a splenic infarct. Microscopic examination of the spleen would most likely reveal
A. Fat necrosis
B. Coagulative necrosis
C. Fibrinoid necrosis
D. Gangrenous necrosis
E. Liquefactive necrosis

116. A 20-year-old woman has a robertsonian translocation involving chromosome


21 and a second acrocentric chromosome. What is the theoretic likelihood of a
functional trisomy 21 if one of her ova is fertilized by a normal sperm?
A. 1 in 1
B. 1 in 2
C. 1 in 3
D. 1 in 4
E. 1 in 1500

117. A 59-year-old man with ischemic heart disease and a history of smoking
complains of increasing shortness of breath. On physical examination, the
patient has swollen legs, an enlarged liver, and fluid in the pleural spaces (bubbly
rales are heard on auscultation). Which of the following hemodynamic disorders
explains the pathogenesis of hepatomegaly in this patient?
A. Arterial thromboembolism
B. Chronic passive congestion
C. Deep venous thrombosis
D. Multiple hepatic infarcts
E. Thrombosis of the hepatic vein

118. Where do most arterial emboli originate?


A. Left atrium or ventricle, aorta, and major arteries
B. Pulmonary vein
C. Deep vein of lower limbs
D. Right atrium, inferior vena cava, and major veins

119. Which of these individuals would not have differences in HLA allele (!!!)
A. Mother and son
B. Siblings
C. Father and daughter
D. Identical twins
Because HLA genes are highly polymorphic, there are always some differences between
individuals (except, of course, identical twins).

120.
Question Text Answer 1 Answer 2 Answer 3 Answer 4 Answer 5
Which of the following genes in BCL gene family is anti-apoptotic? BAD gene BH3 gene BAX gene [Y]Bcl-2 BAK gene
A 70 years-old man with a history of multiple risk factors for coronary Cytoskeletal intermediate filament loss Mitochondrial swelling Reduced protein synthesis [Y]Increased free radical formation Decreased intracellular pH from anaerobic
artery disease(CAD) develops acute myocardial infarction. Thrombolytic glycolysis
therapy is administered to restore coronary blood flow. After this therapy,
the myocardial fiber damage may increase because of which of the
following abnormalities?
Which of the following cellular changes is not characteristic for reversible The cytoplasm may contain so-called “myelin [Y]Nuclear changes such as Karyorrhexis Cellular swelling Membrane blebs
cell injury? figures,” which are collections of phospholipids
Which of the following cellular adaptation to stress is described below? ,, Hyperplasia Dysplasia [Y]Metaplasia Atrophy Hypertrophy
Replacement of one fully differentiated cell type by another type of cell ''
A 12-year-old black man with a known history of sickle cell disease Fat necrosis [Y]Coagulative necrosis Fibrinoid necrosis Gangrenous necrosis Liquefactive necrosis
presents to the emergency department complaining of left upper quadrant
pain suggestive of a splenic infarct. Microscopic examination of the spleen
would most likely reveal
A pathologist notes cloudy swelling, hydropic change and fatty change in Early neoplastic change Hyaline change Patterns of cell death Postmortem artefact [Y]Reversible cell injury
the liver of a patient with a history of alcohol abuse. These morphological
changes are all examples of:
A circumscribed mass of light yellow crumbly to pasty “cheese-like'' [Y]Caseous necrosis Coagulative necrosis Fibrinous necrosis Gangrenous necrosis
material associated microscopically with a macrophage response is
characteristic of?
Laproscopic examination of the abdomen was performed on a 50 year old Coagulative necrosis Caseous necrosis [Y]Enzymatic fat necrosis Liquefactive necrosis Apoptosis
chronic alcoholic man. The surgeon noted digestion of tissue with soap
formation and calcification. Which of the following is this most likely
characteristic of?
A patient suffers a stroke and has left sided weakness and paralysis in the Coagulative necrosis Enzymatic fat necrosis [Y]Liquefactive necrosis Gangrenous necrosis
upper extremity. The type of necrosis associated with a well-developed
infarct of the brain is:
A 39-year-old man has a health screening examination. He has a routine Excessive ingestion of calcium Apoptosis Vitamin D excess Metastatic calcification [Y]Dystrophic calcification
chest x-ray that shows a 2.5 cm nodule in the right lower lobe. The nodule
has focal calcifications. A wedge resection of the nodule is done. On
microscopic examination the nodule shows caseous necrosis and
calcification. Which of the following processes explains the appearance of
the calcium deposition:
An 8-year-old girl with asthma presents with respiratory distress. She has a Bradykinin Complement proteins Interleukin-1 [Y]Leukotrienes Tumor necrosis factor-α
history of allergies and upper respiratory tract infections. She also has
history of wheezes associated with exercise. Which of the following
mediators of inflammation is the most powerful stimulator of
bronchoconstriction and vasoconstriction in this patient?
Which of the following preformed substances is released from mast cells Bradykinin Hageman factor [Y]Histamine Leukotrienes (SRS-A) Thromboxane A2
and platelets, resulting in increased vascular permeability in the lungs of
the patient described in the previous question?
A 25-year-old machinist is injured by a metal sliver in his left hand. Over Hemorrhage Hemostasis Neutrophil margination Vasoconstriction [Y]Vasodilation
the next few days, the wounded area becomes reddened, tender, swollen,
and feels warm to the touch. Redness at the site of injury in this patient is
caused primarily by which of the following mechanisms?
Which of these prostaglandins mediates fever? PGI2 PGD2 PGF2 [Y]PGE2
What is the function of Thromboxane A2? [Y]Vasoconstriction and platelet aggregation Vasodilation Clot disolution Vasodilation and platelet aggregation
Which protaglandin mediates pain? PGF2 PGD2 [Y]PGE2 PGI2
Which of these is not an component of acute inflammation? Neutrophils Hemodynamic changes Chemical mediators [Y]Lymphocytes
What is rhe first stage of hemodynamic changes of acute inflammation? Massive vasodilation Neutrophil margination [Y]Transient vasoconstriction Increased vascular permeability
65 year old man presents to emergency department with severe and PGI2 NO [Y]TXA2 Bradykinin Vasodilation
porolonged chest pain. ECG show ST segment elevations. Soon after
admission patient died. Autopsy showed occlusion of left anterior
descending artery. Which of the following favored occlusion formation?
20 years old man presents to the physician with increased tendency of [Y]Glycoprotein IIb/IIIa Von Willebrand Factor Glycoprotein Ib Platelet Factor 4 B2 Integrin
bleeding. Genetic testing has shown a mutation of platelet protein
important for Fibrinogen binding. Which of the following proteins is
mutated?
20 years old man presents to the physician with increased tendency of Glycoprotein IIb/IIIa Von Willebrand Factor [Y]Glycoprotein Ib Platelet Factor 4 B2 Integrin
bleeding. Genetic testing has shown a mutation of platelet protein
important for Subendothelial collagen adhesion. Which of the following
proteins is mutated?
Laboratory practitioner is studying the function platelets. He has observed Serotonin [Y]ADP Platelet Factor 4 Von Willebrand Factor TXA2
that addition of certain substance to the platelet medium causes increased
fibrinogen binding to the cells. Which of the following substances is this
practitioner is using?
Deficiency of which of the following proteins will lead to increased Coagulation Factor VII Coagulation Factor X Prothrombin mutation [Y]Glycoprotein Ib Protein C Deficiency
bleeding time?
Where do most arterial emboli originate? [Y]Left atrium or ventricle, aorta, and major Pulmonary vein Deep vein of lower limbs Right atrium, inferior vena cava, and major veins
arteries
Cholesterol crystals detached from atherosclerotic plaques, tumor cells, Gas emboli Air emboli [Y]Solid particle emboli Septic emboli
bone marrow emboli, and parts of bullets are all examples of what type of
emboli?
A 32-year-old male patient admits to the ER after a femoral artery stabbing Gas Emboli [Y]Fat Emboli Pulmonary emboli Systemic thromboembolism
wound. What type of emboli would you most suspect to occur in this
patient?
A 58-year-old woman is brought to the emergency department 4 hours Decreased intravascular oncotic pressure Increased capillary permeability [Y]Increased intravascular hydrostatic pressure Vasoconstriction of arterioles Vasodilatation of capillaries
after vomiting blood and experiencing bloody stools. The patient was
diagnosed with alcoholic cirrhosis 2 years ago. Endoscopy reveals large
esophageal varices, one of which is actively bleeding. Which of the
following best explains the pathogenesis of dilated esophageal veins in this
patient?
A 20-year-old woman presents to the emergency room complaining of α-Fetoprotein IgG Interferon-γ Transforming growth factor-β [Y]Tumor necrosis factor-α
having had a severe headache for 4 hours. Physical examination reveals
numerous small red spots on the extremities and a stiff neck. Her
temperature is 38.7°C. Lumbar puncture returns purulent fluid, with
segmented neutrophils and Gram-negative organisms resembling
meningococci. A few hours later, the patient goes into shock and becomes
comatose. Severe endothelial injury in this patient is primarily mediated by
which of the following proteins?
A 69-year-old man is brought to the emergency room complaining of Adrenals Deep leg veins [Y]Heart Liver Lungs
visual difficulty and weakness. On physical examination, the patient is
aphasic with a right-sided hemiplegia. Retinal hemorrhages are seen
bilaterally. You suspect that a thromboembolus coursed to the left middle
cerebral artery and smaller emboli traveled to the retinal arteries. Which of
the following anatomic sites is the most likely source for these emboli in
this patient?
A 59-year-old man with ischemic heart disease and a history of smoking Arterial thromboembolism [Y]Chronic passive congestion Deep venous thrombosis Multiple hepatic infarcts Thrombosis of the hepatic vein
complains of increasing shortness of breath. On physical examination, the
patient has swollen legs, an enlarged liver, and fluid in the pleural spaces
(bubbly rales are heard on auscultation). Which of the following
hemodynamic disorders explains the pathogenesis of hepatomegaly in this
patient?
A 50-year-old fire fighter emerges from a burning house with third-degree Congestive heart failure Disseminated intravascular coagulation [Y]Hypovolemic shock Pulmonary saddle embolism Toxic shock syndrome
burns over 70% of his body. The patient expires 24 hours later. Which of
the following was the most likely cause of death?
A 23-year-old woman complains of a recent onset of yellowing of her skin Active hyperemia Arterial embolism Hematoma Hemorrhage [Y]Passive hyperemia
and increasing abdominal girth. Physical examination reveals jaundice and
ascites. Ultrasound examination of her abdomen demonstrates thrombosis
of the hepatic veins. A liver biopsy discloses severe sinusoidal dilation
within the centrilobular regions. This pathologic finding is caused by
which of the following hemodynamic disorders?
30 year-old woman presents to her physician with malar rash and joint [Y]Systemic lupus erythematosus Sogren Syndrome Drug induced lupus-like syndrome Rheumatoid Arthritis Myasthenia Gravis
pain. Her laboratory study shows increased Antihistone antibodies. Which
of the followins is the most likely cause of her condition?
34 year-old woman was diagnosed with drug induced lupus-like syndrome. [Y]Hydralazine Rifampin Amiodarone Nifedipine Propranolol
Which of the following drugs might have caused her condition?
Which of the following cells are dominating the early phase of type I Eosinophils Neutrophils [Y]Mast cells B Lymphocytes T Lymphocytes
Hypersensitivity reaction?
Which of the following is essential for Type III Hypersensitivity reaction? [Y]Immune complex formation Antibody binding to cell surface antigen T helper cells activating cell mediated immunity Mast cell degranulation Degranulation of Basophils
IgG4-related disease can affect every organ. [Y]True FALSE
What is Miculicz syndrome? Hyperplasia of the billiary tree Hypertrophic cardiomyopathy and thyroiditis [Y] Enlargment and fibrosis of salivary and Hereditary pulmonary fibrosis
lacrimal glands
Which group of people is most often affected by IgGr-related diseases? Young men Young women Older females [Y]Middle-aged and odlder men
Which of these drugs proved clinically benefitial for patients with IgG4- [Y]Rituximab [Y]Alprazolam Ibuprofen Methotrexate
related disease?
Which of these disease produces systemic fibrosis? [Y]Scleroderma SLE Riedel thyroditis Rheumatoid arthritis
Grafts exchanged between individuals of the same species are called Autograft Xenograft [Y]Allograft Alloplast
A 19-year-old college sophomore is referred by his ophthalmologist
because of the finding of ectopia lentis (dislocation of the lens), which has
resulted in visual difficulties that have interfered with his performance on
the varsity basketball team. The patient is very tall, with long limbs and
long, slender, spiderlike fingers. His chest has a “caved-in” appearance,
and he also has a modest degree of scoliosis. A midsystolic “click” is
heard, and an echocardiogram reveals mitral valve prolapse. The most
likely diagnosis is Ehlers-Danlos syndrome Fabry disease Hurler syndrome [Y]Marfan syndrome
A 20-year-old woman has a robertsonian translocation involving
chromosome 21 and a second acrocentric chromosome. What is the
theoretic likelihood of a functional trisomy 21 if one of her ova is fertilized
by a normal sperm? 1 in 1 1 in 2 [Y]1 in 3 1 in 4 1 in 1500
A 1-year-old female infant is hospitalized for pneumonia. Bacterial
cultures of the sputum have grown Pseudomonas aeruginosa. She has had
two prior hospitalizations for severe respiratory infections. Her mother has
noted that when she kisses her child, the child tastes “salty.” The child has
had weight loss that the mother attributes to frequent vomiting and diarrhea
with bulky,foul-smelling fatty stools. The child is small for her age. Which [Y]Cystic fibrosis transmembrane conductance
of the following critical proteins is altered in this condition? regulator Dystrophin a-1,4-Glucosidase Lysyl hydroxylase
A newly described neurologic disorder is found to affect multiple family
member in three generations that were available for study. In the first
generation, two sisters and one brother were affected. In the second
generation, all of the children of the first-generation sisters were affected,
but none of the descendants of the first- generation son. In the third
generation, all of the children of the affected second- generation women
were affected, but none of the descendants of the second generation men.
The mode of inheritance exemplified here is autosomal dominant autosomal recessive [Y]mitochondrial X-linked dominant
As part of a fourth-year elective, a medical student rotating through a
medical genetics service is assigned to counsel a patient who is concerned
about a family history of hypertension. To be properly prepared for the
counseling session, the student reviews course notes on modes of
inheritance of various disorders. Knowledge of which of the following
modes of inheritance is most pertinent to the upcoming discussion with the
patient? Autosomal dominant Autosomal recessive [Y]Multifactorial X-linked dominant
A 2-year-old child has been followed for mental retardation and slow
development, as well as multiple birth defects. The child has a high-pitched
catlike cry. On examination, microcephaly, hypertelorism, micrognathia,
epicanthal folds, low-set ears, and hypotonia are noted. Karyotypic analysis
would be expected to show [Y] 5p- 22q11- 45,XO 46,XY
The parents of a 17-year-old boy with Down syndrome seek counseling
because they are concerned that their son may develop a life-threatening
disorder known to be associated with his chromosomal abnormality. The
physician should be prepared to discuss which of the following disorders in
terms of its association with Down syndrome? Berry aneurysm of the circle of Willis Creutzfeldt-Jakob disease [Y]Lymphoblastic leukemia Medullary carcinoma of the thyroid
A 14-year-old girl with amenorrhea is concerned because of the delayed
onset of menses. She has shortened stature and a wide, webbed neck; broad
chest; and secondary sexual characteristics consistent with those of a much
younger girl. Which of the following chromosomal changes is most
consistent with these findings? 5p- 22q11- [Y]45,XO 46,XY
A 50-year-old woman of Eastern European Jewish ancestry has a history of
recurrent fractures and easy bruising and is found to have
hepatosplenomegaly and mild anemia. Serum assays reveal elevations of
chitotriosidase and angiotensin-converting enzyme. Assay of cultured
leukocytes most likely reveals marked deficiency of which of the following
enzymes? [Y]Glucocerebrosidase a-1,4-Glucosidase Hexosaminidase A a-L-Iduronidase
During a routine physical examination, a 41-year-old woman is noted to
have blueblack pigmented patches in the sclerae and gray-blue
discoloration of the ear cartilages. The extensor tendons of the hands
exhibit similar discoloration when she is asked to “make a fist.” On
questioning, the patient vaguely remembers hearing her mother say that the
patient had dark discoloration on her diapers when she was an infant. Her
only current complaint is slowly increasing pain and stiffness of the lower
back, hips, and knees. A urine sample darkens on standing. These findings
are characteristic of a deficiency of which of the following enzymes? [Y]Homogentisic oxidase L-Iduronosulfate sulfatase Ketoacid decarboxylase Phenylalanine hydroxylase
A screening test for phenylketonuria (PKU) is performed on umbilical cord
blood from a fair-skinned blond, blue-eyed infant born to dark-
complexioned parents. The test is reported as negative, and no dietary
restrictions are imposed. At 1 year of age, the child is seen again, this time
with obvious signs of severe mental retardation, and a diagnosis of PKU is test should have been performed on maternal
made. The diagnosis was missed at birth because cord blood is not a good source of fetal blood the screening (Guthrie) test has low sensitivity blood [Y]the test was performed too early
A 56-year-old man dies of a 15-year progressive illness characterized by
athetoid movements and deterioration leading to hypertonicity, fecal and
urine incontinence, anorexia and weight loss, and eventually dementia and
death. The disease is known to have an autosomal dominant mode of
inheritance and to be due to an abnormality in a gene on chromosome 4
that is altered by increased numbers of intragenic trinucleotide repeats. In
addition, this disorder has an earlier onset and is more debilitating in a shift from trinucleotide repeats [Y]an increase in the number of defects in membrane receptors and transport
successive generations, a phenomenon that might be due to topentanucleotide repeats trinucleotiderepeats in successive generations systems imprinting variability in successivegenerations
A 54-year-old woman who has been diagnosed with early-stage breast
cancer undergoes surgery for a lumpectomy to remove a small tumor
detected by mammography. The pathology report confirms the early stage
of the cancer and further comments on the fact that there is significant [Y]proliferation of non-neoplastic fibrous
desmoplasia in the surrounding tissue. The term desmoplasia refers to an irregular accumulation of blood vessels maturation and spatial arrangement of cells metastatic involvement of surrounding tissue connective tissue
A 26-year-old female presents with severe pain during menses Bradykinin Histamine Leukotrienes Phospholipase A2 [Y]Prostaglandin F2
(dysmenorrhea). To treat her symptoms, you advise her to take
indomethacin(COX Inhibitor) in the hopes that it will reduce her pain by
interfering with the production of
A 39-year-old female presents with intermittent pelvic pain. Physical Apoptosis [Y]Dystrophic calcification Enzymatic necrosis Hyperparathyroidism Metastatic calcification
examination reveals a 4-cm mass in the area of her right ovary. Histologic
sections from this ovarian mass reveal a papillary tumor with multiple,
scattered small, round, laminated calcifications. These structures are most
likely the result of
A 65-year-old woman falls in the street and strikes the back of his head. Congestion Ecchymosis [Y] Hematoma Petechiae Purpura
Over the next 24 hours, he becomes increasingly somnolent. A head CT
scan shows an accumulation of fluid beneath the dura, compressing the left
cerebral hemisphere. Which of the following terms best describes this
collection of fluid?
A 23-year-old man has a history of increased menstrual blood flow and Fibrin polymerization [Y] Platelet adhesion Platelet aggregation Prothrombin generation vasoconstriction
frequent nosebleeds. On physical examination, petechiae and purpura are
present on the skin of her extremities. patient has normal partial
thromboplastin time (PTT), prothrombin time (PT), and platelet count, but
decreased von Willebrand factor activity. This patient most likely has a
derangement in which of the following steps in hemostasis?

A 30-year-old man is cutting wood alone in the forest and incurs a deep cut Antidiuretic hormone Bicarbonate Catecholamines [Y] Lactate dehydrogenase Prothrombin
to his leg from his chain saw. He loses a large amount of blood. He is not
found until the next day. A marked increase in which of the following
blood analytes is most likely to indicate that he has reached an irreversible
stage of shock?

While camping, a 15-year-old boy is bitten by a mosquito. At the site of the Lividity Ecchymosis Contusion Congestion [Y] Hyperemia
mosquito bite, his skin
turns red and a small but noticeable bump forms.
Which of the following terms best describes the
color change in the skin?

A 66-year-old male has a long-standing history of congestive heart failure Dilated alveolar septal capillaries filled with red [Y]Fibrotic and thickened alveolar septal Loss of alveolar septa, resulting in large Congestion of centrilobular sinusoids Diffuse macrovesicular steatosis of the liver
due to uncontrolled hypertension and has had multiple admissions to the blood cells capillaries airspaces
hospital for treatment of pulmonary edema. During his most recent
admission, he sustains a fatal cardiac dysrhythmia. Which of the following
is likely to be observed in this patient’s lungs at autopsy?

A 71-year-old male with a history of poorly controlled hypertension due to [Y] Increased hydrostatic pressure Increased vascular permeability Decreased colloid osmotic pressure Lymphatic obstruction Sodium retention
noncompliance with medications is brought to the emergency room by his
family because of increasing shortness of breath. An X-ray of the chest
reveals bilateral pleural effusions and enlargement of the heart. Given these
features, of the following, which is the most likely causative mechanism
for the condition producing his shortness of breath?

A 68-year-old male with poorly controlled hypertension presents to the Bilirubin [Y] Hemosiderin Melanin Lipofuscin Calcium
emergency room with shortness of breath. A chest X-ray reveals bilateral
pleural effusions. An S3 gallop is heard and crackles (rales) are present in
both lung fields. Which of the following pigments is most likely to be
found within macrophages in his lungs?

A 60-year-old female status post right radical mastectomy and right Increased plasma oncotic pressure Renal sodium retention [Y] Increased interstitial protein Increased vascular permeability Decreased plasma albumin
axillary lymph node dissection develops chronic edema of the right arm.
Which of the following is the cause of her edema?

A 76-year-old man is evaluated in the emergency department for Septic shock Anaphylactic shock [Y] Cardiogenic shock Hypovolemic shock Distributive shock
hypotension and dyspnea. His temperature is 97.8°F (36.5°C), pulse
125/min, and blood pressure 88/58 mm Hg. Bilateral crackles are present
in the lung fields. Pedal edema is noted bilaterally. His extremities are
cool, and his urine output is minimal. A plain chest radiograph reveals
cardiomegaly and pulmonary edema. What is the pathophysiology of his
shock?
A pathologist is examining tissue removed from a 57-year-old deceased The heart A kidney The spleen The brain [Y] A lung
male. The tissue has
preservation of normal architecture; however, there is loss of nuclear and
cytoplasmic basophilia. Associated with these changes are abundant
extravasated red blood cells in the tissue. The man had no resuscitation
performed. Of the following, where did this tissue most likely originate?
A. The heart
B. A kidney
C. The spleen
D. The brain
E. A lung
A 76-year-old man is found dead at home. Microscopic examination of [Y] Pleural effusion Aortic dissection Deep venous thrombosis Disseminated intravascular coagulation Pericardial tamponade
samples of the liver and lung reveals centrilobular hemorrhagic necrosis of
the liver and hemosiderin-laden macrophages in the lungs. Of the
following, what additional finding at autopsy would be most likely?

A 40-year-old male is brought to the emergency Acute pulmonary embolism [Y] Septic shock Cardiogenic shock Anaphylactic shock Acute myocardial infarction
department by his wife because of his shortness
of breath. He reports the onset of a cough two days
prior with increasing shortness of breath. On arrival
his temperature is 100.5°F (38.0°C), his heart rate is 135/min, and blood
pressure is 88/55 mm Hg. Physical examination is unremarkable except for
tachycardia. The patient is alert and oriented. His leukocyte count is 18,000
/μL, his pCO2 on arterial blood gas is 30 mm Hg, and serum lactic acid is
slightly elevated at 4 mmol/L. A plain chest radiograph shows a right lower
lobe infiltrate. What is the most likely diagnosis?

A 37-year-old female with a history of deep venous thrombosis during her t-PA prevents platelet activation by inhibiting t-PA prevents platelet activation by inhibiting [Y] t-PA increases the degradation of fibrin. t-PA binds anti-thrombin III, leading to thrombin t-PA binds and inactivates clotting factors.
first pregnancy and two spontaneous abortions is evaluated in the GpIIb-IIIa complex. GpIb receptors. inactivation.
emergency department. Forty minutes prior to arrival she experienced the
sudden onset of weakness of the right face, right arm, and right leg.
Neurologic examination reveals flaccid paralysis of the right side of the
body with expressive aphasia without visual field defect. Magnetic
resonance imaging (MRI) of the brain and basic lab work including
complete blood count, prothrombin time (PT), and activated
thromboplastin time (aPTT) are ordered in the emergency department. The
treating physician administer tissue plasminogen activator (t-PA) to the
patient according to hospital protocol in the treatment of acute stroke.
Which of the following is true regard-
ing the mechanism of t-PA?

A 17-year-old male sustains a mid-shaft femur fracture after a fall from a Aortic dissection [y] Fat embolism Venous thromboembolism Acute myocardial infarction Disseminated intravascular coagulation
horse. The fracture
is repaired operatively and the patient is discharged. On the second
postoperative day he becomes acutely dyspneic. In the emergency
department he is hypoxic and tachypneic, appears confused, and has a
petechial rash on his neck and anterior thorax. What is the most likely
diagnosis?

A 32-year-old woman has routine lab work done as part of her annual Anticoagulation with Coumadin Oral antibiotics [Y] Warm compresses Topical corticosteroids Incision and drainage
physical examination. She returns to the clinic 3 days after her blood draw
complaining of pain in the left antecubital fossa. On examination there is
mild erythema without induration, no palpable fluctuence, and the basilic
vein is tender and palpated as a nodular “cord.” What is the appropriate
management?
A graduate student in an immunology lab is Macrophage Neutrophil Eosinophil Natural killer cell [Y] Lymphocyte
studying a certain cell that plays a role in adaptive
immunity. Of the following, which cell type might
the student be studying?

A graduate student has developed a protein that Brain Heart [Y] Intestine Liver Kidney
binds to CD3 and blocks its interaction with TCR, which is composed of γ
and δ subunits. Of the following, which organ is the student studying?
A 32-year-old male with a known severe peanut allergy is inadvertently The immediate reaction is triggered by IgM The immediate allergic reaction is triggered by [Y] This type of allergic reaction is caused by Prostaglandins do not play a role in this type of Previous exposure to the antigen is not required
exposed to peanuts by eating a homemade cookie. Within minutes of bound to antigen. activation of eosinophils. excessive a TH2 response. reaction. for this type of reaction.
exposure he develops a widespread rash and difficulty breathing, which
resolve with a self-administered epinephrine injection. Which of the
following statements is most characteristic regarding this type of allergic
reaction?

A 42-year-old woman presents to her primary [Y] Binding of antigen to IgE on mast cell Recognition of antigen associated with MHC-1 Binding of polysaccharide to membrane-bound Destruction of cells coated with IgM Inflammation due to deposition of antigen-
care physician complaining of itching and watery surfaces lectin receptors antibody complex
eyes, runny nose, and frequent sneezing. She denies fever or cough. She
experiences similar symptoms every year in the spring. Which of the
following is responsible for her symptoms?

A blood type O+ infant is delivered to a multiparous blood type O- mother Crosslinking of IgE on mast cell surfaces [Y] IgG binding to cell surfaces Deposition of antigen antibody complexes in the T-cell mediated cytotoxicity Antibody mediated activation of apoptosis
without prenatal care. The infant is born with severe anemia, jaundice, and fetal tissue
severe edema. What is the underlying mechanism of this disease?

A 32-year-old man hiking in Guatemala is bitten by a rattlesnake. He is Chikungunya fever Lyme disease [Y] Serum sickness Malaria Autoimmune hemolytic anemia
treated with equine antivenin. He recovers; however, 2 weeks afterward, he
develops rash, fever, and polyarthralgia. Laboratory evaluation reveals a
leukocyte count of 3200/μL, a hemoglobin concentration of 16.0 g/dL, a
platelet count of 95,000/μL, and a serum creatinine of 1.8 mg/dL. What is
the most likely diagnosis?

A 26-year-old Hispanic female presents to her primary care physician Positive SS-A Positive SS-B Positive anti-cyclic citrullinated peptide Positive rheumatoid factor [Y] Positive anti-Smith
complaining of 2 months of (CCP-IgG)
fatigue. She has persistent arthralgias of the distal
and proximal interphalangeal joints of both hands
and reports that when the weather turns cold her hands frequently change
colors. Her physical examination is normal and there is no tenderness or
defor mity of the joints of the hands. A complete blood count is normal and
her antinuclear antibody titer is elevated at 1:90. Which of the following
additional tests would confirm the diagnosis of lupus erythematosus?

A 20-year-old man steps into an elevator full of people with influenza who CD4+ cell [Y] Dendritic cell Macrophage Natural killer cell Neutrophil
are coughing and sneezing. The influenza viral particles that he inhales
attach to respiratory epithelium, and viral infection reduces MHC class I
molecules displayed on these epithelial cells. Which of the following
immune cells is most likely to rapidly destroy the virally infected cells?

In an experiment, antigen is used to induce an immediate (type I) [Y] CD4+ lymphocytes Dendritic cells Macrophages Natural killer cells Neutrophils
hypersensitivity response. Cytokines are secreted that are observed to
stimulate IgE production by B cells, promote mast cell growth, and recruit
and activate eosinophils in this response. Which of the following cells is
most likely to be the source of these cytokines?
A 33-year-old man is evaluated for recurrent sinusitis. He has had episodes Transfusion reaction Pneumonia Parasitic gastrointestinal infection Crohn’s disease [Y] Lymphoid malignancy
of bacterial sinusitis occurring at least four times a year for the past 3
years. Complete blood count is normal, as is a total hemolytic complement
panel (CH50). Serum IgG, IgM, and IgE levels are normal; however,
serum IgA levels are undetectable. Which of the following complications is
this patient at least risk for?

A 30-year-old sexually active woman undergoes a routine physical Antibiotic therapy [Y] Excision Ovarian removal Screening of family members Watchful waiting
examination. There are no abnormal findings. A Pap smear is obtained as
part of the pelvic examination. Cytologically, the cells obtained on the
smear from the cervix show severe epithelial dysplasia (high-grade
squamous intraepithelial lesion). Which of the following therapeutic
options is most appropriate for this woman?

A 40-year-old man has a positive stool guaiac test during a routine physical [Y] Adenoma Carcinoma Choristoma Hamartoma Hyperplasia
examination. A colonoscopy is performed and a 0.9-cm, circumscribed,
pedunculated mass on a short stalk is found in the upper rectum. Which of
the following terms best describes this lesion?

A 63-year-old female is being evaluated by her Bone scan CT chest with contrast Skin biopsy [Y] Upper and lower endoscopy Bone marrow aspirate
family physician. Over the past 3 months she has
developed a velvety hyperpigmentation of the skin
of the posterior neck and bilateral axilla. She has lost 15 lbs. since her last
visit four months ago and has a body mass index of 23 kg/m2. Routine
blood work is normal except for a hemoglobin level of 8.6 g/dL. Her
HbA1c is 5.4%, and a plain chest radiograph is normal. What is the next
step in the evaluation?

A 4-year-old boy is evaluated by his pediatrician for 1 week of intermittent medulloblastoma Polycystic kidney disease Clear cell sarcoma Rhabdoid tumor [Y] Wilms tumor
hematuria. On examination he appears well developed and well nourished,
except a right flank mass is palpable. A biopsy of themass is performed,
which reveals primitive looking small blue cells with glomeruloid and
tubuloid structures. What is the diagnosis?

The parents of a 17-year-old female bring her to the emergency room after Medulloblastoma Hepatic neuroblastoma Ewing sarcoma [Y] Cardiac rhabdomyoma Retinoblastoma
she had a brief seizure at home from which she recovered. A CT scan is
performed in the emergency room, identifying a cortical nodule in the left
cerebral hemisphere. A CT scan of the body is also performed, identifying
a 3.0 cm mass in the left kidney, which is subsequently biopsied and
diagnosed as an angiomyolipoma. Of the following, which other tumor is
this patient at increased risk for?
Question text Answer 1 Answer 2 Answer 3 Answer 4 Answer 5
A 73-year-old is rushed to the Emergency Room following a collapse at home. [Y]Coagulative Necrosis Macrophage and T cell infiltration Fatty Necrosis Liquefactive necrosis Contracted Scar
Despite resuscitation efforts, he dies. Five weeks ago, he had suffered a
myocardial infarction. What would be the most likely histological findings on
his heart?
A 24-year old man is involved in a motor vehicle collision Septic emboli [Y]Fat embolism Air embolus Athero emboli
and sustains a fracture of his left femur. While in the hospital, 2 days after the
accident he is noted to be dyspneic and disoriented, and he develops petechiae
on his trunk and axilla. What is the most likely diagnosis?
Which of the following ultrastructural features of cell injury are considered Swelling of endoplasmic reticulum Cytoplasmic vacuoles [Y]Pyknotic nuclei Cell swelling
irreversible?
The earliest event in acute inflammation is: Endothelial contraction Fibroblast prolifereation Macrophage migration [Y]Vasodilation
All of the following are true about Reversible Cell Injury except: swelling of cell organelles and dissociation of ribosomes from decreased energy production by [Y]nuclear degeneration
entire cell endoplasmic reticulum mitochondria
Which of the following is frequently seen with autoimmune disorders? [Y]Fibrinoid necrosis Liquefactive necrosis Gangrenous necrosis Coagulative necrosis Fatty Necrosis
The mechanism of action of Prostacyclin (PGI2) is best described by which of causes vasoconstriction stimulates platelet aggregation [Y]causes vasodilation and main chemotactic factor for
the following statements? inhibits platelet aggregation neutrophils
Which of the following complement components are involved in opsonization? C5a C5-9 C1 [Y]C3b
Which one of the following relationships is NOT correct? fibrinoid necrosis →autoimmune coagulative necrosis →ischemia fat necrosis → pancreatitis and [Y]liquefactive necrosis -
disease trauma to the breast tuberculosis
Which statement concerning keloid is true? In contrast to normal or [Y]Keloids result from abnormal Lower incidences are seen in darker Keloids tend to regress over several
hypertrophic scars, keloidal tissue wound healing in response to skin skinned individuals of African, months
do not extends beyond the initial trauma or inflammation Asian, and Hispanic descent.
site of trauma.
Which one of the following relationships is NOT correct? TNF Stimulates expression of IL-1 action is Similar to TNF; sustained production of TNF [Y]TNF is an important mediator
endothelial adhesion molecules and greater role in fever contributes to cachexia of platelet
secretion of aggregation and vasoconstriction
other cytokines
Which of the following substances are main chemotactic factors for neutrophils? [Y]IL-8, Leukotriene B4, C5a Thromboxane A2 Prostacyclin TNF and IL1
Cellular changes seen in apoptosis include all of the following, EXCEPT Cell shrinkage Formation of cytoplasmic blebs and Chromatin condensation [Y]Cell swelling
apoptotic bodies
Coagulation necrosis is commonly seen in Brain [Y]Heart Pancreas autoimmune diseases
Which of the following is best stained by Congo red stain? Glycogen Fat Mucin [Y]Amyloid
A 50-year-old policeman has been seen by his family physician for a 5-year [Y]Metaplasia Atrophy Hyperplasia Hyperthrophy Atrophy
history of “heartburn.” He has been intermittently taking ranitidine, a histamine-
2 blocking agent, with some relief. An upper endoscopic examination that was
performed recently revealed some reddish discoloration and friability of the
lower esophageal region. A biopsy of the lower esophagus was performed, and
the microscopic examination revealed columnar cells containing goblet cells.
What is the most likely mechanism of esophageal changes ?
What is the most common cause and site of origin for pulmonary emboli? [Y]Deep venous thrombosis Long bone trauma Mesenteric vein obstruction Right ventricle stasis
A 2-year-old girl of Ashkenazi Jewish descent is being evaluated for mental Fabry disease Gaucher disease [Y]Niemann-Pick disease Tay-Sachs disease
retardation, seizures, and ataxia. Physical examination finds enlargement of the
liver, spleen, and several lymph nodes. A bone marrow biopsy reveals
aggregates of lipid-laden macrophages (foam cells).
These cells contain excessive amounts of sphingomyelin and cholesterol.
Further workup finds a deficiency of the enzyme sphingomyelinase. Which of
the following is the most likely diagnosis?
Which is NOT evidence of irreversible cell injury? [Y]acute cell swelling nuclear pyknosis rupture of the lysosomes karyolysis
You're most likely to see caseous necrosis in myocardium [Y]tuberculous lung gangrenous diabetic foot Brain
You're most likely to see liquefactive necrosis in myocardium tuberculous lung gangrenous diabetic foot [Y]Brain
A 12-year old boy with a 1-day history of sore throat [Y]The predominant cell type is The duration may be for months Plasma cells are frequently present Lymphocytes are present at the start
was seen by his GP. On physical examination, the most the neutrophil of the process
prominent finding was a purulent pharyngeal exudate. polymorph
He most likely has an acute inflammation. In acute
inflammation which of the following is true?
The histopathology report for a tissue biopsy shows that it is acute Vasoconstriction [Y]Increased blood flow Nitrate release Diuresis
inflammation. Which of the following features is seen in a tissue undergoing
acute inflammation?
Which of the following statements about necrosis is correct? Maintains plasma membrane Does not result in an inflammatory Is an energy-dependent process [Y]May be stimulated by oxygen
integrity response free radicals
Apototic cells usually exhibit distinctive morphological features. Which of the Is only a pathological process [Y] Results in cell shrinkage and DNA is cleaved by proteases Results in an inflammatory
following is true of pure apoptosis? fragmentation response
The histopathology report for a tissue biopsy shows that it is fat necrosis. Which Brain aneurysm Muscle injury Myocardial infarction [Y]Trauma to breast
of the following situation is most likely to exhibit fat necrosis?
After a 12-hour plane flight, a 53-year-old female flight attendent noticed a Antiphospholipid antibody Malignancy sometimes decrease the [Y]Is predisposed by blood stasis The most common site of venous
heavy ache over the right calf, with associated swelling and tenderness. Which decreases risk of arterial risk of venous thrombosis is the portal vein
of the following is true of thrombosis? and venous thrombosis thrombosis
Which of the following is true of amyloidosis? Amyloid does not take up Congo Has no systemic effects The most common amyloid in [Y]Amyloid refers to the
red stain reactive systemic abnormal fibrous, extracellular,
amyloidosis is the AL type proteinaceous deposits found in
organs and tissues.
White infarcts occur in: [Y]kidney the small intestine lung sigmoid colon
Which of the followings is the mechanism of edema in patients with congestive Decreased plasma oncotic pressure Increase vascular permeability [Y]C. Increase hydrostatic Lymphatic obstruction.
heart failure? pressure.
“Nutmeg Liver “is seen in: Hemorrhage due to liver trauma Thrombosis of the portal veins. Liver necrosis [Y]Chronic passive congestion
Petechiae is best defined as: Subcutaneous hemorrhage Subcutaneous edema in association Hemorrhage into the thoracic cavity [Y]Skin hemorrhages appearing
measuring 1-2 cm. with heart failure as minute spots measuring 1-2
mm
Which of the following is FALSE regarding anti-phospholipid syndrome? Can be seen in association with Patients at risk of repeated Patients are at risk of recurrent Can have false positive serological [Y]Patients at risk of abnormal
systemic lupus erythematosus. miscarriages. venous thrombosis and pulmonary test for syphilis bleeding.
emboli
Disorders that predispose to thrombosis include all of the following EXCEPT: Pancreatic carcinoma Pregnancy. [Y] Vitamin K deficiency Sickle cell anemia Oral contraceptive pills
A pathologist examines a section of bronchial tissue obtained during a Anaplasia Dysplasia Hyperplasia [Y]Metaplasia Neoplasia
transbronchial biopsy performed on an individual who smokes. She notes that
the bronchial lining consists of sev eral layers of well-differentiated, organized
squamous epithelia contained above the base ment membrane. Which of the
following terms best describes the pathologist’s finding?

A pathologist examines a renal biopsy from a 45-year-old man with nephrotic glycogen hemosiderin. [Y]amyloid copper.
syndrome and requests a Congo red stain to confirm the nature of an amorphous
acidophilic extracellular hyaline substance localized within the mesangial
matrix of the glomeruli. A positive test confirms the presence of

Which of the following statements about apoptosis is correct? pathologic process in which cellular Characterized by cellular swelling,
[Y]is a type of cell death that is
membranes fatty induced by a
are destroyed, enzymes and other change, plasma membrane blebbing tightly regulated suicide program
constituents leak out, and and loss of microvilli, in which cells destined
local inflammation is induced to mitochondrial swelling, dilation of
to die activate intrinsic enzymes
clear the damaged cells the ER, and eosinophilia that degrade the cells’
genomic DNA and nuclear and
cytoplasmic proteins
Which of the following is false about Lipofuscin? Lipofuscin is an insoluble pigment, Lipofuscin is composed Lipofuscin is not injurious to the [Y]Lipofuscin is a hemoglobin-
also known as of polymers of lipids and cell or its derived, golden yellowto-
lipochrome or wear-and-tear phospholipids in complex functions. Its importance lies in its brown, granular, or crystalline
pigment. with protein being a telltale sign pigment
of free radical injury and lipid
peroxidation
Which of the following is true of a dystrophic calcification? [Y]When the deposition occurs The deposition of calcium salts in dystrophic calcification is almost Dystrophic calcification is
locally otherwise normal tissues is an always encountered in normal tissue
in dying tissues, it is known as example of dystrophic calcification results from hypercalcemia
dystrophic calcification secondary to some disturbance
in calcium metabolism.
Which one of the following relationships is NOT correct? keratin filaments are characteristic desmin filaments are characteristic vimentin filaments are [Y]neurofibrillary
of epithelial cells of muscle cells characteristic of connective tissue tangle found in the in liver cells is
cells characteristic of alcoholic
liver disease

A 15-year-old girl incurs a cut to the sole of her foot after stepping on a piece of Glycoprotein IIb/IIIa Thrombomodulin [Y]Prostacyclin Thromboxane
broken glass. On examination, a superficial 0.5-cm laceration ceases to bleed
within 5 minutes after application of local pressure. Which of the following
substances is released by endothelium and is most likely to counteract platelet
aggregation near this site of injury?
Which of the following statements about Integrins are correct? The initial rolling interactions are These [Y]TNF and IL-1 induce
mediated by are low-affinity interactions with a endothelial expression of ligands
Intergrins fast off-rate, so they for integrins
are easily disrupted by the flowing
blood.
Which of the following is not an action of Leukotrienes? intense vasoconstriction bronchospasm increased permeability [Y]suppression of inflammation
of venules by inhibiting neutrophil
chemotaxis
What is the name for the protein-rich extravascular fluid produced in transudate [Y]Exudate Pus blood
inflammation?
Which of the below is true regarding Serous Inflammation? Serous exudate is characteristic of [Y]Serous inflammation is Serous inflammation is
inflammation marked by the exudation of characterized by the production
in the lining of body cavities, such cellpoor of pus
as the meninges, fluid into spaces created by cell
pericardium and pleura injury or into body
cavities lined by the peritoneum,
pleura, or pericardium
Which is not true regarding Bernard soulier syndrome? Bernard soulier syndrome is a deficiency of GPIb receptor for [Y]inherited deficiency of GpIIb- affects males and females equally
Autosomal recessive platelet vWF causes impaired platelet IIIa results in a bleeding
disorder adhesion disorder called Bernard soulier
syndrome
A 65-year-old man develops worsening congestive heart failure 2 weeks after an Eosinophils Epithelioid cells [Y]Macrophages Neutrophils Plasma cells
acute myocardial infarction. An echocardiogram shows a markedly decreased
ejection fraction. Now, capillaries, fibroblasts, collagen, and inflammatory cells
have largely replaced the infarcted myocardium. Which of the following
inflammatory cell types in this lesion plays the most important role in the
healing process?
A 32-year-old man is involved in a vehicular accident and sustains fractures of [Y]Fat embolism Right hemothorax Amnionic Embolism Athero emboli
the right femur and tibia and the left humerus. The fractures are stabilized
surgically. He is in stable condition for 2 days, but then suddenly becomes
severely dyspneic. Which of the following complications from his injuries is the
most likely cause of his sudden respiratory difficulty?

On day 28 of her menstrual cycle, a 23-year-old woman experiences onset of [Y]Apoptosis Atrophy Caseous necrosis Liquefactive necrosis
menstrual bleeding that lasts for 6 days. She has had regular cycles since
menarche. Which of the following processes most likely occurs in her
endometrial cells to initiate the onset of menstrual bleeding?

In an experiment, thrombus formation is studied in areas of vascular damage. Platelet factor 4 Prothrombin [Y]Thrombomodulin Tumor necrosis factor (TNF)
The propagation of a thrombus in an area of vascular injury to adjacent normal 13
arteries is prevented. Which of the following substances diminishes thrombus
propagation by activating protein C?
A neonate born to 41-year-old woman in her 39th w eek of gestation has [Y]Meiotic non-disjunction Expansion of trinucleotide Deletion of chromosomal part Robertsonian translocation
a flattened face and epicanthal folds, oblique palpebral fissures and a repeats
single palmar crease. The child's echocardiography reveals an
endocardial cushion defect. Which of the following most likely occurred
prior to conception?
A 15-year-old patient is referred to your office by a teacher who is Short stature, broad chest, Macroorchidism, large jaw and [Y]Tall stature, gynecomastia, Short stature, hypotonia, obesity
concerned about the patient's learning abilities. After evaluating the amenorrhea ears infertility
patient, you diagnose mild mental retardation. Cytogenetic studies of the
patient's buccal mucosal cells reveal a 47 XXY karyotype. Further
evaluation is likely to reveal:
A 14-year-old male is diagnosed with mild mental retardation. Short stature, broad chest, Tall stature, gynecomastia, Short broad hands, transverse [Y]Macroorchidism, large jaw
Cytogenetic studies show a small gap nearthe amenorrhea infertility palmar crease and ears
tip of the long arm of the X chromosome. Physical examination of this
patient is likely to show:
An infant born prematurely to a 42-year-old Caucasian female is small Trisomy 21 Trisomy 13 [Y]Trisomy 18 47,XXY
for gestational age. Physical
examination reveals microcephaly, low-set ears, prominent occiput and
small mandible. The infant's fists are
clenched and the fingers overlap. A bilateral foot deformity is observed.
Which of the following is the most
likely karyotype abnormality in this infant?
A stillborn fetus delivered at the 23rd week of gestation is found to have Trisomy 21 Trisomy 18 Trisomy 13 47,XXY [Y]45,XO
an edematous neck and broad chest.
Autopsy reveals aortal coarctation, a bicuspid aortic valve, and kidneys
that are fused at the midline. Which of
the following is the most likely karyotype abnormality in this fetus?
A 23-year-old Caucasian male with mild mental retardation has large [Y]Fragile X syndron1e Marfan syndrome 47,XYY karyotype Klinefelter syndrome
ears, a long face, a prominent mandible,
and large testes. His hand joints are hyperextensible on physical
examination. Which of the following is the
most likely diagnosis in this patient?
An infant born prematurely to a 38-year-old Caucasian female is small Trisomy 18 Trisomy 21 [Y]Trisomy 13 Marfan syndrome
for gestational age. Physical
examination reveals a bilateral cleft lip, microcephaJy, and
microphthalmos. Viscera protrude from an
umbilical opening in the child's abdominal wall. Which of the following
karyotypes is most likely in this case?
A 40-year-old pregnant woman at 12 weeks gestation comes to the From the mother only [Y]From the father only Half from the father and half Mostly from the mother
emergency department due to vaginal from the mother
bleeding associated with lower pelvic pain. On examination, there is
bright red blood in the vagina and the
uterus appears larger than expected given the patient's gestational age.
There are no fetal heart tones audible
on Doppler ultrasound. The patient undergoes dilation and curettage in
the operating room, and a friable
mass of tissue consisting of many thin-walled cysts is evacuated from her
uterus. Karyotype examination of
the tissue shows it to have a 46 XX genotype. Further analysis would
most likely show nuclear chromosomes
derived from which of the following sources?
An infant born to 38-year-old primigravida develops carpopedal spasms Second branchial cleft Fourth branchial arch [Y]Third pharyngeal pouch Third branchial arch
soon after birth. On diagnostic
workup, the infant is found to have an absence of the thymic shadow on
X-ray and narrowing of the aortic arch
on angiography. Which of the following embryonic structure derivatives
failed to develop in this patient?
During kidney transplantation in a patient with end-stage polycystic Immediate hypersensitivity [Y]Antibody-mediated Immune complex-mediated Cell-mediated hypersensitivity
kidney disease, the surgeon notices that the graft becomes cyanotic and hypersensitivity damage
mottled soon after he connects graft vessels with recipient vessels. The
blood flow to the graft eventually ceases, and no urine is produced.
Which of the following best explains the
findings observed by the surgeon?
Soon after birth, a term infant develops tetany with marked [Y]DiGeorge anomaly Severe combined Common variable Bruton disease Hyper IgM syndrome
hypocalcemia. This is treated, but at one month of age, a systolic heart immunodeficiency immunodeficiency
murmur is heard on auscultation of the chest. Later in infancy, it is noted
that the baby has been almost constantly ill with one infection after
another, including respiratory syncytial virus, Candida, and Pneumocystis
carinii (jirovecii) pneumonia diagnosed. Which of the following primary
immunodeficiency disorders is the child most likely to have?
An example of type III immune complex disease is: Contact dermatitis Allergies Graft rejection [Y]Serum sickness Atopy
Which of the following statements is true? Adenocarcinoma is a malignant Carcinoma is a neoplasm Sarcoma is a malignant tumor of [Y]Desmoplasia is a tumor-
tumor of mesenchymal origin derived from all three germ cell
epithelial origin induced proliferation of non-
layers neoplastic fibrous connective
tissue
Which of the following term best describes Choristoma ? Are most often named by the [Y]This is a small non- This benign neoplasm most
tissue of origin neoplastic area of normal often arises from surface
tissue misplaced within epithelium, This is a non-neoplastic,
another disorganized, tumorlike
organ, such as pancreatic overgrowth of cell types
tissue within the wall of the regularly found within an
stomach affected organ
Which of the following statements regarding smoking is CORRECT? It is a stimulant that increases [Y]It inhibits the ciliary It causes lung cancer, but not It contains particulate matter and It is recommended for pregnant
the ciliary actions of the actions of the respiratory bladder cancer. irritants, the most dangerous women
respiratory epithelial lining and contributes to being carbon
lining obstructive pulmonary disease. dioxide.
Because of the increased risk of anaphylaxis, plasma-containing blood Complement deficiency Hereditary angioedema Systemic lupus erythematosus [Y] IgA deficiency Combined variable immune
products should deficiency
NOT be given to patients who have which of the following conditions?
A 50-year-old man presents to the hospital after a briefბepisode of chest Choristoma [Y]Hamartoma Adenoma Sarcoma
wall pain. An ECG and cardiac enzyme tests reveal no abnormalities. A
routine chest radiograph reveals only a single 2-cm coin lesion in the
right lower lobe of the lung with a “popcorn” pattern of calcifications.
The man is scheduled for surgery, and the lesion is removed. The
pathology report describes the mass as a disorganized nodule of cartilage,
with no cellular features of malignancy. What is the general term for this
lesion?
A 3-year-old child is being evaluated by his physician during an office severe combined chronic granulomatous disease hyper-IgM syndrome [Y]Chédiak-Higashi syndrome
visit. Over the past 2 years, the child has had several bouts of ostitis immunodeficiency syndrome
media and one severe case of pneumonia, which required hospitalization.
At the time of the examination, the physician also notes that the
child’s skin is generally very white. The child’s mother mentions that one
of the child’s grandmother’s sisters died at a young age from recurrent
infections. After further laboratory testing, the physician makes a
diagnosis of
A 9-month-old girl with a history of recurrent pulmonary infections is Bruton X-linked DiGeorge syndrome Isolated IgA defi ciency [Y]Severe combined
found to have a congenital defi ciency of adenosine deaminase, which is agammaglobulinemia immunodefi ciency
associated with a virtual absence of
lymphocytes in her peripheral lymphoid organs. What is the appropriate
diagnosis?
A 20-year-old woman has an ovarian tumor removed. The surgical Adenoma Chondroma Hamartoma Teratocarcinoma [Y]Teratoma
specimen is 10 cm in diameter and cystic. The cystic
cavity is found to contain black hair and sebaceous material. Histologic
examination of the cyst wall reveals a variety of benign differentiated
tissues, including skin, cartilage, brain, and mucinous glandular
epithelium. What is the diagnosis?
A 45-year-old woman presents with abdominal pain and Leiomyosarcoma Myxoma [Y]Leiomyoma Rhabdomyoma
vaginal bleeding. A hysterectomy is performed and shows a
benign tumor of the uterus derived from a smooth muscle cell.
What is the appropriate diagnosis?
A 4-year-old girl presents for a preschool physical examination. The Cigarette smoking [Y]Alcohol abuse Inadequate nutrition Poorly controlled diabetes
child has a small head circumference, thin upper lip, mellitus
and low-bridge nose. She shows evidence of mild mental retardation. Her
parents state that she is often “emotional.” Which of the following
maternal causes of birth defects most
likely accounts for these clinicopathologic fi ndings?
All of the following statements about lead poisoning are correct except: The developing brain is highly The major source of exposure to [Y]Lead containing water used Excess lead causes CNS defects Lead interferes with the
sensitive to methyl mercury, mercury is contaminated fish to irrigate in children and peripheral remodeling of cartilage
which accumulates in the CNS rice fields in Japan caused a neuropathy in adults and causes anemia by interfering
disease in postmenopausal with hemoglobin synthesis.
women known as itai-itai
Marasmus is not characterized by [Y]swelling of limbs replacing tissue proteins impaired growth protein deficiency
An 8-month-old male infant is admitted to the hospital because of a Adenosine deaminase [Y]Bruton tyrosine kinase Wiskott-Aldrich syndrome IL-2 receptor
bacterial respiratory infection. The infant has no previous history of viral protein
or fungal infections. The infant responds to appropriate antibiotic
therapy,
but is readmitted several weeks later because of severe otitis media. Over
the next several months, the infant is admitted to the hospital multiple
times for recurrent bacterial infections. Initial workup at this time finds
very low serum levels of IgG along with an absence of CD19+ cells.
Further evaluation finds a lack of B cells past the pre-B stage. The
disorder this
boy most likely has is associated with a defect involving which one of the
following?
Which one of the listed disorders is a type of childhood malnutrition that Beriberi [Y]Kwashiorkor Marasmus Osteomalacia Pellagra
is caused by insufficient dietary protein and most commonly occurs when
a child is weaned from breast milk and placed on a diet that is high in
starches and carbohydrates but deficient in protein?
A 6-month-old boy is being evaluated for failure to thrive, along with Heparan sulfate Glucocerebroside Heparan sulfate [Y]Sphingomyelin
persistent vomiting, seizures, and a low-grade fever. Physical
examination
finds a protuberant abdomen due to enlargement of both the liver and
spleen, along with a “cherry-red spot” on his retina and diffuse enlarged
lymph nodes. A bone marrow biopsy reveals an abnormal diffuse
proliferation of foamy macrophages filled with lipid (“foam cells”).
Electron microscopy reveals cytoplasmic bodies that resemble concentric
lamellated
myelin figures within the foamy macrophages. Rare parallel palisading
lamella (“zebra bodies”) are also seen in the cytoplasm of these cells with
electron microscopy. Which of the following substances is most likely to
be
found at abnormally high levels within these foamy macrophages?
Paternal uniparental disomy involving chromosome 15 is a possible Smith-Magenis syndrome Prader-Willi syndrome [Y]Angelman syndrome Beckwith-Wiedemann syndrome
mechanism for the development of which one of the listed disorders?
A 34-year-old Jewish woman presents with weakness and a feeling of
“fullness” in the left side of her abdomen. A medical history finds that
she
is Jewish and her family is from the eastern portion of Europe (Ashkenazi
Jews). Physical examination reveals a markedly enlarged spleen, but no
neurologic abnormalities are found. Examination of her retina is within
normal limits. Laboratory examination reveals decreased numbers of red
blood cells, white blood cells, and platelets in the peripheral blood.
Elevated
levels of angiotensin converting enzyme (ACE) and acid phosphatase are
found in the peripheral blood. Histologic sections from a bone marrow
biopsy reveal numerous macrophages that have a delicate fibrillar
(“crumpled
tissue paper”) cytoplasm due to the accumulation of PAS-positive
material. Sandhoff disease
What is the best diagnosis for this individual? Neiman-Pick disease [Y]Gaucher disease Tay-Sachs disease
A 57-year-old female presents to general gynecology clinic for evaluation Alpha fetoprotein
of a pelvic mass. The mass was detected on a routine visit to her primary
care doctor during abdominal palpation. In the office, she receives a
transvaginal ultrasound, which reveals a mass measuring 11 cm in
diameter. In the evaluation of this mass, elevation of which tumor marker
would be suggestive of an ovarian cancer? S-100 [Y]CA-125 Beta-hCG
Slow respiration, slow pulse, and consrtiction of pupil occures due to
drug addiction of Cocaine and heroin [Y]Morphine and opium Alcohol and thalidomide Nicotine and caffeine
Which of the following is a hallucinogen? [Y]LSD Heroin Morphine Codein
Wernicke's encephalopathy is atributed to Severe riboflavin deficiency Excessive blood alcohol content [Y]thamine deficiency Excessive GGT levels
Which of the following features is included in the determination of tumor
stage in this disease? Degree of nuclear pleomorphism Loss of polarity [Y]Lymph node involvement Number of mitotic figures Surface ulceration
Children 6 to 10 years old in the same community are observed by the Cadmium Copper Iron [Y]Lead
local physician to be doing poorly in school, which has been attributed to
behavioral problems. Their par ents state that these children have poor
appetites, complain of
nausea, and have frequent headaches. On physical examina tion, they
have decreased sensation to touch over the lower extremities. They
exhibit loss of fine motor control of move ment and have a slightly ataxic
gait. A representative CBC shows hemoglobin of 11.8 g/dL, hematocrit
of 35.2%, MCV of 82 μm3, platelet count of 282,300/mm3, and WBC
count of 4745/mm3. Examination of the peripheral blood smear shows
basophilic stippling of the RBCs. Excessive chronic ingestion
of which of the following substances is most likely to explain these
findings?
An 8-year-old girl with numerous hypopigmented, ulcer ated, and crusted Hereditary albinism Li-Fraumeni syndrome Neurofi bromatosis, type I [Y]Xeroderma pigmentosum
patches on her face and forearms develops an indurated, crater-like, skin
nodule on the back of her left hand. Biopsy of this skin nodule discloses
a squamous cell car cinoma. Molecular biology studies reveal that this
patient has germline mutations in the gene encoding a nucleotide exci
sion repair enzyme. What is the appropriate diagnosis?
A neonate was noted to have mild growth retardation and facial [Y]Alcohol Cocaine Heroin Marijuana
dysmorphology. The mother was a known abuser of several substances.
This infant’s problem most likely resulted from maternal intake of which
of the following?
All of the following are the examples of the route through which HIV can Unprotected sexual contact with From infected mother to the [Y]From the mosquito bite Exposure to contaminated blood
be transmitted from one person to another, EXCEPT? an infected person fetus carrying the virus and blood products
A 34-year-old male experiences low-grade fever and decreased urine Preformed antibodies against Host B cell sensitization against [Y]Host T cell sensitization Graft B cell sensitization against
output one week after kidney graft ABO antigens graft MHC antigens against graft MHC antigens host MHC antigens
transplantation. Graft biopsy demonstrates dense interstitial infiltration by
mononuclear cells. Which of the
following is the most likely cause of this patient's current condition?
A 26-year-old woman presents to the dermatology clinic for fatigue, [Y]systemic lupus Sjogren's syndrome Sclerodermia Marfans Syndrome
weakness, and fevers for the past month. She reports significant weight erythematosus
loss despite eating a normal diet. She reports that she sunburns very
easily and has a facial rash that is hard to cover with makeup. On
physical exam, she has a butterfly rash with nasolabial sparing on her
face, several discoid lesions on her fingers, and a erythematous rash on
her chest in a V-neck distribution. On laboratory exam, she has a highly
positive antinuclear antibody and positive anti-double-stranded DNA
antibody.What is the most likely diagnosis?
Pathology Final

1) A 1-year-old boy presents with a delay in motor development.


Progressive muscle weakness and blindness ensue, and the patient dies
within a year. The brain at autopsy shows swollen neurons that contain
numerous lysosomes filled with lipid. Which of the following is most
likely diagnosis?
Tay-Sachs disease

2) An infant born to a 36-year-old Caucasian female demonstrates


some dysmorphic facial features and a holosystolic murmur at the left
sternal border. Karyotype anal lysis is consistent with trisomy 21. Which
of the following additional findings would be most expected in this
infant?
Single palmar crease

3) A 22 year-old intravenous drug user has regularly sought human


immunodeficiency virus (HIV) testing, always with negative results. He
admits to carelessly sharing needles on multiple occasions with
individuals later found to be HIV-positive. He has heard that there is an
inherited genetic basis for some people to be relatively “immune” to
HIV infection. The genetic change that he is referring to is a mutation in
a gene coding for which of the following proteins?
CCR5
4) which of the following tissues is the most susceptible to radiation
injury?
Lymph and haemopoietic

5) A 25-year-old man with a history of mitral valve prolapse (MVP)


presents with 1day of sharp, tearing chest pain at the center of his
chest that now radiates toward his neck. Physical exam reveals low
blood pressure, moderate sternal depression, and a tall, thin man with
long arms (PIC 2). What is likely to be abnormal in this patient?
Fibrillin 1

6) A 46-year-old Caucasian male presents to your office with a slowly


growing neck mass. The mass is stony hard on palpation and seems to
be fixed to the adjacent tissues. After initial evaluation, combination
chemotherapy is prescribed to the patient. Several weeks later the
mass significantly decreases in size, and biopsy demonstrates many
shrunken eosinophilic cells within the tumor. Which of the following
substances | released from mitochondria most likely triggered the
observed cellular changes?
Cytochrome c
7) which of the following is not example of trinucleotide repeat
disorder?
Wilson disease

8) 2-year-old boy with Bruton’s agammaglobulinemia has recurrent


pneumonia caused by streptococcus pneumoniae. Which of the
following defects is the most likely cause of increased susceptibility
infections?
Leukocyte adhesion molecule detect

9) Hypoalbuminemia is usually seen in patients of chronic liver disease.


There are several factors which contribute to it. Which factor is the
biggest contributor in the causation of collection of fluid in peritoneum
(ascites)?
Low plasma cortical osmotic pressure

10) Dysplasia
is characterized by cellular pleomorphism.

11) regarding apoptosis, which of the following is FALSE?


Associated with inflammation

12) A 23-year-old woman comes to the urgent care center complaining


of a rash around her neck. For Valentine's Day, her fiancé gave her a
beautiful gold necklace. Since wearing this necklace, she has developed
a skin rash notable for pale-red, raised, itchy papules and vesicles
around her neck. You suspect that she has developed contact
dermatitis from her necklace. Which of the following is the most likely
mechanism of her rash development?
Type 4 delayed cell-mediated hypersensitivity
13) this child has moderately severe mental retardation. Chromosome
analysis shows 46 chromosomes. Which of the following types of
genetic mutation is most likely responsible for his condition?
Balanced translocation

14) which of following protein is pro-apoptotic?


BAK

15) A 63-year-old woman discovers a stump in her right breast.


Mammography confirms the presence of a suspicious “lump,” and a
needle core biopsy is performed to determine whether the mass is
malignant. The pathology report confirms that the mass is indeed
cancerous and that the tissue demonstrates amplification of the Her-
2/neu oncogene. The gene product of Her-2/neu is what kind of
protein?
receptor tyrosine kinase

16) A 22-year-old woman with an uncomplicated pregnancy develops


sudden dyspnea with cyanosis and hypotension intrapartum during
routine vaginal delivery of a term infant. She has a generalized seizure
and becomes comatose. Her condition does not improve over the next
2 days. Which of the following findings is most likely to be present in
her peripheral pulmonary arteries?
Amniotic fluid

17) an example of trinucleotide-repeat mutation associated genetic


disorder is
Fragile X syndrome

18) A 60-year-old female is suffering from renal failure and is on


hemodialysis since the last years. She developed carpal tunnel
syndrome. Which of the following will be associated?
Beta 2 microglobulin

19) what is the cytokine that stimulates collagen synthesis?


TGF-B

20) which of the following is a source od histamine during


inflammation?
Platelets
21) Cellular events in acute inflammation include all of the following
except:
Margination of macrophages to vessel wall
22) with regard to the role of complement in the acute inflammatory
response, which is false
C3a activate the lipoxygenase pathway in WBC

23) 26-year old rock climber fractures his left femur after falling from a
height of20 meters. He was hospitalized and over the next several days
develops progressive respiratory problems. time of autopsy, oil red-O
positive material is seen in the small blood vessels of the lungs and
brain. Which of the following is the most likely diagnosis?
Fat emboli

24) A 60-year-old man with unstable angina (a form of acute coronary


syndrome) is treated with an intravenously admins tered glycoprotein
lib-illa inhibitor. The mechanism of action of this agent is:
Inhibit platelet aggregation

25) metaplasia is
Associated with vitamin A deficiency which may involve
respiratory epithelium

26)
GENERAL PATHOLOGY
GENERAL PATHOLOGY
1.Which
1. ofthe
Which of following is
the following is leastlikely
leastlikely totofacilitate
facilitate
chemotaxis:
chemotaxis:

A. LTB4
A. LTB4
B. IL8
B. IL8
C.C5a
C. C5a
D. TGF
D. TGF
TGF
TGF
A 56
A 56 year
year old hypertensive
old hypertensive malemale presented with
presented with
leftsided
left hemiparesis
sided hemiparesis on waking
on wakingup.A
up. A CT CT scan
scan
was
was done
done which revealed
which revealed an infarct
an infarct in
in the
the
distribution
distribution of the
of theMCA. Which of
MCA. Which of the
thefollowing
following
is the
is theexpected gross pathological
expected gross pathological findinginin the
finding the
brain?
brain?

A. Coagulative
Coagulative necrosis
necrosis
W D>

B. Caseous
Caseous necrosis
necrosis
moO

C. Liquefactive
Liquefactive necrosis
necrosis
D. Fibrinoid
Fibrinoid necrosis
necrosis
E. Gangrenous
Gangrenous necrosis
necrosis
Liquefactive necrosis
Liquefactive necrosis
A 50
A 50 year
year old
old male was
male was recently diagnosed
recently diagnosed with Cancer.
with Cancer.
Onphysical examination,
Onphysical examination, he appears
he appears
emaciated. Which
emaciated. Which of the
of the’ followingisis also
following also called
called
cachectinand
cachectinand is responsible
is responsible forforthe weight
the weight loss
loss
and
and wasting noted
wasting notedinin the
the patient?
patient?

A.. IL1
IL1
moO >D

B. IL2
IL2
C. PANS
PAF
D.. TGF-beta
TGF-beta
E. TNF
TNF
TNF
TNF
What is
What is the
the interluekin
interluekin that
that is
is vital
vital for
for neutrophil
neutrophil chemotaxis?
chemotaxis?

A. IL-1
A. IL-1
B. IL-2
B. IL-2
C. IL-6
C. IL-6
D. IL-8
D. IL-8
E.IL-10
E.IL-10
IL-3
IL-8
This
This is considered
is considered an an irreversible
irreversible histologic
histologic
manifestation of
manifestation of cellular
cellular injury
injury

A.. Cellular swelling


Cellular swelling
moOWO D>

B. Nuclear chromatin
Nuclear chromatin clumping
clumping
C. Nuclear pyknosis
Nuclear pyknosis
D.. Ribosomal
Ribosomal detachment
detachment
E. Membrane blebbing
Membrane blebbing
Nuclear pyknosis
Nuclear pyknosis
A patient
A patient diagnosed
diagnosed with drug-
with drug-
induced lupus
induced lupus
wouldmost likely
wouldmost likely be
be positive
positive for
forwhich
which

autoantibody?
autoantibody?

A. Anti-SS-A
A. Anti-SS-A
B.
B. B. Anti-centromere
B. Anti-centromere
C.
C. C. Anti-histone
C. Anti-histone
D.
D. D. Anti-dsDNA
D. Anti-dsDNA
E.
E. E. Anti-mitochondrial
E. Anti-mitochondrial
The following
The following presents
presents with
with
granulomatous inflammation
granulomatous inflammation
EXCEPT:
EXCEPT:

A.Histoplasmosis
A.Histoplasmosis
B.B. Tuberculosis
B.B. Tuberculosis
C.C. Sarcoidosis
C.C. Sarcoidosis
D.D. Schistomiasis
D.D. Schistomiasis
E.E. Molluscum
E.E. Molluscum contagiosum
contagiosum
Molluscum contagiosum
Molluscum contagiosum
What is
What is the
the cytokine
cytokine that
that stimulates
stimulates collagen
collagen
Synthesis
Synthesis

A. TNF
A. TNF
B. IL-1
B. IL-1
C. VEGF
C. VEGF
D. TGF-B
D. TGF-B
E. PDGF
E. PDGF
TGF-B
TGF-B stimulates collagen
stimulates collagen synthesis,
synthesis,

IL-1 stimulates
IL-1 stimulates fever
fever as
as well
well as
as TNF,
TNF,

VEGF causes
VEGF causes angiogenesis,
angiogenesis,

PDGFcauses collagenase
PDGFcauses collagenase secretion
secretion
The classic
The classic anaphylactic
anaphylactic reactionis
reactionis due
due to:
to:

A. Th1
A. Th1
B. Th2
B. Th2
C. NK
C. NK cell
cell
D. Macrophage
D. Macrophage
E. Noneof
E. Noneof thethe above
above
ALeTB en CxpoOsure
nines

b class ||

Th2 cell

IL-4
IL-13
FceRI receptor Y
es

§ E a

IL-4 ——> i479


Mast cell a B cell
; Ww v WU
degranulation “ww
Histamine, CysLTs, PGs, Cytokines

Airway smooth J | Epithelial


muscle cells Blood § J 4 J 4 teells
vessel | S| Se |) Se | Sege
yy ety Pee gee
Among
Among the following
the following adaptations
adaptations of
of the body
the body to
to
injury,
injury, which
which of
of the following
the followingisis aa_ fertile
fertile ground
ground
for neoplasia?
for neoplasia?

A.
A. Hyperplasia
Hyperplasia
B.
B. Atrophy
Atrophy
C.
C. Hypertrophy
Hypertrophy
D.
D. Metaplasia
Metaplasia
E.
E. None
None of
of the above
the above
Metaplasia
Metaplasia
This can
This can be
be aa__ pathologic
pathologic or
or physiologic
physiologic process
process
which
which is
is describedas
describedas the increase
the increase inin’ the number
the number of
of cells in
cells in
anorgan.
anorgan.

Ns
A. Hypertrophy
Hypertrophy
Se
B. Hyperplasia
Hyperplasia
(oF
C. Atrophy
Atrophy
DyD. Metaplasia
Metaplasia
E.E. Anaplasia
Anaplasia
Hyperplasia
Hyperplasia
1. An example of a tissue or organ composed of Permanent Parenchymal cells is:
a) Liver
b) Bone Marrow
c) Small Intestinal Mucosa
d) Heart
e) Renal Tubules
The type of necrosis that occurs in Peripancreatic tissue in Acute Pancreatitis is :
a) Liquefaction
b) Fat
c) Coagulation
d) Gummatous
e) Fibrinoid
Change of Columnar Epithelium of the Bronchi into Mature Squamous Epithelium is called:
a) Metaplasia
b) Dysplasia
c) Hyperplasia
d) Neoplasia
e) Hypertrophy
Metastatic Calcification:
a) Is due to hypercalcemia
b) Is due to hypocalcemia
c) Occurs in Necrotic Tissue
d) Occurs at sites of Chronic Inflammation
e) Is due to malignancy
Which of the following is a feature of Irreversible Cell Injury:
a) Glycogen stores are depleted
b) Cytoplasmic sodium increases
¢) Nuclei undergo karyorrhexis
d) Intracellular pH diminishes
e) Blebs form on cell membranes
. An Amputated lower limb from a diabetic patient showing black discoloration of the skin and sof
issues with areas of yellowish exudates is characterized as:
a) Neoplasia
b) Gangrenous Necrosis
c) Coagulopathy
d) Hemosiderosis
e) Gas gangrene
PS er oe Te il a ne nt a
. In a 60 year old male, Gangrene of toes is most likely associated with:
a) Diabetes Mellitus
b) Heart Failure
c) Blunt force trauma
d) AIDS
e) Type III hypersensitivity reaction
0. The presence of Columnar Epithelium with Goblet cells in the Lower Oesophagus is most
onsistent with:
a) Dysplasia
b) Hyperplasia
c) Carcinoma
d) Ischaemia
e) Metaplasia
Which of the following processes explain the appearance of Calcium deposition in Tuberculous
mph Nodes:
a) Dystrophic calcification
b) Apoptosis
c) Hypercalcaemia
d) Metastatic calcification
e) Neoplastic change
Lipofuscin deposition is most likely to result from:
a) Nuclear pyknosis
b) Myocardial fiber hypertrophy
c) Coagulative necrosis
. Which of the following cellular changes is most likely to present irreversible cellular injury:
a) Epithelial dysplasia
b) Cytoplasmic fatty metamorphosis
c¢) Nuclear pyknosis
d) Atrophy
e) Anaerobic Glycolysis
. Which of the following is an Anti-Oxidant:
a) Glutathione Peroxidase
b) Catalase
c) Hydrogen peroxide
d) NADPH oxidase
e) Myeloperoxidase
Which of the following is the most likely pathologic alteration following occlusion of the left middle
ebral artery by a sterile thrombus:
a) Cerebral softening from liquefactive necrosis
b) Pale infarction with coagulative necrosis
c) Predominantly the loss of glial cells
d) Recovery of damaged neurons if the vascular supply is re-established
e) Wet gangrene with secondary bacterial infection
Which of the following type of necrosis is most commonly associated with ischaemic injury:
a) Coagulation Necrosis
b) Liquefaction Necrosis
c) Caseous Necrosis
d) Fat Necrosis
e) Gangrenous Necrosis
Enzymes involved in the inactivation of free radicals include:
a) Superoxide dismutase
b) Vitamin E
c) Both A and B
d) Neither A nor B
e) a-antitrypsin
Hepatocytes are an example of:
a) Permenant cells
b) Stable cells
c) Labile cells
d) Metaplasia
e) Mesenchymal cells
Calcium may play a role in cell injury by:
a) Causing ATP depletion
b) Activating phospholipases
c) Inducing autophagocytosis
d) Regulating pyknosis
e) Reducing intracellular pH
An area of keratinizing squamous epithelium lining a major bronchus is an example of:
a) Heterotopia
b) Metaplasia
c) Dysplasia
d) Atrophy
e) Neoplasia
The enzymes responsible for liquefaction in an abscess are derived mainly from:
a) Tissue
b) Serum
c) Lymph
d) Neutrophils
e) Lymphocytes
Intracellular system(s) sensitive to cell injury is/are:
a) Aerobic respiration
b) Maintenance of the integrity of cell membrane
c) Synthesis of protein
d) Integrity of genetic apparatus
e) All of the above
Which of the following can undergo apoptosis:
a) Cells infected with virus
b) Cells with DNA damage
c) Increased oxidant within the cell
d) None of the above
e) All of the above
Apoptosis:
a) Massive necrosis
b) Foreign body phagocytosis
c) Lipoprotein synthesis
d) Programmed cell death
IVE JO ViTeric
Enzymes involved in the inactivation of free radicals include:
a) Superoxide dismutase
b) Vitamin E
c) Both A an B
d) Neither A nor B
e) a-antitrypsin
The following is true about hyperplasia except:
a) Can be physiological
b) Is a precancerous condition
c) Is reversible
d) Is due to excess hormone stimulation
e) Can be associated with hypertrophy
Early clumping of nuclear chromatin is most closely associated with
a) Reduced intracellular pH
b) Increased intracellular pH
c) Denatured proteins and RNA loss
d) Decreased DNA synthesis
e) Release of lysosomal enzymes
UOTIDJUT SVIYALIOUUOD
BLIOSSION UUM UBWIOM pyO IRdK 6] B Ul 9qny URIdOT]R] Ia] JY] JO ssaosqy (a
UONIILJUI [BIGIIIO JUDO B YIM ULUIOM P]O IBdA ZO (P
SNI[[OUL SOJIQRIP YIM ULUIOM P]O IRDA (CC B UI AJIUUDI]X9 JAMO] SNOUDISURD (9
sIsopnoioqny Areuowuynd YM UPUI plo ed | p B UI BUOTNURIS SuTTeOH CT
BIWUDOTeO.IOdAY
WY VUIOUIOIVS JSVIIQ WIOI] SOSEISLIOUI YIM UPLIOM pyO IBdA C¢ (PB
[BUI UIIS SI UOTBVII]VD dYydo.1sAqg Jo sjduivx9d 3soq YL
Which of the following will show hypertrophy:
a) The uterine myometrium in pregnancy
b) The female breast at puberty
c) The liver following partial resection
d) The ovary following menopause
e) The cervix with chronic inflammation
Which of the following is most likely to give rise to Metaplasia:
a) Tanning of the skin following sunlight exposure
b) Lactation following pregnancy
c) Vitamin A deficiency
d) Acute Myocardial Infarction
Which of the following is deposited in myocardium in advanced age:
a) Hemosidrin
b) Lipochrome
c) Glycogen
d) Cholesterol
e) Calcium Salts
Which of the following tissues is most likely to be least affected by Ischemia:
a) Skeletal muscle
b) Small intestinal epithelium
c) Retina
d) Myocardium
e) Hippocampus
An amputated foot of a diabetic will most likely show:
a) Neoplasia
b) Gangrenous Necrosis
c) Coagulopathy
d) Hemosiderosis
e) Caseation
Which of the following processes explains the appearance of calcium deposition i1

a) Dystrophic calcification
b) Apoptosis
c) Hypercalcemia
d) Metastatic calcification
e) Neoplastic change
Which of the following types of necrosis is grossly opaque & "chalk" white:
a) Coagulation necrosis
b) Liquefaction
c) Caseous necrosis
d) Fat necrosis
e) Gangrenous necrosis
ee ee SAD
Hypoplasia is due to:
a) Cell Loss
b) Atrophy
¢) Inadequate development
d) Disuse
e) Ischemia
The loss of individual cell through fragmentation of individual cell nucleus is best described as:
a) Coagulative Necrosis
b) Mitochondrial Poisoning
c) Phagocytosis
d) Apoptosis
e) Liquefaction
The light brown perinucleur pigment seen in an old man muscle fiber is:
a) Hemosidrin
b) Lipofuscin
c) Glycogen
d) Melanin
e) Calcium
Which of the following is significant in Reperfusion Injury:
a) Generation of Free Radicals
b) Decreased pH
c) Cytokines released by damaged hypoxic cells
d) BothA & B
e) BohA & C
Reperfusion Injury is due to:
a) Generation of Oxygen Free Radicals
b) Additional recruitment of polymorphs by cytokines from hepatic cells
¢) BothA & B
d) None of the above
A cardinal sign of Inflammation is:
a) Vasoconstriction
b) Redness
c) Leukocyte Margiation
d) Vasodilation
e) Slowing of the Circulation
After initiating an acute inflammatory event, the third in a sequence of changes in vascular flow is :
a) Vasoconstriction
b) Redness
c) Leukocyte Margiation
d) Vasodilation
e) Slowing of the Circulation
By definition, which cells are involved in organization:
a) Kidney tubular and Liver Parenchymal cells
b) Macrophages and Endothelial Cells
¢) Endothelial Cells and Fibroblasts
d) Fibroblasts and Fat Cells
Phagocytosis is enhanced by:
a) C5a
b) Bradykinin
c) Lysozyme
d) C3b
e) Serotonin
The tumor of inflammation is due primarily to:
a) Arteriolar Dilation
b) Venous Dilation
c) Capillary Dilation
d) Increased Intracellular Fluid
e) Increased Extracellular Fluid
mediator of acute inflammation that causes increased vascular permeability and pain is
a) Endotoxin
b) Complement
c) Histamine
d) Bradykinin
Multinucleated Giant Cells of the foreign body type origin from:
a) Nuclear Division of Granulocytes
b) Atypical Regeneration of Epithelium
c) Megakaryocytes
d) Fusion of Macrophages
e) Multiplication of Nuclei Surrounding Fibrocytes
. Histamine is thought to be the direct cause of:
a) Leukocytosis
b) Emigration
c) Phagocytosis
d) Increased Vascular Permeability
e) All of the Above
Phagocytes from an individual deficient in myeloperoxiase (MPO) will most likely be less effective

a) Killing Bacteria
b) Recognizing Bacteria
c) Marginating in vessels
d) Undergoing Chemotaxis
e) Engulfing Bacteria
A purulent exudates is generally characterized by the presence of:
a) Mucous
b) Macrophages and connective tissue
¢) Neutrophils and necrotic debris
d) Serous Fluid
e) Precipitated Protein
hich of the following best describes a granuloma:
a) A small nodule of granulation tissue
b) Atumor composed of granulocytes
c) Asmall hard mass of fibroblasts and collagen
d) Inflammation primarily composed of lymphocytes
e) An aggregate of activated macrophages
. The emigration and accumulation of neutrophils in the acute inflammation reaction is
result of:
a) Active hyperemia
b) Hydrostatic Pressure
£) Increased Microvascular Permeability
d) Chemotaxis
e) None of the above
. Which of the following is expressed on endothelial cells in acute inflammation for leukocyte
esion:
a) Interferon Gamma
b) Hageman Factor
c) Lysozyme
d) E-selectin
e) Prostacyclin
Fever and Leukocytes in which most leukocytes are neutrophils typical of:
a) Acute Bacterial Infection
b) Acute Viral Infection
c) Chronic Viral Infection
d) Parasitemia
e) Foreign Body Giant Cell Reaction
Epitheloid Cells and Langerhans Giant Cells in granulomas are derived from:
a) Neutrophils
b) Eosinophils
c) Mast Cells
d) Macrophage
e) Lymphocytes
Prostaglandins are formed from Arachidonic Acid through the action of which enzyme pathway ?
a) Cyclooxygenase
b) Lipoxygenase
c) Myeloperoxidase
d) Phospholipase A
e) Glutathione Reductase
128. Which of the following cell types is most likely to be most characteristic of a foreign body
reaction:
a) Mast cell
b) Eosinophil
¢) Giant cell
d) Neutrophils
e) Plasma cell
129. Which of the following is needed for anti-microbial killing during Phagocytosis:
a) Glutathione peroxidase
b) C3b
c) Interleukin |
d) NADPH oxidase
e) Myeloperoxidase
eficiency of which of the following will reduce phagocytosis by neutrophi
a) C3a
b) Leukotriene A4
eo) C36
d) Leukotriene B4
e) CSa
}. Apoptosis does not induce inflammation because:
a) The injury is too mild to induce inflammation
b) Phagocytic cells secrete cytokines that inhibit inflammation
c) The process occurs only in avascular tissues
d) Apoptosis occurs only in immune deficient tissues
e) The patient is using antibiotics
. Osponization is the:
a) Formation of free radicals
b) Degradation of bacteria by lysozymes
c) Engulfment of antigen by leukocytes
d) Coating of antigen by antibodies
e) Processing of antigens by antigen presenting cells
Chronic Inflammation is characterized by the presence of:
a) Polymorphs, monocytes and exudate
b) Plasma cells, lymphocytes, fibroblasts and monocytes
c) Plasma cells, polymorphs, lymphocytes and monocytes
d) Plasma cells, basophils, eosinophils, fibroblasts and monocytes
The following are true of macrophages except:
a) They are phagocytic cells
b) They produce interleukins
c) Are antigen-presenting cells
d) They are derived trom blood monocytes
e) They are the main cells in chronic inflammation
Which of the following is true about an Exudate:
a) Occurs in heart failure
b) Is due to low plasma oncotic pressure
c) Has a low specific gravity
d) Has a high protein content
e) Is due to lymphatic obstruction
. Infertility and short stature in a woman with a 45x karyotype are typical of:
a) Rlinefelter Syndrome
b) Turner Syndrome
c) Down Syndrome
d) Achondroplasia Dwarfism
e) Cystic Fibrosis
soyeuu ATUO sjoaJTV (9
DAISSOIOY [BULOSOINY (p
JOYJOU dy} WoT poysoyuy TF
aSvaSIPJO aSNevd UOWIUUOD (q
Joye] OY} WIOL poysoyuy (e
[318 SUIS [VLIPUOYIOJW UI SUONLINIA
pJO SIVA (VE ULY] SSI] IB SIOYIOPY (9
JUBUILUOG [ePWUOsSO NY (p
Cc] AWIOSII] (9
8] Atos] (q
[7 Awosiy (t
‘SI JUIOIPUAS UMOG
dAOQGK OY} JO SUON (5
SIOPIOSIP JURUILUOp [BUTOSOINK ULY] IIDAQS OO ATTeNSN ory (P
SUOT]EINU MOU 0} SNP ATUOUUUUOD Pd}LIOYUl ory (9
A]UO JdYJOU dy} WOT poyiayuUl ary (q
[| A]JUO J9Y LI DY} WIOI, poyoyUr ary (e
SS1IPAOSIP IAISSIIOA [BUTOSOINY *
All of the following are autosomal recessive except;
a) Cystic Fibrosis
b) Sickle Cell Anemia
c) Thalassemia
d) Hemochormatosis
e) Diabetes Mellitus
Mendelian disorders are due to:
a) Chromosomal abnormalities
b) Single gene defect
c) Environmental Factors
d) Mitochondrial gene mutations
e) X-linked mutations
The primary role of which of the following in the closure of wounds healing by second in
a) Neutrophils
b) Connective Tissue
c) Macrophages
d) Granulation Tissue
e) Granuloma
. The process of regeneration:
a) Does not restore prior function
b) Invariability leads to scar formation
c) Refers to healing by proliferation of stromal elements
d) Occurs in tissues composed of labile and stable cells
e) Is synonymous with repair
The following factors delay healing of damaged tissue:
a) Scurvy
b) Ionizing Radiation
c) Immobilization
d) Infections
e) Glucocorticoids
Which of the following does not promote fracture healing:
a) Steroid therapy
b) Adequate blood supply
c) Immobilization
d) Adequate Vitamin C supply
The causes of Hypovolemic shock do not include:
a) Severe vomiting
b) Severe diarrhea
c) Extensive burning
d) Myocardial Infarction
e) Severe Trauma
rombosis can be caused by all of the following
a) Endothelial injury
b) Turbulent blood flow
c) Leiden mutation
d) Factor VIII deficiency
e) Atrial Fibrillation
NRRU RIE Sy
NRRU RIE Sy
9) which of the following conditions will cause edema?
Decrease in plasma proteins

10) newborn presents with a prominent acciput, micrognathia, low set


ears and rodenbottom feet genetic d/o caused by nondisjunction is
called?
Edwards syndrome

11)Which among the following is involved in staging a particular


malignancy except
Size of the mass

12) the kidneys shown in this image are from 24-year-old man. What
would have been most likely cause of his death?
Metastatic renal cell carcinoma

13) A 58 year old male, hypertensive was rushed to the Emergency


Room due to loss of consciousness. His son recalled that his father
suddenly complained of very severe headache after their breakfast
followed by loss of consciousness. What is the most probable
diagnosis?
Subarachnoid hemorrhage

14) Baretts esophagus is what type of abnormal cell/tissue change?


Metaplasia

15) this is considered an irreversible histologic manifestation of cellular


injury
Nuclear pyknosis

16) membrane proteins that recognize a variety of microbe-derived


molecules and stimulate innate immune responses against the
microbes
Toll like receptors

17) what type of substance accumulates in Niemann pick disease Type


C?
Cholesterol

18) anti-centromere antibodies are present in


CREST syndrome

19) in aseptic meningitis


The most commonly identified agent is an enterovirus

20) metaplasia
Is the process that occurs in Barett’s esophagus
21) choristomas
Collection of normal cells or tissues in abnormal locations

22) which of the following is not an immune complex mediated


disease?
Poststreptococcal glomerulonephritis

23) during the acute inflammatory process, integrins attach to:


Receptors on neighboring leukocytes

24) 12 year old male was brought to your clinic due to pruritic, pink to
skin-colored papules with central umbilication. What is your primary
differential diagnosis?
Molluscum contagiosum

24) the sequence of cellular events in inflammation is:


Chemotaxis-margination-diapedesis-phagocytosis

25) true of cancer cachexia:


Weight loss results more from loss of muscle than of fat

26) macrophages may secrete


Oxygen free radicals

27) which of the following karyotypes is associated with the classic


pattern of Klinefelter syndrome?
47, XXY

28) hallmark of malignant transformation


Rapid growth

29) Dystrophic calcification


Is formed by crystalline calcium phosphate mineral

30) A 55-year-old man develops and an acute myocardial infraction


because of sudden occlusion of the left anterior descending coronary
artery. The areas of myocardial necrosis within the ventricle can best be
described as
Fibrinoid necrosis

31) various experimental models states that the most important way of
promoting cellular longevity is through caloric restriction. Which of the
family of proteins is responsible for such affect?
Sirtuins

32) Glanzmann thrombasthenia is a deficiency of?


Gpllb-illa complex

33) a benign neoplasm characterized by nests of melanocytes along the


epidermal basal layer in the papillary dermis is diagnostic of which of
the following?
Compound nevus

34) A 5 year old boy was admitted for high grade fever with nuchal
rigidity on PE. CSF likely examination revealed moderate pleocytosis
with lymphocyte predominance, with increased protein and decreased
glucose. What is the most likely diagnosis
Acute bacterial meningitis
Pathology Qbank Questions

1. Which of the following is best stained by Congo red stain?


a. Mucin
b. Fat
c. Amyloid
d. Glycogen

2. White infarcts occur in:


a. Kidney
b. Small intestine
c. Sigmoid colon
d. Lung

3. On day 28 of her menstrual cycle, a 23-year-old woman experiences onset of


menstrual bleeding that lasts for 6 days. She has had regular cycles since
menarche. Which of the following processes most likely occurs in her
endometrial cells to initiate the onset of menstrual bleeding?
a. Atrophy
b. Apoptosis
c. Liquefactive necrosis
d. Caseous necrosis

4. A 15-year-old girl incurs a cut to the sole of her foot after stepping on a piece
of broken glass. On examination, a superficial 0.5-cm laceration ceases to
bleed within 5 minutes after application of local pressure. Which of the
following substances is released by endothelium and is most likely to
counteract platelet aggregation near this site of injury?
a. Glycoprotein IIb/IIIa
b. Thromboxane
c. Prostacyclin
d. Thrombomodulin

5. You’re almost likely to see liquefactive necrosis in


a. Brain
b. Tuberculous lung
c. Gangrenous diabetic foot
d. Myocardium

6. What is the name of the protein-rich extravascular fluid produced in


inflammation?
a. Pus
b. Blood
c. Transudate
d. Exudate
7. Coagulation necrosis is commonly seen in
a. Heart
b. Pancreas
c. Autoimmune diseases
d. Brain

8. “Nutmeg Liver” is seen in


a. Liver necrosis
b. Chronic passive congestion
c. Thrombosis of the portal veins
d. Haemorrhage due to liver trauma

9. Cellular changes seen in apoptosis include all of the following, EXCEPT


a. Chromatic condensation
b. Formation of cytoplasmic blebs and apoptotic bodies
c. Cell shrinkage
d. Cell swelling

10. A pathologist examines a renal biopsy from a 45-year-old man with nephrotic
syndrome and requests a Congo red stain to confirm the nature of amorphous
acidophilic extracellular hyaline substances localized within the mesangial
matrix of the glomeruli. A positive test confirms the presence of
a. Amyloid
b. Glycogen
c. Hemosiderin
d. Copper

11. Which is NOT evidence of irreversible cell injury?


a. Acute cell swelling
b. Nuclear pyknosis
c. Rupture of the lysosomes
d. Karyolysis

12. A 50-year-old policeman has been seen by his family physician for a 5-year
history of “heartburn.” He has been intermittently taking ranitidine, a
histamine-2 blocking agent, with some relief. An upper endoscopic
examination that was performed recently revealed some reddish discoloration
and friability of the lower esophageal region. A biopsy of the lower esophagus
was performed, and the microscopic examination revealed columnar cells
containing goblet cells. What is the most likely mechanism of esophageal
changes?
a. Atrophy
b. Hypertrophy
c. Metaplasia
d. Hyperplasia

13. A 32-year-old man is involved in a vehicular accident and sustains fractures of


the right femur and tibia and the left humerus. The fractures are stabilized
surgically. He is in stable condition for 2 days, but then suddenly becomes
severely dyspneic. Which of the following complications from his injuries is
the most likely cause of his sudden respiratory difficulty?
a. Athero emboli
b. Aminonic Embolism
c. Fat embolism
d. Right Hemothorax

14. Which of the following substances are main chemotactic factors for
neutrophils?
a. Thromboxane A2
b. IL-8, Leukotriene B4, C5a
c. Postacyclin
d. TNF and IL1

15. What is the most common cause and site of origin for pulmonary emboli?
a. Deep venous thrombosis
b. Mesenteric vein obstruction
c. Long bone trauma
d. Right ventricle stasis

16. In an experiment, thrombus formation is studied in areas of vascular damage.


The propagation of a thrombus in an area of vascular injury to adjacent normal
arteries is prevented. Which of the following substances diminishes thrombus
propagation by activating protein C?
a. Platelet factor 4
b. Thrombomodulin
c. Tumor necrosis factor (TNF) 13
d. Prothrombin

17. Petechiae is best defined as


a. Haemorrhage into the thoracic cavity
b. Subcutaneous haemorrhage measuring 1-2 cm
c. Skin Haemorrhage appearing as minute spots measuring 1-2 mm
d. Subcutaneous edema in association with heart failure

18. A 12-year-old boy with a 1-day history of sore throat was seen by his GP. On
physical examination, the most prominent finding was a purulent pharyngeal
exudate. He most likely has an acute inflammation. In acute inflammation
which of the following is true?
a. The predominant cell type is neutrophil polymorph
b. Lymphocytes are present at the start of the process
c. Plasma cells are frequently present
d. The duration may be for months

19. A 65-year-old man develops worsening congestive heart failure 2 weeks after
an acute myocardial infarction. An echocardiogram shows a markedly
decreased ejection fraction. Now, capillaries, fibroblasts, collagen, and
inflammatory cells have largely replaced the infarcted myocardium. Which of
the following inflammatory cell types in this lesion plays the most important
role in the healing process?
a. Plasma cells
b. Macrophages
c. Eosinophils
d. Neutrophils
e. Endothelioid cells

20. Which of the following complement components are involved in opsonization?


a. C1
b. C3b
c. C5a
d. C5-9

21. You’re most likely to see caseous necrosis in


a. Brain
b. Gangrenous diabetic foot
c. Tuberculous lung
d. Myocardium

22. A 73-year-old is rushed to the Emergency Room following a collapse at home.


Despite resuscitation efforts, he dies. Five weeks ago, he had suffered a
myocardial infarction. What would be the most likely histological findings on
his heart?
a. Coagulative necrosis
b. Macrophage and T cell infiltration
c. Liquefactive necrosis
d. Fatty necrosis
e. Contracted scar

23. Which of the following is NOT an action of Leukotrienes?


a. Suppression of inflammation by inhibiting neutrophil chemotaxis
b. Increased permeability of venules
c. Bronchospasm
d. Intensive vasoconstriction
24. Which of the following is frequently seen with autoimmune disorders?
a. Coagulative necrosis
b. Liquefactive necrosis
c. Fatty necrosis
d. Fibrinoid necrosis
e. Gangrenous necrosis

25. Which of the following is true of amyloidosis?


a. Has no systemic effects
b. Amyloid refers to the abnormal fibrous extracellular, proteinaceous
deposits found in organs and tissues
c. Amyloid does not take up Congo red stain
d. The most common amyloid in reactive systemic amyloidosis is the AL
type

26. Which of the following is false about lipofuscin?


a. Lipofuscin id not injurious to the cell or its function. Its importance lies
in its being a telltale sign of free radical injury and lipid peroxidation
b. Lipofuscin is an insoluble pigment, also known as lipochrome or wear-
and-tear pigment
c. Lipofuscin is a haemoglobin-derived, golden yellow-brown, granular, or
crystalline pigment
d. Lipofuscin is composed of polymers of lipids and phospholipids in
complex with protein

27. Which is NOT true regarding Bernard soulier syndrome?


a. Bernard soulier syndrome is a autosomal recessive platelet disorder
b. Inherited deficiency of GpIIb-IIa results in a bleeding disorder called
Bernard soulier syndrome
c. Affects males and females equally
d. Deficiency of GPIb receptor for vWF causes impaired platelet adhesion

28. A 2-year-old girl of Ashkenazi Jewish descent is being evaluated for mental
retardation, seizures, and ataxia. Physical examination finds enlargement of
the liver, spleen, and several lymph nodes. A bone marrow biopsy reveals
aggregates of lipid-laden macrophages (foam cells). These cells contain
excessive amounts of sphingomyelin and cholesterol. Further workup finds a
deficiency of the enzyme sphingomyelinase. Which of the following is the most
likely diagnosis?
a. Gaucher disease
b. Tay-Sachs disease
c. Niemann-Pick disease
d. Fabry disease
29. Which of the following statements about integrins are correct?
a. TNF and IL-1 induce endothelial expression of ligands and integrins
b. The initial rolling interactions are mediated by intergrins
c. These are low-affinity with a fast off-rate, so they are easily disrupted by
the flowing blood.

30. Which statement concerining Keloid is true?


a. In contrast to normal or hypertrophic scars, keloidal tossue do not
extends beyond the initial site of trauma
b. Keloids tend to regress over several months
c. Keloid result from abnormal wound healing in response to skin trauma
or inflammation
d. Lower incidences are seen in darker skinned individuals of Africa, Asia
and Hispanic descent

31. Which of the following is FALSE regarding anti-phospholipid syndrome?


a. Patients at risk of repeated miscarriages
b. Patients are at risk of recurrent venous thrombosis and pulmonary
emboli
c. Patients at risk of abnormal bleeding
d. Can be seen in association with systemic lupus erythematosus
e. Can have false positive serological test for syphilis

32. Which of the following is the mechanisms of edema in patients with


congestive heart failure?
a. C. increased hydrostatic pressure
b. Decreased plasma oncotic pressure
c. Increase vascular permeability
d. Lymphatic obstruction

33. Apoptotic cells usually exhibit distinctive morphological features. Which of


the following is true of pure apoptosis?
a. Results in cell shrinkage and fragmentation
b. Is only a pathological process
c. DNA is cleaved by proteases
d. Results in an inflammatory response

34. Which of the following relationships is NOT correct?


a. Fat necrosis à pancreatitis and trauma to the breast
b. Liquefactive necrosis à tuberculosis
c. Coagulative necrosis àischemia
d. Fibrinoid necrosis à autoimmune disease
35. Which of the below is true regarding Serous inflammation?
a. Serous inflammation is marked by the exudation of cellpoor fluid
spaces created by cell injury or into body cavities lined by peritoneum,
pleural or pericardium
b. Serous inflammation is characterized by the production of pus
c. Serous exudate is characteristic of inflammation in the lining of body
cavities, such as the meninges, pericardium, and pleura

36. A 24-year-old man is involved in a motor vehicle collision and sustains a


fracture of his left femur. While in the hospital, 2 days after the accident he is
noted to be dyspneic and disoriented, and he develops petechiae on his trunk
and axilla. What is the most likely diagnosis?
a. Septic emboli
b. Fat embolism
c. Air embolus
d. Athero Emboli

37. Which of the following ultrastructural features of cell injury are considered
irreversible?
a. Swelling endoplasmic reticulum
b. Cytoplasmic vacuoles
c. Pyknotic nuclei
d. Cell swelling

38. The earliest event in acute inflammation is:


a. Endothelial contraction
b. Fibroblast proliferation
c. Macrophage migration
d. Vasodilation

39. All of the following are true about Reversible Cell Injury except:
a. Swelling of cell organelles and entire cell
b. Dissociation of ribosomes from endoplasmic reticulum
c. Decreased energy production by mitochondria
d. Nuclear degeneration

40. The mechanism of action of Prostacyclin (PGI2) is best described by which of


the following statements?
a. Causes vasoconstriction
b. Stimulates platelet aggregation
c. Causes vasodilation and inhibits platelet aggregation
d. Main chemotactic factor for neutrophils
41. Which one of the following relationships is NOT correct?
a. TNF Stimulates expression of endothelial adhesion molecules and
secretion of other cytokines
b. IL-1 action is Similar to TNF; greater role in fever
c. sustained production of TNF contributes to cachexia
d. TNF is an important mediator of platelet aggregation and
vasoconstriction

42. Which of the following statements about necrosis is correct?


a. Maintains plasma membrane integrity
b. Does not result in an inflammatory response
c. Is an energy-dependant process
d. May be stimulated by oxygen free radicals

43. The histopathology report for a tissue biopsy shows that it is fat necrosis.
Which of the following situation is most likely to exhibit fat necrosis?
a. Brain aneurysm
b. Muscle injury
c. Myocardial infarction
d. Trauma to breast

44. After a 12-hour plane flight, a 53-year-old female flight attendant noticed a
heavy ache over the right calf, with associated swelling and tenderness. Which
of the following is true of thrombosis?
a. Antiphospholipid antibody decreases risk of arterial and venous
thrombosis
b. Malignancy sometimes decreases the risk of venous thrombosis
c. Is predisposed by blood stasis
d. The most common site of venous thrombosis is the portal vein

45. Disorders that predispose to thrombosis include all of the following EXCEPT:
a. Pancreatic carcinoma
b. Pregnancy
c. Vitamin K deficiency
d. Sickle cell anemia

46. A pathologist examines a section of bronchial tissue obtained during a


transbronchial biopsy performed on an individual who smokes. She notes that
the bronchial lining consists of several layers of well-differentiated, organized
squamous epithelia contained above the basement membrane. Which of the
following terms best describes the pathologist’s finding?
a. Anaplasia
b. Dysplasia
c. Hyperplasia
d. Metaplasia
47. Which of the following is true of a dystrophic calcification?
a. When the deposition occurs locally in dying tissues, it is known as
dystrophic calcification
b. The deposition of calcium salts in otherwise normal tissues is an
example of dystrophic calcification
c. dystrophic calcification is almost always results from hypercalcemia
secondary to some disturbance in calcium metabolism.
d. Dystrophic calcification is encountered in normal tissue

48. Which one of the following relationships is NOT correct?


a. keratin filaments are characteristic of epithelial cells
b. desmin filaments are characteristic of muscle cells
c. vimentin filaments are characteristic of connective tissue cells
d. neurofibrillary tangle found in the in-liver cells is characteristic of
alcoholic liver disease

49. A neonate born to 41-year-old woman in her 39th week of gestation has a
flattened face and epicanthal folds, oblique palpebral fissures and a single
palmar crease. The child's echocardiography reveals an endocardial cushion
defect. Which of the following most likely occurred prior to conception?
a. Meiotic non-disjunction
b. Expansion of trinucleotide repeats
c. Deletion of chromosomal part
d. Robertsonian translocation

50. A 15-year-old patient is referred to your office by a teacher who is concerned


about the patient's learning abilities. After evaluating the patient, you diagnose
mild mental retardation. Cytogenetic studies of the patient's buccal mucosal
cells reveal a 47 XXY karyotype. Further evaluation is likely to reveal:
a. Short stature, broad chest, amenorrhea
b. Macroorchidism, large jaw and ears
c. Tall stature, gynecomastia, infertility
d. Short stature, hypotonia, obesity

51. A 14-year-old male is diagnosed with mild mental retardation. Cytogenetic


studies show a small gap near the tip of the long arm of the X chromosome.
Physical examination of this patient is likely to show:
a. Short stature, broad chest, amenorrhea
b. Tall stature, gynecomastia, infertility
c. Short broad hands, transverse palmar crease
d. Macroorchidism, large jaw and ears

52. An infant born prematurely to a 42-year-old Caucasian female is small for


gestational age. Physical examination reveals microcephaly, low-set ears,
prominent occiput, and small mandible. The infant's fists are clenched, and the
fingers overlap. A bilateral foot deformity is observed. Which of the following is
the most likely karyotype abnormality in this infant?"
a. Trisomy 21
b. Trisomy 13
c. Trisomy 18
d. 47, XXY

53. A stillborn fetus delivered at the 23rd week of gestation is found to have an
edematous neck and broad chest. Autopsy reveals aortal coarctation, a
bicuspid aortic valve, and kidneys that are fused at the midline. Which of the
following is the most likely karyotype abnormality in this fetus?
a. Trisomy 21
b. Trisomy 18
c. Trisomy 13
d. 47, XXY
e. 45, XO

54. A 23-year-old Caucasian male with mild mental retardation has large ears, a
long face, a prominent mandible, and large testes. His hand joints are
hyperextensible on physical examination. Which of the following is the most
likely diagnosis in this patient?
a. Fragile X syndrome
b. Marfan syndrome
c. 47,XYY karyotype
d. Klinefelter syndrome

55. An infant born prematurely to a 38-year-old Caucasian female is small for


gestational age. Physical examination reveals a bilateral cleft lip,
microcephaJy, and microphthalmos. Viscera protrude from an umbilical
opening in the child's abdominal wall. Which of the following karyotypes is
most likely in this case?
a. Trisomy 18
b. Trisomy 21
c. Trisomy 13
d. Marfan syndrome

56. A 40-year-old pregnant woman at 12 weeks gestation comes to the emergency


department due to vaginal bleeding associated with lower pelvic pain. On
examination, there is bright red blood in the vagina and the uterus appears
larger than expected given the patient's gestational age. There are no fetal
heart tones audible on Doppler ultrasound. The patient undergoes dilation and
curettage in the operating room, and a friable mass of tissue consisting of
many thin-walled cysts is evacuated from her uterus. Karyotype examination of
the tissue shows it to have a 46 XX genotype. Further analysis would most
likely show nuclear chromosomes derived from which of the following
sources?
a. From the mother only
b. From the father only
c. Half from the father and half from the mother
d. Mostly from the mother

57. An infant born to 38-year-old primigravida develops carpopedal spasms soon


after birth. On diagnostic workup, the infant is found to have an absence of the
thymic shadow on X-ray and narrowing of the aortic arch on angiography.
Which of the following embryonic structure derivatives failed to develop in this
patient?
a. Second branchial cleft
b. Fourth branchial arch
c. Third pharyngeal pouch
d. Third branchial arch

58. During kidney transplantation in a patient with end-stage polycystic kidney


disease, the surgeon notices that the graft becomes cyanotic and mottled soon
after he connects graft vessels with recipient vessels. The blood flow to the
graft eventually ceases, and no urine is produced. Which of the following best
explains the findings observed by the surgeon?
a. Immediate hypersensitivity
b. Antibody-mediated hypersensitivity
c. Immune complex-mediated damage
d. Cell-mediated hypersensitivity

59. Soon after birth, a term infant develops tetany with marked hypocalcemia. This
is treated, but at one month of age, a systolic heart murmur is heard on
auscultation of the chest. Later in infancy, it is noted that the baby has been
almost constantly ill with one infection after another, including respiratory
syncytial virus, Candida, and Pneumocystis carinii (jirovecii) pneumonia
diagnosed. Which of the following primary immunodeficiency disorders is the
child most likely to have?
a. DiGeorge anomaly
b. Severe combined immunodeficiency
c. Common variable immunodeficiency
d. Bruton disease
e. Hyper IgM syndrome

60. An example of type III immune complex disease is:


a. Contact dermatitis
b. Allergies
c. Graft rejection
d. Serum sickness

61. Which of the following statements is true?


a. Adenocarcinoma is a malignant tumor of mesenchymal origin
b. Carcinoma is a neoplasm derived from all three germ cell layers
c. Sarcoma is a malignant tumor of epithelial origin
d. Desmoplasia is a tumor-induced proliferation of non-neoplastic fibrous
connective tissue
62. Which of the following term best describes Choristoma?
a. Are most often named by the tissue of origin
b. This is a small non-neoplastic area of normal tissue misplaced within
another organ, such as pancreatic tissue within the wall of the stomach
c. This benign neoplasm most often arises from surface epithelium
d. This is a non-neoplastic, disorganized, tumorlike overgrowth of cell
types regularly found within an affected organ

63. Which of the following statements regarding smoking is CORRECT?


a. It is a stimulant that increases the ciliary actions of the respiratory
epithelial lining
b. It inhibits the ciliary actions of the respiratory lining and contributes to
obstructive pulmonary disease.
c. It causes lung cancer, but not bladder cancer.
d. It contains particulate matter and irritants, the most dangerous being
carbon dioxide.
e. It is recommended for pregnant women

64. A 50-year-old man presents to the hospital after a brief episode of chest wall
pain. An ECG and cardiac enzyme tests reveal no abnormalities. A routine
chest radiograph reveals only a single 2-cm coin lesion in the right lower lobe
of the lung with a “popcorn” pattern of calcifications. The man is scheduled for
surgery, and the lesion is removed. The pathology report describes the mass
as a disorganized nodule of cartilage, with no cellular features of malignancy.
What is the general term for this lesion?
a. Choristoma
b. Hamartoma
c. Adenoma
d. Sarcoma

65. A 3-year-old child is being evaluated by his physician during an office visit.
Over the past 2 years, the child has had several bouts of ostitis media and one
severe case of pneumonia, which required hospitalization. At the time of the
examination, the physician also notes that the child’s skin is generally very
white. The child’s mother mentions that one of the child’s grandmother’s
sisters died at a young age from recurrent infections. After further laboratory
testing, the physician makes a diagnosis of
a. severe combined immunodeficiency syndrome
b. chronic granulomatous disease
c. hyper-IgM syndrome
d. Chédiak-Higashi syndrome
66. A 9-month-old girl with a history of recurrent pulmonary infections is found to
have a congenital deficiency of adenosine deaminase, which is associated with
a virtual absence of lymphocytes in her peripheral lymphoid organs. What is
the appropriate diagnosis?
a. Bruton X-linked agammaglobulinemia
b. DiGeorge syndrome
c. Isolated IgA deficiency
d. Severe combined immunodeficiency

67. A 20-year-old woman has an ovarian tumor removed. The surgical specimen is
10 cm in diameter and cystic. The cystic cavity is found to contain black hair
and sebaceous material. Histologic examination of the cyst wall reveals a
variety of benign differentiated tissues, including skin, cartilage, brain, and
mucinous glandular epithelium. What is the diagnosis?
a. Adenoma
b. Chondroma
c. Hamartoma
d. Teratocarcinoma
e. Teratoma

68. A 45-year-old woman presents with abdominal pain and vaginal bleeding. A
hysterectomy is performed and shows a benign tumor of the uterus derived
from a smooth muscle cell. What is the appropriate diagnosis?
a. Leiomyosarcoma
b. Myxoma
c. Leiomyoma
d. Rhabdomyoma

69. A 4-year-old girl presents for a preschool physical examination. The child has
a small head circumference, thin upper lip, and low-bridge nose. She shows
evidence of mild mental retardation. Her parents state that she is often
“emotional.” Which of the following maternal causes of birth defects most
likely accounts for these clinicopathologic findings?
a. Cigarette smoking
b. Alcohol abuse
c. Inadequate nutrition
d. Poorly controlled diabetes mellitus

70. All of the following statements about lead poisoning are correct except:
a. The developing brain is highly sensitive to methyl mercury, which
accumulates in the CNS
b. The major source of exposure to mercury is contaminated fish
c. Lead containing water used to irrigate rice fields in Japan caused a
disease in postmenopausal women known as itai-itai
d. Excess lead causes CNS defects in children and peripheral neuropathy
in adults
e. Lead interferes with the remodeling of cartilage and causes anemia by
interfering with hemoglobin synthesis.

71. Marasmus is not characterized by


a. swelling of limbs
b. replacing tissue proteins
c. impaired growth
d. protein deficiency

72. An 8-month-old male infant is admitted to the hospital because of a bacterial


respiratory infection. The infant has no previous history of viral or fungal
infections. The infant responds to appropriate antibiotic therapy but is
readmitted several weeks later because of severe otitis media. Over the next
several months, the infant is admitted to the hospital multiple times for
recurrent bacterial infections. Initial workup at this time finds very low serum
levels of IgG along with an absence of CD19+ cells. Further evaluation finds a
lack of B cells past the pre-B stage. The disorder this boy most likely has is
associated with a defect involving which one of the following?
a. Adenosine deaminase
b. Bruton tyrosine kinase
c. Wiskott-Aldrich syndrome protein
d. IL-2 receptor

73. A 6-month-old boy is being evaluated for failure to thrive, along with persistent
vomiting, seizures, and a low-grade fever. Physical examination finds a
protuberant abdomen due to enlargement of both the liver and spleen, along
with a “cherry-red spot” on his retina and diffuse enlarged lymph nodes. A
bone marrow biopsy reveals an abnormal diffuse proliferation of foamy
macrophages filled with lipid (“foam cells”). Electron microscopy reveals
cytoplasmic bodies that resemble concentric lamellated myelin figures within
the foamy macrophages. Rare parallel palisading lamella (“zebra bodies”) are
also seen in the cytoplasm of these cells with electron microscopy. Which of
the following substances is most likely to be found at abnormally high levels
within these foamy macrophages?
a. Heparan sulfate
b. Glucocerebroside
c. Heparan sulfate
d. Sphingomyelin

74. Paternal uniparental disomy involving chromosome 15 is a possible


mechanism for the development of which one of the listed disorders?
a. Smith-Magenis syndrome
b. Prader-Willi syndrome
c. Angelman syndrome
d. Beckwith-Wiedemann syndrome
75. A 34-year-old Jewish woman presents with weakness and a feeling of
“fullness” in the left side of her abdomen. A medical history finds that she is
Jewish and her family is from the eastern portion of Europe (Ashkenazi Jews).
Physical examination reveals a markedly enlarged spleen, but no neurologic
abnormalities are found. Examination of her retina is within normal limits.
Laboratory examination reveals decreased numbers of red blood cells, white
blood cells, and platelets in the peripheral blood. Elevated levels of
angiotensin converting enzyme (ACE) and acid phosphatase are found in the
peripheral blood. Histologic sections from a bone marrow biopsy reveal
numerous macrophages that have a delicate fibrillar (“crumpled tissue paper”)
cytoplasm due to the accumulation of PAS-positive material. What is the best
diagnosis for this individual?
a. Sandhoff disease
b. Neiman-Pick disease
c. Gaucher disease
d. Tay-Sachs disease

76. A 57-year-old female presents to general gynecology clinic for evaluation of a


pelvic mass. The mass was detected on a routine visit to her primary care
doctor during abdominal palpation. In the office, she receives a transvaginal
ultrasound, which reveals a mass measuring 11 cm in diameter. In the
evaluation of this mass, elevation of which tumor marker would be suggestive
of an ovarian cancer?
a. S-100
b. CA-125
c. Beta-hCG
d. Alpha fetoprotein

77. Slow respiration, slow pulse, and consrtiction of pupil occures due to drug
addiction of
a. Cocaine and heroin
b. Morphine and opium
c. Alcohol and thalidomide
d. Nicotine and caffeine

78. Which of the following is a hallucinogen?


a. LSD
b. Heroin
c. Morphine
d. Codein

79. Wernicke's encephalopathy is atributed to


a. Severe riboflavin deficiency
b. Excessive blood alcohol content
c. thamine deficiency
d. Excessive GGT levels
80. Which of the following features is included in the determination of tumor stage
in this disease?
a. Degree of nuclear pleomorphism
b. Loss of polarity
c. Lymph node involvement
d. Number of mitotic figures
e. Surface ulceration

81. Children 6 to 10 years old in the same community are observed by the local
physician to be doing poorly in school, which has been attributed to
behavioural problems. Their parents state that these children have poor
appetites, complain of nausea, and have frequent headaches. On physical
examination, they have decreased sensation to touch over the lower
extremities. They exhibit loss of fine motor control of movement and have a
slightly ataxic gait. A representative CBC shows haemoglobin of 11.8 g/dL,
haematocrit of 35.2%, MCV of 82 μm3, platelet count of 282,300/mm3, and WBC
count of 4745/mm3. Examination of the peripheral blood smear shows
basophilic stippling of the RBCs. Excessive chronic ingestion of which of the
following substances is most likely to explain these findings?
a. Cadmium
b. Copper
c. Iron
d. Lead

82. An 8-year-old girl with numerous hypopigmented, ulcerated, and crusted


patches on her face and forearms develops an indurated, crater-like, skin
nodule on the back of her left hand. Biopsy of this skin nodule discloses a
squamous cell carcinoma. Molecular biology studies reveal that this patient
has germline mutations in the gene encoding a nucleotide excision repair
enzyme. What is the appropriate diagnosis?
a. Hereditary albinism
b. Li-Fraumeni syndrome
c. Neurofi bromatosis, type I
d. Xeroderma pigmentosum

83. A neonate was noted to have mild growth retardation and facial
dysmorphology. The mother was a known abuser of several substances. This
infant’s problem most likely resulted from maternal intake of which of the
following?
a. Alcohol
b. Cocaine
c. Heroin
d. Marijuana
84. All of the following are the examples of the route through which HIV can be
transmitted from one person to another, EXCEPT?
a. Unprotected sexual contact with an infected person
b. From infected mother to the fetus
c. From the mosquito bite carrying the virus
d. Exposure to contaminated blood and blood products

85. A 34-year-old male experiences low-grade fever and decreased urine output
one week after kidney transplantation. Graft biopsy demonstrates dense
interstitial infiltration by mononuclear cells. Which of the following is the most
likely cause of this patient's current condition?
a. Preformed antibodies against graft ABO antigens
b. Host B cell sensitization against graft MHC antigens
c. Host T cell sensitization against graft MHC antigens
d. Graft B cell sensitization against host MHC antigens

86. A 26-year-old woman presents to the dermatology clinic for fatigue, weakness,
and fevers for the past month. She reports significant weight loss despite
eating a normal diet. She reports that she sunburns very easily and has a facial
rash that is hard to cover with makeup. On physical exam, she has a butterfly
rash with nasolabial sparing on her face, several discoid lesions on her
fingers, and a erythematous rash on her chest in a V-neck distribution. On
laboratory exam, she has a highly positive antinuclear antibody and positive
anti-double-stranded DNA antibody. What is the most likely diagnosis?
a. Systemic lupus erythematosus
b. Sjogren's syndrome
c. Sclerodermia
d. Marfans Syndrome

87. Which of the following genes in BCL gene family is anti-apoptotic?


a. BAD gene
b. BH3 gene
c. BAX gene
d. Bcl-2
e. BAK gene

88. A 70 years-old man with a history of multiple risk factors for coronary artery
disease (CAD) develops acute myocardial infarction. Thrombolytic therapy is
administered to restore coronary blood flow. After this therapy, the myocardial
fiber damage may increase because of which of the following abnormalities?
a. Cytoskeletal intermediate filament loss
b. Mitochondrial swelling
c. Increased free radical formation
d. Decreased intracellular pH from anaerobic glycolysis

89. Which of the following cellular changes is not characteristic for reversible cell
injury?
a. The cytoplasm may contain so-called “myelin figures,” which are
collections of phospholipids
b. Nuclear changes such as Karyorrhexis
c. Cellular swelling
d. Membrane blebs
90. Which of the following cellular adaptation to stress is described below?
,,Replacement of one fully differentiated cell type by another type of cell ''
a. Hyperplasia
b. Dysplasia
c. Metaplasia
d. Atrophy

91. A 12-year-old black man with a known history of sickle cell disease presents to
the emergency department complaining of left upper quadrant pain suggestive
of a splenic infarct. Microscopic examination of the spleen would most likely
reveal
a. Fat necrosis
b. Coagulative necrosis
c. Fibrinoid necrosis
d. Gangrenous necrosis

92. A pathologist notes cloudy swelling, hydropic change and fatty change in the
liver of a patient with a history of alcohol abuse. These morphological changes
are all examples of:
a. Early neoplastic change
b. Hyaline change
c. Patterns of cell death
d. Postmortem artefact
e. Reversible cell injury

93. A circumscribed mass of light yellow crumbly to pasty “cheese-like'' material


associated microscopically with a macrophage response is characteristic of?
a. Caseous necrosis
b. Coagulative necrosis
c. Fibrinous necrosis
d. Gangrenous necrosis

94. Laproscopic examination of the abdomen was performed on a 50 year old


chronic alcoholic man. The surgeon noted digestion of tissue with soap
formation and calcification. Which of the following is this most likely
characteristic of?
a. Coagulative necrosis
b. Caseous necrosis
c. Enzymatic fat necrosis
d. Liquefactive necrosis
e. Apoptosis

95. A patient suffers a stroke and has left sided weakness and paralysis in the
upper extremity. The type of necrosis associated with a well-developed infarct
of the brain is:
a. Coagulative necrosis
b. Enzymatic fat necrosis
c. Liquefactive necrosis
d. Gangrenous necrosis
96. A 39-year-old man has a health screening examination. He has a routine chest
x-ray that shows a 2.5 cm nodule in the right lower lobe. The nodule has focal
calcifications. A wedge resection of the nodule is done. On microscopic
examination the nodule shows caseous necrosis and calcification. Which of
the following processes explains the appearance of the calcium deposition:
a. Excessive ingestion of calcium
b. Apoptosis
c. Vitamin D excess
d. Metastatic calcification
e. Dystrophic calcification

97. An 8-year-old girl with asthma presents with respiratory distress. She has a
history of allergies and upper respiratory tract infections. She also has history
of wheezes associated with exercise. Which of the following mediators of
inflammation is the most powerful stimulator of bronchoconstriction and
vasoconstriction in this patient?
a. Bradykinin
b. Complement proteins
c. Interleukin-1
d. Leukotrienes
e. Tumor necrosis factor-α

98. Which of the following preformed substances is released from mast cells and
platelets, resulting in increased vascular permeability in the lungs of the
patient described in the previous question?
a. Bradykinin
b. Hageman factor
c. Histamine
d. Leukotrienes (SRS-A)
e. Thromboxane A2

99. A 25-year-old machinist is injured by a metal sliver in his left hand. Over the
next few days, the wounded area becomes reddened, tender, swollen, and feels
warm to the touch. Redness at the site of injury in this patient is caused
primarily by which of the following mechanisms?
a. Hemorrhage
b. Hemostasis
c. Neutrophil margination
d. Vasoconstriction
e. Vasodilation

100. Which of these prostaglandins mediates fever?


a. PGI2
b. PGD2
c. PGF2
d. PGE2

101. What is the function of Thromboxane A2?


a. Vasoconstriction and platelet aggregation
b. Vasodilation
c. Clot dissolution
d. Vasodilation and platelet aggregation
102. Which protaglandin mediates pain?
a. PGF2
b. PGD2
c. PGE2
d. PGI2

103. Which of these is not an component of acute inflammation?


a. Neutrophils
b. Hemodynamic changes
c. Chemical mediators
d. Lymphocytes

104. What is the first stage of hemodynamic changes of acute inflammation?


a. Massive vasodilation
b. Neutrophil margination
c. Transient vasoconstriction
d. Increased vascular permeability

105. 65 year old man presents to emergency department with severe and
porolonged chest pain. ECG show ST segment elevations. Soon after
admission patient died. Autopsy showed occlusion of left anterior descending
artery. Which of the following favored occlusion formation?
a. PGI2
b. NO
c. TXA2
d. Bradykinin
e. Vasodilation

106. 20 years old man presents to the physician with increased tendency of
bleeding. Genetic testing has shown a mutation of platelet protein important
for Fibrinogen binding. Which of the following proteins is mutated?
a. Glycoprotein IIb/IIIa
b. Von Willebrand Factor
c. Glycoprotein Ib
d. Platelet Factor 4
e. B2 Integrin

107. 20 years old man presents to the physician with increased tendency of
bleeding. Genetic testing has shown a mutation of platelet protein important
for Subendothelial collagen adhesion. Which of the following proteins is
mutated?
a. Glycoprotein IIb/IIIa
b. Von Willebrand Factor
c. Glycoprotein Ib
d. Platelet Factor 4
e. B2 Integrin
108. Laboratory practitioner is studying the function platelets. He has
observed that addition of certain substance to the platelet medium causes
increased fibrinogen binding to the cells. Which of the following substances is
this practitioner is using?
a. Serotonin
b. ADP
c. Platelet Factor 4
d. Von Willebrand Factor
e. TXA2

109. Deficiency of which of the following proteins will lead to increased


bleeding time?
a. Coagulation Factor VII
b. Coagulation Factor X
c. Prothrombin mutation
d. Glycoprotein Ib
e. Protein C Deficiency

110. Where do most arterial emboli originate?


a. Left atrium or ventricle, aorta, and major arteries
b. Pulmonary vein
c. Deep vein of lower limbs
d. Right atrium, inferior vena cava, and major veins

111. Cholesterol crystals detached from atherosclerotic plaques, tumor cells,


bone marrow emboli, and parts of bullets are all examples of what type of
emboli?
a. Gas emboli
b. Air emboli
c. Solid particle emboli
d. Septic emboli

112. A 32-year-old male patient admits to the ER after a femoral artery


stabbing wound. What type of emboli would you most suspect to occur in this
patient?
a. Gas Emboli
b. Fat Emboli
c. Pulmonary emboli
d. Systemic thromboembolism

113. A 58-year-old woman is brought to the emergency department 4 hours


after vomiting blood and experiencing bloody stools. The patient was
diagnosed with alcoholic cirrhosis 2 years ago. Endoscopy reveals large
esophageal varices, one of which is actively bleeding. Which of the following
best explains the pathogenesis of dilated esophageal veins in this patient?
a. Decreased intravascular oncotic pressure
b. Increased capillary permeability
c. Increased intravascular hydrostatic pressure
d. Vasoconstriction of arterioles
e. Vasodilatation of capillaries

114. A 20-year-old woman presents to the emergency room complaining of


having had a severe headache for 4 hours. Physical examination reveals
numerous small red spots on the extremities and a stiff neck. Her temperature
is 38.7°C. Lumbar puncture returns purulent fluid, with segmented neutrophils
and Gram-negative organisms resembling meningococci. A few hours later, the
patient goes into shock and becomes comatose. Severe endothelial injury in
this patient is primarily mediated by which of the following proteins?
a. α-Fetoprotein
b. IgG
c. Interferon-γ
d. Transforming growth factor-β
e. Tumor necrosis factor-α

115. A 69-year-old man is brought to the emergency room complaining of


visual difficulty and weakness. On physical examination, the patient is aphasic
with a right-sided hemiplegia. Retinal hemorrhages are seen bilaterally. You
suspect that a thromboembolus coursed to the left middle cerebral artery and
smaller emboli traveled to the retinal arteries. Which of the following anatomic
sites is the most likely source for these emboli in this patient?
a. Adrenals
b. Deep leg veins
c. Heart
d. Liver
e. Lungs

116. A 59-year-old man with ischemic heart disease and a history of smoking
complains of increasing shortness of breath. On physical examination, the
patient has swollen legs, an enlarged liver, and fluid in the pleural spaces
(bubbly rales are heard on auscultation). Which of the following hemodynamic
disorders explains the pathogenesis of hepatomegaly in this patient?
a. Arterial thromboembolism
b. Chronic passive congestion
c. Deep venous thrombosis
d. Multiple hepatic infarcts
e. Thrombosis of the hepatic vein

117. A 50-year-old fire fighter emerges from a burning house with third-
degree burns over 70% of his body. The patient expires 24 hours later. Which
of the following was the most likely cause of death?
a. Congestive heart failure
b. Disseminated intravascular coagulation
c. Hypovolemic shock
d. Pulmonary saddle embolism
e. Toxic shock syndrome

118. A 23-year-old woman complains of a recent onset of yellowing of her


skin and increasing abdominal girth. Physical examination reveals jaundice
and ascites. Ultrasound examination of her abdomen demonstrates thrombosis
of the hepatic veins. A liver biopsy discloses severe sinusoidal dilation within
the centrilobular regions. This pathologic finding is caused by which of the
following hemodynamic disorders?
a. Active hyperemia
b. Arterial embolism
c. Hematoma
d. Hemorrhage
e. Passive hyperemia
119. 30 year-old woman presents to her physician with malar rash and joint
pain. Her laboratory study shows increased Antihistone antibodies. Which of
the followins is the most likely cause of her condition?
a. Systemic lupus erythematosus
b. Sogren Syndrome
c. Drug induced lupus-like syndrome
d. Rheumatoid Arthritis
e. Myasthenia Gravis

120. 34 year-old woman was diagnosed with drug induced lupus-like


syndrome. Which of the following drugs might have caused her condition?
a. Hydralazine
b. Rifampin
c. Amiodarone
d. Nifedipine
e. Propranolol

121. Which of the following cells are dominating the early phase of type I
Hypersensitivity reaction?
a. Eosinophils
b. Neutrophils
c. Mast cells
d. B Lymphocytes
e. T Lymphocytes

122. Which of the following is essential for Type III Hypersensitivity reaction?
a. Immune complex formation
b. Antibody binding to cell surface antigen
c. T helper cells activating cell mediated immunity
d. Mast cell degranulation
e. Degranulation of Basophils

123. IgG4-related disease can affect every organ.


a. True
b. False

124. What is Miculicz syndrome?


a. Hyperplasia of the billiary tree
b. Hypertrophic cardiomyopathy and thyroiditis
c. Enlargment and fibrosis of salivary and lacrimal glands
d. Hereditary pulmonary fibrosis

125. Which group of people is most often affected by IgGr-related diseases?


a. Young men
b. Young women
c. Older females
d. Middle-aged and odlder men

126. Which of these drugs proved clinically benefitial for patients with IgG4-
related disease? (Multiple answers)
a. Rituximab
b. Alprazolam
c. Ibuprofen
d. Methotrexate

127. Which of these disease produces systemic fibrosis?


a. Scleroderma
b. SLE
c. Riedel thyroditis
d. Rheumatoid arthritis

128. Grafts exchanged between individuals of the same species are called
a. Autograft
b. Xenograft
c. Allograft
d. Alloplast

129. A 19-year-old college sophomore is referred by his ophthalmologist


because of the finding of ectopia lentis (dislocation of the lens), which has
resulted in visual difficulties that have interfered with his performance on the
varsity basketball team. The patient is very tall, with long limbs and long,
slender, spiderlike fingers. His chest has a “caved-in” appearance, and he also
has a modest degree of scoliosis. A midsystolic “click” is heard, and an
echocardiogram reveals mitral valve prolapse. The most likely diagnosis is
a. Ehlers-Danlos syndrome
b. Fabry disease
c. Hurler syndrome
d. Marfan syndrome

130. A 20-year-old woman has a robertsonian translocation involving


chromosome 21 and a second acrocentric chromosome. What is the theoretic
likelihood of a functional trisomy 21 if one of her ova is fertilized by a normal
sperm?
a. 1 in 1
b. 1 in 2
c. 1 in 3
d. 1 in 4
e. 1 in 1500

131. A 1-year-old female infant is hospitalized for pneumonia. Bacterial


cultures of the sputum have grown Pseudomonas aeruginosa. She has had
two prior hospitalizations for severe respiratory infections. Her mother has
noted that when she kisses her child, the child tastes “salty.” The child has
had weight loss that the mother attributes to frequent vomiting and diarrhea
with bulky,foul-smelling fatty stools. The child is small for her age. Which of
the following critical proteins is altered in this condition?
a. Cystic fibrosis transmembrane conductance regulator
b. Dystrophin
c. a-1,4-Glucosidase
d. Lysyl hydroxylase

132. A newly described neurologic disorder is found to affect multiple family


member in three generations that were available for study. In the first
generation, two sisters and one brother were affected. In the second
generation, all of the children of the first-generation sisters were affected, but
none of the descendants of the first- generation son. In the third generation, all
of the children of the affected second- generation women were affected, but
none of the descendants of the second generation men. The mode of
inheritance exemplified here is
a. autosomal dominant
b. autosomal recessive
c. mitochondrial
d. X-linked dominant

133. As part of a fourth-year elective, a medical student rotating through a


medical genetics service is assigned to counsel a patient who is concerned
about a family history of hypertension. To be properly prepared for the
counseling session, the student reviews course notes on modes of inheritance
of various disorders. Knowledge of which of the following modes of
inheritance is most pertinent to the upcoming discussion with the patient?
a. Autosomal dominant
b. Autosomal recessive
c. Multifactorial
d. X-linked dominant

134. A 2-year-old child has been followed for mental retardation and slow
development, as well as multiple birth defects. The child has a high-pitched
catlike cry. On examination, microcephaly, hypertelorism, micrognathia,
epicanthal folds, low-set ears, and hypotonia are noted. Karyotypic analysis
would be expected to show
a. 5p-
b. 22q11-
c. 45,XO
d. 46,XY

135. The parents of a 17-year-old boy with Down syndrome seek counseling
because they are concerned that their son may develop a life-threatening
disorder known to be associated with his chromosomal abnormality. The
physician should be prepared to discuss which of the following disorders in
terms of its association with Down syndrome?
a. Berry aneurysm of the circle of Willis
b. Creutzfeldt-Jakob disease
c. Lymphoblastic leukemia
d. Medullary carcinoma of the thyroid

136. A 14-year-old girl with amenorrhea is concerned because of the delayed


onset of menses. She has shortened stature and a wide, webbed neck; broad
chest; and secondary sexual characteristics consistent with those of a much
younger girl. Which of the following chromosomal changes is most consistent
with these findings?
a. 5p-
b. 22q11-
c. 45,XO
d. 46,XY

137. A 50-year-old woman of Eastern European Jewish ancestry has a


history of recurrent fractures and easy bruising and is found to have
hepatosplenomegaly and mild anemia. Serum assays reveal elevations of
chitotriosidase and angiotensin-converting enzyme. Assay of cultured
leukocytes most likely reveals marked deficiency of which of the following
enzymes?
a. Glucocerebrosidase
b. a-1,4-Glucosidase
c. Hexosaminidase A
d. a-L-Iduronidase
138. During a routine physical examination, a 41-year-old woman is noted to
have blueblack pigmented patches in the sclerae and gray-blue discoloration
of the ear cartilages. The extensor tendons of the hands exhibit similar
discoloration when she is asked to “make a fist.” On questioning, the patient
vaguely remembers hearing her mother say that the patient had dark
discoloration on her diapers when she was an infant. Her only current
complaint is slowly increasing pain and stiffness of the lower back, hips, and
knees. A urine sample darkens on standing. These findings are characteristic
of a deficiency of which of the following enzymes?
a. Homogentisic oxidase
b. L-Iduronosulfate sulfatase
c. Ketoacid decarboxylase
d. Phenylalanine hydroxylase

139. A screening test for phenylketonuria (PKU) is performed on umbilical


cord blood from a fair-skinned blond, blue-eyed infant born to dark-
complexioned parents. The test is reported as negative, and no dietary
restrictions are imposed. At 1 year of age, the child is seen again, this time
with obvious signs of severe mental retardation, and a diagnosis of PKU is
made. The diagnosis was missed at birth because
a. cord blood is not a good source of fetal blood
b. the screening (Guthrie) test has low sensitivity
c. test should have been performed on maternal blood
d. the test was performed too early

140. A 56-year-old man dies of a 15-year progressive illness characterized by


athetoid movements and deterioration leading to hypertonicity, fecal and urine
incontinence, anorexia and weight loss, and eventually dementia and death.
The disease is known to have an autosomal dominant mode of inheritance and
to be due to an abnormality in a gene on chromosome 4 that is altered by
increased numbers of intragenic trinucleotide repeats. In addition, this
disorder has an earlier onset and is more debilitating in successive
generations, a phenomenon that might be due to
a. a shift from trinucleotide repeats topentanucleotide repeats
b. an increase in the number of trinucleotiderepeats in successive
generations
c. defects in membrane receptors and transport systems
d. imprinting variability in successivegenerations

141. A 54-year-old woman who has been diagnosed with early-stage breast
cancer undergoes surgery for a lumpectomy to remove a small tumor detected
by mammography. The pathology report confirms the early stage of the cancer
and further comments on the fact that there is significant desmoplasia in the
surrounding tissue. The term desmoplasia refers to
a. an irregular accumulation of blood vessels
b. maturation and spatial arrangement of cells
c. metastatic involvement of surrounding tissue
d. proliferation of non-neoplastic fibrous connective tissue

142. A 26-year-old female presents with severe pain during menses


(dysmenorrhea). To treat her symptoms, you advise her to take
indomethacin(COX Inhibitor) in the hopes that it will reduce her pain by
interfering with the production of
a. Bradykinin
b. Histamine
c. Leukotrienes
d. Phospholipase A2
e. Prostaglandin F2

143. A 39-year-old female presents with intermittent pelvic pain. Physical


examination reveals a 4-cm mass in the area of her right ovary. Histologic
sections from this ovarian mass reveal a papillary tumor with multiple,
scattered small, round, laminated calcifications. These structures are most
likely the result of
a. Apoptosis
b. Dystrophic calcification
c. Enzymatic necrosis
d. Hyperparathyroidism
e. Metastatic calcification

144. A 65-year-old woman falls in the street and strikes the back of his head.
Over the next 24 hours, he becomes increasingly somnolent. A head CT scan
shows an accumulation of fluid beneath the dura, compressing the left cerebral
hemisphere. Which of the following terms best describes this collection of
fluid?
a. Congestion
b. Ecchymosis
c. Hematoma
d. Petechiae
e. Purpura

145. A 23-year-old man has a history of increased menstrual blood flow and
frequent nosebleeds. On physical examination, petechiae and purpura are
present on the skin of her extremities. patient has normal partial
thromboplastin time (PTT), prothrombin time (PT), and platelet count, but
decreased von Willebrand factor activity. This patient most likely has a
derangement in which of the following steps in hemostasis?
a. Fibrin polymerization
b. Platelet adhesion
c. Platelet aggregation
d. Prothrombin generation
e. vasoconstriction

146. A 30-year-old man is cutting wood alone in the forest and incurs a deep
cut to his leg from his chain saw. He loses a large amount of blood. He is not
found until the next day. A marked increase in which of the following blood
analytes is most likely to indicate that he has reached an irreversible stage of
shock?
a. Antidiuretic hormone
b. Bicarbonate
c. Catecholamines
d. Lactate dehydrogenase
e. Prothrombin
147. While camping, a 15-year-old boy is bitten by a mosquito. At the site of
the mosquito bite, his skin turns red and a small but noticeable bump forms.
Which of the following terms best describes the color change in the skin?
a. Lividity
b. Ecchymosis
c. Contusion
d. Congestion
e. Hyperemia

148. A 66-year-old male has a long-standing history of congestive heart


failure due to uncontrolled hypertension and has had multiple admissions to
the hospital for treatment of pulmonary edema. During his most recent
admission, he sustains a fatal cardiac dysrhythmia. Which of the following is
likely to be observed in this patient’s lungs at autopsy?
a. Dilated alveolar septal capillaries filled with red
blood cells
b. Fibrotic and thickened alveolar septal capillaries
c. Loss of alveolar septa, resulting in large airspaces
d. Congestion of centrilobular sinusoids
e. Diffuse macrovesicular steatosis of the liver

149. A 71-year-old male with a history of poorly controlled hypertension due


to noncompliance with medications is brought to the emergency room by his
family because of increasing shortness of breath. An X-ray of the chest reveals
bilateral pleural effusions and enlargement of the heart. Given these features,
of the following, which is the most likely causative mechanism for the
condition producing his shortness of breath?
a. Increased hydrostatic pressure
b. Increased vascular permeability
c. Decreased colloid osmotic pressure
d. Lymphatic obstruction
e. Sodium retention

150. A 68-year-old male with poorly controlled hypertension presents to the


emergency room with shortness of breath. A chest X-ray reveals bilateral
pleural effusions. An S3 gallop is heard and crackles (rales) are present in both
lung fields. Which of the following pigments is most likely to be found within
macrophages in his lungs?
a. Bilirubin
b. Hemosiderin
c. Melanin
d. Lipofuscin
e. Calcium

151. A 60-year-old female status post right radical mastectomy and right
axillary lymph node dissection develops chronic edema of the right arm. Which
of the following is the cause of her edema?
a. Increased plasma oncotic pressure
b. Renal sodium retention
c. Increased interstitial protein
d. Increased vascular permeability
e. Decreased plasma albumin
152. A 76-year-old man is evaluated in the emergency department for
hypotension and dyspnea. His temperature is 97.8°F (36.5°C), pulse 125/min,
and blood pressure 88/58 mm Hg. Bilateral crackles are present in the lung
fields. Pedal edema is noted bilaterally. His extremities are cool, and his urine
output is minimal. A plain chest radiograph reveals cardiomegaly and
pulmonary edema. What is the pathophysiology of his shock?
a. Septic shock
b. Anaphylactic shock
c. Cardiogenic shock
d. Hypovolemic shock
e. Distributive shock

153. A pathologist is examining tissue removed from a 57-year-old deceased


male. The tissue has
preservation of normal architecture; however, there is loss of nuclear and
cytoplasmic basophilia. Associated with these changes are abundant
extravasated red blood cells in the tissue. The man had no resuscitation
performed. Of the following, where did this tissue most likely originate?
a. The heart
b. A kidney
c. The spleen
d. The brain
e. A lung

154. A 76-year-old man is found dead at home. Microscopic examination of


samples of the liver and lung reveals centrilobular hemorrhagic necrosis of the
liver and hemosiderin-laden macrophages in the lungs. Of the following, what
additional finding at autopsy would be most likely?
a. Pleural effusion
b. Aortic dissection
c. Deep venous thrombosis
d. Disseminated intravascular coagulation
e. Pericardial tamponade

155. A 40-year-old male is brought to the emergency department by his wife


because of his shortness
of breath. He reports the onset of a cough two days prior with increasing
shortness of breath. On arrival his temperature is 100.5°F (38.0°C), his heart
rate is 135/min, and blood pressure is 88/55 mm Hg. Physical examination is
unremarkable except for tachycardia. The patient is alert and oriented. His
leukocyte count is 18,000/μL, his pCO2 on arterial blood gas is 30 mm Hg, and
serum lactic acid is slightly elevated at 4 mmol/L. A plain chest radiograph
shows a right lower lobe infiltrate. What is the most likely diagnosis?
a. Acute pulmonary embolism
b. Septic shock
c. Cardiogenic shock
d. Anaphylactic shock
e. Acute myocardial infarction
156. A 37-year-old female with a history of deep venous thrombosis during
her first pregnancy and two spontaneous abortions is evaluated in the
emergency department. Forty minutes prior to arrival she experienced the
sudden onset of weakness of the right face, right arm, and right leg. Neurologic
examination reveals flaccid paralysis of the right side of the body with
expressive aphasia without visual field defect. Magnetic resonance imaging
(MRI) of the brain and basic lab work including complete blood count,
prothrombin time (PT), and activated thromboplastin time (aPTT) are ordered in
the emergency department. The treating physician administer tissue
plasminogen activator (t-PA) to the patient according to hospital protocol in
the treatment of acute stroke. Which of the following is true regarding the
mechanism of t-PA?
a. t-PA prevents platelet activation by inhibiting GpIIb-IIIa complex.
b. t-PA prevents platelet activation by inhibiting GpIb receptors.
c. t-PA increases the degradation of fibrin.
d. t-PA binds anti-thrombin III, leading to thrombin inactivation.
e. t-PA binds and inactivates clotting factors.

157. A 17-year-old male sustains a mid-shaft femur fracture after a fall from a
horse. The fracture
is repaired operatively and the patient is discharged. On the second
postoperative day he becomes acutely dyspneic. In the emergency department
he is hypoxic and tachypneic, appears confused, and has a petechial rash on
his neck and anterior thorax. What is the most likely diagnosis?
a. Aortic dissection
b. Fat embolism
c. Venous thromboembolism
d. Acute myocardial infarction
e. Disseminated intravascular coagulation

158. A 32-year-old woman has routine lab work done as part of her annual
physical examination. She returns to the clinic 3 days after her blood draw
complaining of pain in the left antecubital fossa. On examination there is mild
erythema without induration, no palpable fluctuence, and the basilic vein is
tender and palpated as a nodular “cord.” What is the appropriate
management?
a. Anticoagulation with Coumadin
b. Oral antibiotics
c. Warm compresses
d. Topical corticosteroids
e. Incision and drainage

159. A graduate student in an immunology lab is studying a certain cell that


plays a role in adaptive
immunity. Of the following, which cell type might the student be studying?
a. Macrophage
b. Neutrophil
c. Eosinophil
d. Natural killer cell
e. Lymphocyte
160. A graduate student has developed a protein that binds to CD3 and
blocks its interaction with TCR, which is composed of γ and δ subunits. Of the
following, which organ is the student studying?
a. Brain
b. Heart
c. Intestine
d. Liver
e. Kidney

161. A 32-year-old male with a known severe peanut allergy is inadvertently


exposed to peanuts by eating a homemade cookie. Within minutes of exposure
he develops a widespread rash and difficulty breathing, which resolve with a
self-administered epinephrine injection. Which of the following statements is
most characteristic regarding this type of allergic reaction?
a. The immediate reaction is triggered by IgM bound to antigen.
b. The immediate allergic reaction is triggered by activation of eosinophils.
c. This type of allergic reaction is caused by excessive a TH2 response.
d. Prostaglandins do not play a role in this type of reaction.
e. Previous exposure to the antigen is not required for this type of
reaction.

162. A 42-year-old woman presents to her primary care physician


complaining of itching and watery eyes, runny nose, and frequent sneezing.
She denies fever or cough. She experiences similar symptoms every year in
the spring. Which of the following is responsible for her symptoms?
a. Binding of antigen to IgE on mast cell surfaces
b. Recognition of antigen associated with MHC-1
c. Binding of polysaccharide to membrane-bound lectin receptors
d. Destruction of cells coated with IgM
e. Inflammation due to deposition of antigen-antibody complex
163. A blood type O+ infant is delivered to a multiparous blood type O-
mother without prenatal care. The infant is born with severe anemia, jaundice,
and severe edema. What is the underlying mechanism of this disease?
a. Crosslinking of IgE on mast cell surfaces
b. IgG binding to cell surfaces
c. Deposition of antigen antibody complexes in the fetal tissue
d. T-cell mediated cytotoxicity
e. Antibody mediated activation of apoptosis

164. A 32-year-old man hiking in Guatemala is bitten by a rattlesnake. He is


treated with equine antivenin. He recovers; however, 2 weeks afterward, he
develops rash, fever, and polyarthralgia. Laboratory evaluation reveals a
leukocyte count of 3200/μL, a hemoglobin concentration of 16.0 g/dL, a platelet
count of 95,000/μL, and a serum creatinine of 1.8 mg/dL. What is the most likely
diagnosis?
a. Chikungunya fever
b. Lyme disease
c. Serum sickness
d. Malaria
e. Autoimmune hemolytic anemia
165. A 26-year-old Hispanic female presents to her primary care physician
complaining of 2 months of fatigue. She has persistent arthralgias of the distal
and proximal interphalangeal joints of both hands and reports that when the
weather turns cold her hands frequently change colors. Her physical
examination is normal and there is no tenderness or defor mity of the joints of
the hands. A complete blood count is normal and her antinuclear antibody titer
is elevated at 1:90. Which of the following additional tests would confirm the
diagnosis of lupus erythematosus?
a. Positive SS-A
b. Positive SS-B
c. Positive anti-cyclic citrullinated peptide (CCP-IgG)
d. Positive rheumatoid factor
e. Positive anti-Smith

166. A 20-year-old man steps into an elevator full of people with influenza
who are coughing and sneezing. The influenza viral particles that he inhales
attach to respiratory epithelium, and viral infection reduces MHC class I
molecules displayed on these epithelial cells. Which of the following immune
cells is most likely to rapidly destroy the virally infected cells?
a. CD4+ cell
b. Dendritic cell
c. Macrophage
d. Natural killer cell
e. Neutrophil

167. In an experiment, antigen is used to induce an immediate (type I)


hypersensitivity response. Cytokines are secreted that are observed to
stimulate IgE production by B cells, promote mast cell growth, and recruit and
activate eosinophils in this response. Which of the following cells is most
likely to be the source of these cytokines?
a. CD4+ lymphocytes
b. Dendritic cells
c. Macrophages
d. Natural killer cells
e. Neutrophils

168. A 33-year-old man is evaluated for recurrent sinusitis. He has had


episodes of bacterial sinusitis occurring at least four times a year for the past 3
years. Complete blood count is normal, as is a total hemolytic complement
panel (CH50). Serum IgG, IgM, and IgE levels are normal; however, serum IgA
levels are undetectable. Which of the following complications is this patient at
least risk for?
a. Transfusion reaction
b. Pneumonia
c. Parasitic gastrointestinal infection
d. Crohn’s disease
e. Lymphoid malignancy
169. A 30-year-old sexually active woman undergoes a routine physical
examination. There are no abnormal findings. A Pap smear is obtained as part
of the pelvic examination. Cytologically, the cells obtained on the smear from
the cervix show severe epithelial dysplasia (high-grade squamous
intraepithelial lesion). Which of the following therapeutic options is most
appropriate for this woman?
a. Antibiotic therapy
b. Excision
c. Ovarian removal
d. Screening of family members
e. Watchful waiting

170. A 40-year-old man has a positive stool guaiac test during a routine
physical examination. A colonoscopy is performed and a 0.9-cm,
circumscribed, pedunculated mass on a short stalk is found in the upper
rectum. Which of the following terms best describes this lesion?
a. Adenoma
b. Carcinoma
c. Choristoma
d. Hamartoma
e. Hyperplasia

171. A 63-year-old female is being evaluated by her family physician. Over


the past 3 months she has developed a velvety hyperpigmentation of the skin
of the posterior neck and bilateral axilla. She has lost 15 lbs. since her last visit
four months ago and has a body mass index of 23 kg/m2. Routine blood work
is normal except for a hemoglobin level of 8.6 g/dL. Her HbA1c is 5.4%, and a
plain chest radiograph is normal. What is the next step in the evaluation?
a. Bone scan
b. CT chest with contrast
c. Skin biopsy
d. Upper and lower endoscopy
e. Bone marrow aspirate

172. A 4-year-old boy is evaluated by his pediatrician for 1 week of


intermittent hematuria. On examination he appears well developed and well
nourished, except a right flank mass is palpable. A biopsy of themass is
performed, which reveals primitive looking small blue cells with glomeruloid
and tubuloid structures. What is the diagnosis?
a. medulloblastoma
b. Polycystic kidney disease
c. Clear cell sarcoma
d. Rhabdoid tumor
e. Wilms tumor
173. The parents of a 17-year-old female bring her to the emergency room
after she had a brief seizure at home from which she recovered. A CT scan is
performed in the emergency room, identifying a cortical nodule in the left
cerebral hemisphere. A CT scan of the body is also performed, identifying a 3.0
cm mass in the left kidney, which is subsequently biopsied and diagnosed as
an angiomyolipoma. Of the following, which other tumor is this patient at
increased risk for?
a. Medulloblastoma
b. Hepatic neuroblastoma
c. Ewing sarcoma
d. Cardiac rhabdomyoma
e. Retinoblastoma

You might also like

pFad - Phonifier reborn

Pfad - The Proxy pFad of © 2024 Garber Painting. All rights reserved.

Note: This service is not intended for secure transactions such as banking, social media, email, or purchasing. Use at your own risk. We assume no liability whatsoever for broken pages.


Alternative Proxies:

Alternative Proxy

pFad Proxy

pFad v3 Proxy

pFad v4 Proxy